You are on page 1of 154

All India Test Series TEST 3 : PTS-SEC-GEO-01

07 Feb 2021
Total Number of Questions: 100

Correct: 27

Incorrect: 26

Not Attempted: 47

Completed in: 1 h 5 m 14 s

Your Total Score:

36.58 / 200.0 = 18%


 Performance
All the correct answers are marked bold

All the correct answers selected by You is marked by 

All the incorrect answers selected by You is marked by 

1
Q. Consider the following statements: 

1. Rust bowl is a geographic region which has large deposits of poor quality Iron ore. 
2. The Pittsburgh region of the United States and Shikoku Islands of Japan are the
examples of Rust bowl regions. 

Which of the statements given above is/are correct?


Q. निम्नलिखित कथनों पर विचार कीजिए:

1. रस्ट बाउल (Rust bowl) एक भौगोलिक क्षे तर् है जहां खराब गु णवत्ता वाले लौह अयस्क की बड़ी
मात्रा होती है ।
2. सं युक्त राज्य अमे रिका का पिट् सबर्ग क्षे तर् और जापान का शिकोकू द्वीप, रस्ट बाउल क्षे तर् ों के
उदाहरण हैं ।

उपर्युक्त कथनों में से कौन सा/से सही है /हैं ?

A  1 only
केवल 1

2 only
केवल 2

C  Both 1 and 2
1 और 2 दोनों

D  Neither 1 nor 2
न तो 1, न ही 2
Solution : B

Explanation: 
Statement 1 is incorrect: Rust bowl is a geographic region that was formerly a
manufacturing or industrial powerhouse but now facing industrial decline. It is also
known as Rust Belt. 
Statement 2 is correct: The Pittsburgh region of the United States and Shikoku
Islands of Japan are the examples of Rust Bowl regions. 
व्याख्या : 
कथन 1 गलत है : रस्ट बाउल (Rust bowl) एक भौगोलिक क्षे तर् है जो पहले एक विनिर्माण या
प्रमु ख औद्योगिक क्षे तर् था, ले किन अब औद्योगिक गिरावट का सामना करना पड़ रहा है । इसे रस्ट
बे ल्ट (Rust Belt) के नाम से भी जाना जाता है ।
कथन 2 सही है : सं युक्त राज्य अमे रिका का पिट् सबर्ग क्षे तर् और जापान का शिकोकू द्वीप, रस्ट बाउल
क्षे तर् ों के उदाहरण हैं ।
 1 m 37 s

-0.67/2.0 

2
Q. With reference to Hermatypic corals, which of the following statements is/are
correct?

1. They are also known as stony corals. 


2. They can survive in temperature ranges between 25° and 30°C. 

Select the correct answer using the codes given below: 


Q. हर्मेटे पिक कोरल (Hermatypic corals) के सं दर्भ में , निम्नलिखित कथनों में से कौन सा/से सही
है /हैं ?

1. इन्हें स्टोनी कोरल के रूप में भी जाना जाता है ।


2. ये 25 से 30 डिग्री से ल्सियस तापमान में जीवित रह सकते हैं ।
निम्नलिखित कू टों का उपयोग करके सही उत्तर का चयन कीजिए:

A  1 only
केवल 1

B  2 only
केवल 2

C  Both 1 and 2
1 और 2 दोनों

D  Neither 1 nor 2
न तो 1, न ही 2
Solution : C

Explanation: 
Statement 1 is correct: Hermatypic corals are also known as stony corals because
they are reef-building corals. Stony corals and zooxanthellae live in a mutualistic
association. They get their food from zooxanthellae (Single-celled organisms), which
use sunlight for photosynthesis, and in return provide some nutrients as waste and
protection to zooxanthellae from currents and herbivores. 
Statement 2 is correct: To survive, Hermatypic corals need the following
conditions-

 Temperature ranges between 25° and 30°C. 


 Sunlight to produce food. 
 Clear shallow waters less than 80m deep with low turbidity. 
 A narrow range of salinity. 

व्याख्या : 
कथन 1 सही है : हर्मेटे पिक कोरल को स्टोनी कोरल के रूप में भी जाना जाता है , क्योंकि ये भित्ति
बनाने वाले प्रवाल (reef-building corals) हैं । स्टोनी कोरल और ज़ोक्सां थेला (zooxanthellae )
पारस्परिक सहयोग में रहते हैं । ये अपने भोजन को ज़ोक्सां थेला (एकल-कोशिका वाले जीव) से
प्राप्त करते हैं । ज्ञात हो कि  ज़ोक्सां थेला प्रकाश सं श्ले षण के लिए सूर्य के प्रकाश का उपयोग
करता है । स्टोनी कोरल अपशिष्ट के रूप में इन्हें कुछ पोषक तत्व प्रदान करते हैं तथा धारा और
शाकाहारियों से इनकी सु रक्षा भी करते हैं ।
कथन 2 सही है : जीवित रहने के लिए, हर्मैटिपिक कोरल को निम्नलिखित स्थितियों की आवश्यकता
होती है -

 तापमान 25 ° और 30 ° C के बीच होना चाहिए।


 भोजन बनाने के लिए धूप।
 कम मै लापन (turbidity) वाले 80 मीटर से कम गहरे उथले पानी।
 लवणता की एक सं कीर्ण श्रेणी।

 
 2 m 18 s

-0.67/2.0 

3
Q. The MSME Emergency Response Programme, which was seen recently in the
news is supported by:
Q. हाल ही में चर्चा में रहा एमएसएमई आपातकालीन उपाय कार्यक् रम (MSME Emergency
Response Programme), निम्नलिखित में से किसके द्वारा समर्थित है ?

A  World Bank
विश्व बैं क

B  New Development Bank


न्यू डे वलपमें ट बैं क

C  Asian Development Bank


एशियाई विकास बैं क

D  Asian Infrastructure Investment Bank


एशियाई अवसं रचना निवे श बैं क
Solution : A

Explanation: 
The Government of India and the World Bank has signed an agreement to
support MSME severely impacted by the COVID-19 crisis. The World Bank’s MSME
Emergency Response program will address the immediate liquidity and credit needs
of some 1. 5 million viable MSMEs to help them withstand the impact of the current
shock and protect millions of jobs in India. 
व्याख्या : 
भारत सरकार और विश्व बैं क ने COVID-19 सं कट से गं भीर रूप से प्रभावित MSME क्षे तर् का
सहयोग करने के लिए एक समझौते पर हस्ताक्षर किए हैं । विश्व बैं क के एमएसएमई आपातकालीन
उपाय  कार्यक् रम से लगभग 1. 5 मिलियन व्यवहार्य एमएसएमई की तात्कालिक तरलता और
क् रे डिट जरूरतों का समाधान होगा, जिससे उन्हें वर्तमान विपरीत स्थिति का सामना करने और भारत
में लाखों नौकरियों को बचाने में मदद मिले गी।
 28 s

2.0/2.0 

4
Q. With reference to the human settlement pattern, which one of the following
statements is incorrect?
Q. मानव बस्ती पै टर्न (human settlement pattern) के सं दर्भ में , निम्नलिखित कथनों में से कौन
सा गलत   है ?

A  In a linear settlement, houses are located along the roads.


एक रै खिक बस्ती (linear settlement) में , घर सड़कों के किनारे स्थित होते हैं ।

B  In a rectangular settlement, roads cut each other at 90 degrees.


ू रे को काटती हैं ।
एक आयताकार बस्ती (rectangular settlement) में , सड़कें 90 डिग्री पर एक दस

C  Y shaped settlements develop where three roads meet each other.


ू रे से मिलती हैं ।
Y आकार की बस्तियाँ वहाँ बनती हैं जहाँ तीन सड़कें एक-दस

D  In star-shaped settlements, several roads converge at a point.


तारे के आकार की बस्तियों (star-shaped settlements) में , कई सड़कें एक बिं दु पर मिलती हैं ।
Solution : C

Explanation: 
Option (a) is correct: In a linear settlement, houses are located along the roads or
along a railway line, river, canal edge of a valley or along a levee. 
Option (b) is correct: In a rectangular settlement, roads cut each other at 90
degrees. They are found in plain areas or wide intermontane valleys. 
Option (c) is incorrect: T-shaped settlements develop at tri-junctions of the roads
while Y-shaped settlements emerge at the places where two roads converge on the
third road. 
Option (d) is correct: In star-shaped settlements, several roads converge and the
houses built along the roads.
व्याख्या : 
विकल्प (a) सही है : एक रै खिक बस्ती (linear settlement) में , घर सड़कों  या रे लवे लाइन के
किनारे , नदी, घाटी के नहर या बाँ ध के किनारे स्थित होते हैं ।
विकल्प (b) सही है : एक आयताकार बस्ती (rectangular settlement) में , सड़कें 90 डिग्री पर एक
ू रे को काटती हैं । ये   मै दानी क्षे तर् ों या विस्तृ त अं तरापर्वतीय  घाटियों में पाए जाते हैं ।
दस
विकल्प (c) गलत है : T-आकार की बस्तियां सड़कों के त्रि-जं क्शनों (tri-junctions) पर बनती हैं ,
जबकि Y-आकार की बस्तियां उन स्थानों पर बनती हैं जहां दो सड़कें तीसरी सड़क पर मिलती हैं ।
विकल्प (d) सही है : तारे के आकार की बस्तियों (star-shaped settlements) में , कई सड़कें एक
बिन्दु पर मिलती हैं और सड़कों के किनारे घर बने होते हैं ।
 32 s

-0.67/2.0 

5
Q. With reference to the Special Window for Affordable and Mid-Income Housing
fund or SWAMIH investment fund, which was recently in news, consider the
following statements: 

1. This fund will help in completing the construction of stalled projects. 


2. Projects pending before the National Company Law Tribunal are not eligible for
funding. 
3. SBICAP Ventures is an Investment Manager of this fund. 

Which of the statements given above is/are correct?


Q. हाल ही में चर्चा में रहे अफोर्डेबल & मिड-इनकम हाउसिं ग फंड या SWAMIH निवे श फंड हे तु
विशे ष विं डो के सं दर्भ में , निम्नलिखित कथनों पर विचार करें :

1. इस फंड से रुकी हुई परियोजनाओं के निर्माण को पूरा करने में मदद मिले गी।
2. ने शनल कंपनी लॉ ट्रिब्यूनल के समक्ष लंबित परियोजनाएं वित्तपोषण हे तु पात्र नहीं हैं ।
3. SBICAP वें चर (Ventures)  इस फंड का एक निवे श प्रबं धक है ।

उपर्युक्त कथनों में से कौन सा/से सही है /हैं ?

A  1 only
केवल 1

B  1 and 3 only
केवल 1 और 3

C  2 and 3 only
केवल 2 और 3

1, 2 and 3
1, 2 और 3
Solution : B

Explanation: 
Statement 1 is correct: SWAMIH Investment Fund has been formed to complete
construction of stalled, Real Estate Regulatory Authority(RERA)-registered
affordable and mid-income category housing projects which are stuck due to paucity
of funds. However, funding is available only for projects that have completed at least
30% of the construction and further development required. 
Statement 2 is incorrect: Projects undergoing a corporate insolvency resolution
process before the NCLT can be considered for funding through the Special
Window. 
Statement 3 is correct: The Investment Manager of the Fund is SBICAP Ventures,
a wholly-owned subsidiary of SBI Capital Markets. 
व्याख्या : 
कथन 1 सही है : SWAMIH इन्वे स्टमें ट फंड का गठन पूरी तरह से रुके हुए रियल एस्टे ट विनियामक
प्राधिकरण (RERA) पं जीकृत सस्ती और मध्यम आय वर्ग की आवासीय परियोजनाओं को पूर्ण करने
के लिए किया गया है जो निधियों की कमी के कारण अटकी हुई हैं । हालां कि, यह फंडिं ग केवल उन
परियोजनाओं के लिए उपलब्ध है , जहां कम से कम 30% निर्माण कार्य पूरा हो गया है और आगे के
कार्य  की आवश्यकता है ।
कथन 2 गलत है : कॉर्पोरेट इनसॉल्वें सी रिज़ॉल्यूशन के लिए एनसीएलटी (NCLT) के समक्ष लंबित 
परियोजनाओं के लिए  विशे ष विं डो के माध्यम से वित्त पोषण पर विचार किया जा सकता है ।
कथन 3 सही है : फंड का निवे श प्रबं धक SBICAP वें चर्स है , जो एसबीआई कैपिटल मार्के ट् स की पूर्ण
स्वामित्व वाली सहायक कंपनी है ।
 12 s

6
Q. With reference to Sand dunes, consider the following statements: 

1. In Barchans, points or wings are directed towards the wind direction. 


2. Transverse dunes are aligned perpendicular to the wind direction. 
3. Longitudinal dunes form when the supply of sand is poor and wind direction is
constant. 

Which of the statements given above is/are correct?


Q. बालू के टिब्बे (Sand dunes) के सं दर्भ में , निम्नलिखित कथनों पर विचार कीजिए:

1. शाखाओं वाले   टिब्बे (Barchans) में ,  बिं दु या भु जा हवा की दिशा की ओर निर्देशित होते हैं ।
2. अनु पर् स्थ टिब्बे हवा की दिशा से लं बवत श्रेणीबद्ध होते हैं ।
3. जब रे त की आपूर्ति कम होती है और हवा की दिशा स्थिर होती है तो अनु दैर्ध्य टिब्बे (Longitudinal
dunes) बनते हैं ।

उपर्युक्त कथनों में से कौन सा/से सही है /हैं ?

A  1 and 2 only
केवल 1 और 2

B  3 only
केवल 3

C  2 and 3 only
केवल 2 और 3

D  1, 2 and 3
1, 2 और 3
Solution : C

Explanation:
 Figure :Types of
Sand dunes
Statement 1 is incorrect: Barchans are the crescent-shaped dunes with their points
or wings directed away from the direction of the wind. 
Statement 2 is correct: Transverse dunes are aligned perpendicular to the wind
direction. These dunes form when the wind direction is constant and the source of
sand is an elongated feature at right angles to the wind direction. They may be very
long and low in height. 
Statement 3 is correct: Longitudinal dunes form when the supply of sand is poor
and wind direction is constant. They appear as long ridges of considerable length but
low in height.
व्याख्या :
Figure :Types of
Sand dunes

कथन 1 गलत है : शाखाओं वाले   टिब्बे (Barchans) अर्धचं दर् ाकार टिब्बे होते   हैं जिनके बिन्दु या
भु जा हवा की दिशा से दरू निर्देशित होते हैं ।
कथन 2 सही है : अनु पर् स्थ टिब्बे (Transverse dunes) हवा की दिशा के लं बवत सं रेखित होते हैं ।
जब हवा की दिशा स्थिर होती है और रे त का स्रोत हवा के दिशा में समकोण पर एक दिर्घीभूत
विशे षता होती है , तो ये टीले बन जाते हैं । वे ऊंचाई में बहुत लं बे और कम हो सकते हैं ।
कथन 3 सही है : जब बालू की आपूर्ति कम होती है और हवा की दिशा स्थिर होती है तो अनु दैर्ध्य
टिब्बे (Longitudinal dunes) बनते हैं । ये   काफी लं बाई की लं बी लकीर के रूप में दिखाई दे ते हैं
ले किन ऊंचाई  कम होती है ।
 
 21 s

7
Q. With reference to ocean currents, which of the following statements is/are
correct?

1. Warm currents usually occur on the east coast of the continents in low and middle
latitudes. 
2. Cold currents occur on the west coast of the continents in higher latitudes in the
Northern Hemisphere. 
3. Mixing of warm and cold currents replenishes the oxygen and favours the growth of
planktons. 

Select the correct answer using the codes given below: 


Q. महासागरीय धाराओं के सं दर्भ में , निम्नलिखित कथनों में से कौन सा/से सही है /हैं ?

1. गर्म धाराएं आमतौर पर महाद्वीपों के पूर्वी तट पर निम्न और मध्य अक्षां शों में होती हैं ।
2. उत्तरी गोलार्ध में उच्च अक्षां शों में महाद्वीपों के पश्चिमी तट पर ठं डी धाराएँ होती हैं ।
3. गर्म और ठं डी धाराओं का मिश्रण ऑक्सीजन की पु नःपूर्ति करता है और प्लवक की वृ दधि ् का कारण
बनता है ।

निम्नलिखित कू टों का उपयोग करके सही उत्तर का चयन कीजिए:

A  1 and 2 only
केवल 1 और 2

B  3 only
केवल 3

C  1 and 3 only
केवल 1 और 3

D  1, 2 and 3
1, 2 और 3
Solution : C

Explanation: 
Statement 1 is correct: Warm currents are usually observed on the east coast of
continents in the low and middle latitudes in both hemispheres. 
Statement 2 is incorrect: Cold currents are usually found on the west coast of the
continents in the low and middle latitudes in both the hemispheres. In the higher
latitudes, they occur on the east coast in the Northern Hemisphere. 
Statement 3 is correct: The mixing of warm and cold currents replenishes the
oxygen and favours the growth of planktons. Due to this, the best fishing grounds of
the world exist mainly in these zones where warm and cold currents meet as
planktons are the primary food for the fishes. 
व्याख्या : 
कथन 1 सही है : दोनों गोलार्धों में आमतौर पर निम्न और मध्य अक्षां शों में महाद्वीपों के पूर्वी तट पर
गर्म धाराएँ दे खी जाती हैं ।
कथन 2 गलत है : ठं डी धाराएँ आमतौर पर महाद्वीपों के पश्चिमी तट पर दोनों गोलार्धों में निम्न और
मध्य अक्षां शों में पाई जाती हैं । उच्च अक्षां शों में , ये उत्तरी गोलार्ध में पूर्वी तट पर पायी जाती हैं ।
कथन 3 सही है : गर्म और ठं डी धाराओं का मिश्रण ऑक्सीजन की पु नःपूर्ति करता है और प्लवक
् में योगदान दे ता है । इसके कारण, दुनिया के सबसे अच्छे मछली पकड़ने के
(planktons) की वृ दधि
जगह मु ख्य रूप से इन क्षे तर् ों में मौजूद हैं , जहां गर्म और ठं डी धाराएँ मिलती हैं और प्लैं कटन
मछलियों का प्राथमिक भोजन भी है ।
 58 s

-0.67/2.0 

8
Q. With reference to African Swine Fever (ASF), consider the following statements: 

1. It was reported in India for the first time only in 2020. 


2. It only spreads from animals to other animals. 
3. This disease has no treatment. 

Which of the above statements is/are correct?


Q. अफ् रीकी स्वाइन बु खार (ASF) के सं दर्भ में , निम्नलिखित कथनों पर विचार कीजिए : 

1. 2020 में पहली बार भारत में यह बीमारी सामने आया था।
2. यह केवल जानवरों से दसू रे जानवरों में फैलता है ।
3. इस बीमारी का कोई इलाज नहीं है ।

उपर्युक्त कथनों में से कौन सा/से सही है /हैं ?

A  1 only
केवल 1

B  2 and 3 only
केवल 2 और 3

C  1 and 2 only
केवल 1 और 2

1, 2 and 3
1, 2 और 3
Solution : D

Explanation: 
Context: In July 2020, the pig farms in Assam and Arunachal Pradesh suffered
major losses followed by an outbreak of African Swine Fever (ASF). 
Statement 1 is correct: The July 2020 outbreak of ASF in India is the first time that
the disease has been reported in the country. It was first detected in Africa in the
1920s. 
Statement 2 is correct: It is not a threat to human health and cannot be transmitted
from pigs to humans. It can be spread by ticks that feed on infected animals. Hence,
it only spreads from animals to other animals. 
Statement 3 is correct: There is no treatment available for this disease. The only
way to stop this deadly disease is to cull all affected animals. 
व्याख्या : 
संदर्भ : जु लाई 2020 में , असम और अरुणाचल प्रदे श में सु अर फार्मों को अफ् रीकी स्वाइन बु खार
(ASF) के प्रकोप से बड़ा नु कसान हुआ।
कथन 1 सही है : भारत में अफ् रीकी स्वाइन बु खार (ASF) का प्रकोप  जु लाई 2020 में पहली बार
दे खने को मिला। यह पहली बार 1920 के दशक में अफ् रीका में पाया गया था।
कथन 2 सही है : इससे मानव स्वास्थ्य को कोई खतरा नहीं है और यह सूअरों से मनु ष्यों में नहीं फ़ैल
सकता है । यह सं क्रमित जानवरों पर निर्भर परजीवी (ticks) से फैल सकता है । इसलिए, यह केवल
ू रे जानवरों में फैलता है ।
जानवरों से दस
कथन 3 सही है : इस बीमारी का कोई उपचार उपलब्ध नहीं है । इस घातक बीमारी को रोकने का
एकमात्र तरीका सभी प्रभावित जानवरों को मारना है ।
 16 s

-0.67/2.0 

9
Q. With reference to the Igneous rocks, consider the following statements: 

1. It is formed as a result of solidification and crystallization of magma. 


2. The texture of these rocks depends on the rate and depth of formation. 
3. Pegmatite and Diorite are examples of Igneous rocks. 

Which of the statements given above is/are correct?


Q. आग्ने य चट् टानों के सं दर्भ में , निम्नलिखित कथनों पर विचार कीजिए :

1. यह मै ग्मा के जमने और क्रिस्टलीकरण के परिणामस्वरूप बनता है ।


2. इन चट् टानों की बनावट, निर्माण की दर और गहराई पर निर्भर करती है ।
3. पै गमाटाइट (Pegmatite) और डायोराइट (Diorite), आग्ने य चट् टानों के उदाहरण हैं ।

उपर्युक्त कथनों में से कौन सा/से सही है /हैं ?

A  1 only
केवल 1

B  2 and 3 only
केवल 2 और 3

C  1 and 2 only
केवल 1 और 2

D  1, 2 and 3
1, 2 और 3
Solution : D

Explanation: 
Statement 1 is correct: Igneous rocks are formed out of solidification of magma
(molten rock below the surface) and lava (molten rock above the surface). As
magma cools below 1300°C, minerals start to crystallize within it. They are also
known as primary rocks. 
Statement 2 is correct: Igneous rocks are classified based on texture. The texture
depends upon the size and arrangement of grains or other physical conditions of the
materials. If molten material is cooled slowly at great depths, mineral grains may be
very large. Sudden cooling(at the surface) results in small and smooth grains. 
Statement 3 is correct: Granite, Gabbro, Basalt, Pegmatite, Diorite are some of the
examples of igneous rocks. 
व्याख्या : 
कथन 1 सही है : मै ग्मा (सतह के नीचे पिघली हुई चट् टान) और लावा (सतह के ऊपर पिघली हुई
चट् टान) के जमने से आग्ने य चट् टानें बनती हैं । जै से ही मै ग्मा 1300 डिग्री से ल्सियस से नीचे ठं डा
होता है , इसके भीतर स्थित खनिजों का क्रिस्टलीकरण होता है । इन्हें प्राथमिक चट् टानों के रूप में
भी जाना जाता है ।
कथन 2 सही है : बनावट के आधार पर आग्ने य चट् टानों को वर्गीकृत किया गया है । यह बनावट दाने
(grains) या पदार्थकी अन्य भौतिक स्थितियों के आकार और क् रम पर निर्भर करती है । अगर अगर
पिघला हुआ पदार्थ अधिक गहराई में धीरे -धीरे ठं डा होता है , तो खनिज के दाने (mineral grains)
बहुत बड़े हो सकते हैं । अचानक ठं डा होने (सतह पर) के परिणामस्वरूप छोटे और चिकने दाने
(grains) निकलते हैं ।
कथन 3 सही है : ग्रेनाइट, गै बर् ो (Gabbro), बे साल्ट, पै गमाटाइट (Pegmatite), डायोराइट
(Diorite) आग्ने य चट् टानों के कुछ उदाहरण हैं ।
 49 s

2.0/2.0 

10
Q. Consider the following statements regarding the Karst topography: 

1. These are produced by the action of groundwater on limestone. 


2. Stalagmites are hanging deposits as icicles. 
3. Stalactites rise up from the floor of the caves. 

Which of the statements given above is/are correct?


Q. कार्स्ट स्थलाकृति (Karst topography) के बारे में निम्नलिखित कथनों पर विचार कीजिए :

1. ये चूना पत्थर पर भूजल की क्रिया से निर्मित होते हैं ।


2. निलं बी निक्षे प या स्टै ले ग्माइट् स (Stalagmites),  हिमवर्तिका (icicles) के रूप में लटकते निक्षे प हैं ।
3. आरोही निक्षे प या स्टै ले क्टाइट् स (Stalactites) गु फाओं के तल से ऊपर उठते हैं ।

उपर्युक्त कथनों में से कौन सा/से सही है /हैं ?

A  1 only
केवल 1

B  2 and 3 only
केवल 2 और 3

C  1 and 3 only
केवल 1 और 3

D  1, 2 and 3
1, 2 और 3
Solution : A

Explanation: 
Statement 1 is correct: Karst topography is formed by the action of
groundwater on limestone or dolomitic region through the processes of solution and
deposition. 
Statement 2 is incorrect: Stalactites hang as icicles of different diameters. Normally
they are broad at their bases and taper towards the free ends. 
Statement 3 is incorrect: Stalagmites rise up from the floor of the caves and are
formed due to dripping water from the surface or through the thin pipe of the
stalactite, immediately below it.  

Figure: Karst Topography

व्याख्या : 
कथन 1 सही है : कार्स्ट स्थलाकृति का निर्माण चूना पत्थर या डोलोमिटिक क्षे तर् पर भूजल की क्रिया
द्वारा विलयन और निक्षे पण की प्रक्रियाओं से होता है ।
कथन 2 गलत है : स्टै ले क्टाइट् स अलग-अलग व्यास के हिमवर्तिका (icicles) के रूप में लटकते होते
हैं । आम तौर पर ये अपने आधार पर चौड़े होते हैं और मु क्त छोर की ओर शं कुनु मा होते हैं ।
कथन 3 गलत है : स्टै ले ग्माइट् स गु फाओं के तल  से ऊपर उठते हैं तथा सतह से पानी टपकने के
कारण या इसके ठीक नीचे स्टै ले क्टाइट के पतले पाइप के माध्यम से   बनते हैं ।
Figure: Karst Topography
 1 m 15 s

-0.67/2.0 

11
Q. With reference to deep oceanic trenches, which of the following statements is/are
correct?

1. They are faults located in divergent plate boundaries. 


2. They are associated with active volcanoes and earthquakes. 
3. They are associated with the formation of island archipelagos. 

Select the correct answer using the codes given below: 


Q. गहरी समु दर् ी खाइयों (deep oceanic trenches) के सं दर्भ में , निम्नलिखित कथनों में से कौन
सा/से सही है /हैं ?

1. ये अपसारी प्ले ट सीमाओं में स्थित भ्रंश (Fault)  हैं ।


2. ये सक्रिय ज्वालामु खियों और भूकंपों से सं बंधित हैं ।
3. ये द्वीपसमूहों (island archipelagos) के निर्माण  से सं बद्ध हैं । 

निम्नलिखित कू टों का उपयोग करके सही उत्तर का चयन कीजिए:

A  1 only
केवल 1
B  2 only
केवल 2

C  2 and 3 only
केवल 2 और 3

D  1, 2 and 3
1, 2 और 3
Solution : C

Explanation: 
Oceanic trenches are very long and narrow depressions on the seafloor. These are
the deepest parts of the oceans and some of the deepest natural spots on Earth. 
Statement 1 is incorrect: Oceanic trenches are located in convergent plate
boundaries, where two or more tectonic plates meet. At many convergent plate
boundaries, dense lithosphere melts or slides beneath the less dense lithosphere in
a process called subduction, thus creating a trench. 
Statement 2 is correct: Deep trenches are generally associated with active
volcanoes and earthquakes since there is a tectonic activity that occurs in these
plate boundaries. Hence the world's largest trenches are located in Pacific ‘Ring of
Fire’. 
Statement 3 is correct: Subduction of convergent boundary plate generates an
upwelling of molten crust that results in mountain ridges and volcanic islands
parallel to the deep oceanic trench. For example, the Japanese Archipelago, the
Aleutian Islands, and other locations around the Pacific ‘Ring of Fire’. 
व्याख्या : 
महासागरीय खाइयाँ समु दर् ी तट पर बहुत लं बे और सं कीर्ण गड्ढे (depressions) हैं । ये महासागरों
के सबसे गहरे भाग हैं और पृ थ्वी पर स्थित सबसे गहरे प्राकृतिक धब्बे ( natural spots) भी हैं ।
कथन 1 गलत है : महासागरीय खाइयां अभिसारी प्ले ट सीमाओं में स्थित होती हैं , जहाँ दो या अधिक
टे क्टॉनिक प्ले ट्स मिलती हैं । कई अभिसारी प्ले ट के किनारों पर, पृ थ्वी का सघन ठोस भाग पिघलता
है या कम सघन ठोस भाग के नीचे खिसकता है । इस प्रक्रिया को निम्नस्खलन या सबडक्शन
(subduction) कहा जाता है । इस प्रकार एक खाई का निर्माण होता है ।
कथन 2 सही है : गहरी खाइयाँ आमतौर पर सक्रिय ज्वालामु खियों और भूकंपों से सं बंधित होती हैं ,
क्योंकि इन प्ले ट सीमाओं में एक विवर्तनिक गतिविधि होती है । इसलिए दुनिया की सबसे बड़ी
खाइयां पै सिफिक 'रिं ग ऑफ फायर' में स्थित हैं ।
कथन 3 सही है : अभिसारी सीमा प्ले ट के निम्नस्खलन या सबडक्शन (subduction) से पिघला हुआ
पपड़ी ऊपर आ जाता है जिसके परिणामस्वरूप पर्वतीय टीले और ज्वालामु खी द्वीप गहरी समु दर् ी
खाई के समानांतर बन जाते हैं । उदाहरण के लिए, जापानी द्वीपसमूह, अल्यूशियन द्वीप समूह
(Aleutian Islands) और पे सिफ़िक रिं ग ऑफ फायर ’के आसपास के अन्य स्थान।
 
 1 m 37 s

-0.67/2.0 

12
Q. With reference to “National Education Policy (NEP), 2020”, consider the following
statements: 

1. It will bring the uncovered age group of 3-6 years under the school curriculum. 
2. It promotes teaching up to at least Grade 5 in the mother tongue. 
3. Higher Education Commission of India (HECI) will be set up as an umbrella body for
the entire higher education. 
4. It aims to increase public investment in the Education sector to reach 10% of GDP
by 2030. 

Which of the statements given above are correct?


Q. "राष्ट् रीय शिक्षा नीति (NEP), 2020" के सं दर्भ में , निम्नलिखित कथनों पर विचार कीजिए:

1. इसमें शिक्षा से वं चित 3-6 आयु -वर्ग (uncovered age group) के बच्चों को स्कू ली कार्यक् रम के
तहत लाया जाएगा।
2. यह   कम से कम ग्रेड 5 तक के विद्यार्थियों को मातृ भाषा में शिक्षण को बढ़ावा दे ता है ।
3. भारतीय उच्च शिक्षा आयोग (HECI) की स्थापना पूरी उच्च शिक्षा के लिए एक छतरी निकाय
(umbrella body) के रूप में की जाएगी।
4. इसका उद्दे श्य  शिक्षा क्षे तर् में सार्वजनिक निवे श को 2030 तक सकल घरे लू उत्पाद के 10% तक
पहुंचाना  है ।

उपर्युक्त कथनों में से कौन से सही हैं ?

A  1 and 2 only
केवल 1 और 2

B  3 and 4 only
केवल 3 और 4

C  1, 2 and 3 only


केवल 1, 2 और 3

1, 2, 3 and 4
1, 2, 3 और 4
Solution : A

Explanation: Recently, the Union Cabinet has approved the new National Education
Policy (NEP), 2020. 
Statement 1 is correct: As per NEP, 2020 the current 10+2 system to be replaced
by a new 5+3+3+4 curricular structure corresponding to ages 3-8, 8-11, 11-14, and
14-18 years respectively, thus it brings the hitherto uncovered age group of 3-6
years under school curriculum. 
Statement 2 is correct: The New Education Policy emphasis on promoting
multilingualism and Indian languages; the medium of instruction until at least Grade
5, but preferably till Grade 8 and beyond, will be the mother tongue. 
Statement 3 is incorrect: The New Education Policy 2020 aims to establish the
Higher Education Commission of India (HECI) as a single umbrella body for the
entire higher education, excluding medical and legal education. 
Statement 4 is incorrect: The New Education Policy has not set the time limit to
achieve the target of increasing the spending for the educational sector from the
current 4.6% to 6% of GDP and aims to reach it at the earliest.
व्याख्या : हाल ही में , केंद्रीय मं त्रिमं डल ने नई राष्ट् रीय शिक्षा नीति (NEP), 2020 को मं जरू ी दी
है ।
कथन 1 सही है : राष्ट् रीय शिक्षा नीति, 2020 के अनु सार वर्तमान 10 + 2 प्रणाली को एक नई 5 +
3 + 3 + 4 + 4 पाठयक् रम सं रचना द्वारा प्रतिस्थापित किया जाएगा जो क् रमशः 3-8, 8-11, 11-14
और 14-18 वर्ष की आयु  के अनु रूप होगी।  इस प्रकार वर्त्तमान शिक्षा नीति में   3-6 वर्ष की आयु
वर्ग के बच्चे  को स्कू ली कार्यक् रम के तहत शामिल किया गया है ।
कथन 2 सही है : नई शिक्षा नीति में बहुभाषावाद और भारतीय भाषाओं को बढ़ावा दे ने पर जोर दिया
गया है ; नई शिक्षा नीति में पाँचवी क्लास तक मातृ भाषा, स्थानीय या क्षे तर् ीय भाषा में पढ़ाई का
माध्यम रखने की बात कही गई है . इसे क्लास आठ या उससे आगे भी बढ़ाया जा सकता है ।
कथन 3 गलत है : नई शिक्षा नीति 2020 का उद्दे श्य चिकित्सा और कानूनी शिक्षा के अलावा सं पर्ण

उच्च शिक्षा के लिए एक ही छत्र निकाय (umbrella body) के रूप में   भारतीय उच्च शिक्षा आयोग 
(HECI) की स्थापना करना है ।
कथन 4 गलत है : नई शिक्षा नीति में शै क्षिक क्षे तर् के लिए खर्च को वर्तमान जीडीपी के 4.6% से
बढ़ाकर 6% करने के लक्ष्य को प्राप्त करने के लिए समय सीमा निर्धारित नहीं की गई है और इसका
उद्दे श्य जल्द से जल्द इसे पूरा करना है ।
 1 m 33 s


-0.67/2.0 

13
Q. Consider the following statements: 

1. If the earth stops rotating, then the pressure increases in the subtropical high-
pressure belt. 
2. If the earth stops rotating, then the subpolar low-pressure belt will cease to exist. 

Which of the statements given above is/are correct?


Q. निम्नलिखित कथनों पर विचार कीजिए :

1. यदि पृ थ्वी का घूर्णन बं द हो जाता है , तो उपोष्णकटिबं धीय उच्च दबाव पे टी में दबाव बढ़ जाएगा।
2. यदि पृ थ्वी का घूर्णन बं द हो जाता है , तो उपध्रुवी कम दबाव वाली पे टी  (subpolar low-
pressure belt) मौजूद नहीं रहे गी।

उपर्युक्त कथनों में से कौन सा/से सही है /हैं ?

A  1 only
केवल 1

B  2 only
केवल 2

C  Both 1 and 2
1 और 2 दोनों

D  Neither 1 nor 2
न तो 1, न ही 2
Solution : B

Explanation: 
Both subtropical high pressure and subpolar low-pressure belts are dynamically
induced pressure systems. The dynamic factor producing high and low
atmospheric pressures operate through centrifugal force. 
Statement 1 is incorrect and Statement 2 is correct: If the earth stops rotating,
due to absence of the centrifugal force of the earth, both the subtropical high-
pressure belt and the subpolar low-pressure belt will cease to exist. 
व्याख्या : 
दोनों उपोष्णकटिबं धीय उच्च दबाव और उपध्रुवी कम-दबाव पे टी  गतिशील रूप से प्रेरित दबाव
प्रणाली हैं । उच्च और निम्न वायु मंडलीय दबाव पै दा करने वाले गतिशील कारक अपकेंद्रीय बल
(centrifugal force) के माध्यम से सं चालित होते हैं ।
कथन 1 गलत है और कथन 2 सही है : यदि पृ थ्वी का घूर्णन बं द हो जाता है , तो पृ थ्वी के अपकेंद्रीय
बल (centrifugal force) की अनु पस्थिति के कारण, उपोष्णकटिबं धीय उच्च दबाव पे टी  और
उपध्रुवी कम दबाव पे टी  दोनों मौजूद नहीं रहें गे ।
 1 m 21 s

2.0/2.0 

14
Q. Which of the following forces are responsible for the occurrence of tides?

1. Sun and moon's gravitational pull. 


2. The centrifugal force of earth's rotation. 
3. Atmospheric pressure and wind changes. 

Select the correct answer using the codes given below:  


Q. निम्नलिखित में से कौन सा/से कारक ज्वार की घटना के लिए उत्तरदायी है /हैं ?

1. सूर्य और चं दर् मा का गु रुत्वाकर्षण बल।


2. पृ थ्वी के घूर्णन का अपकेंद्रीय बल।
3. वायु मंडलीय दाब और हवा में परिवर्तन।

निम्नलिखित कू टों का उपयोग करके सही उत्तर चु निए:

A  1 and 2 only
केवल 1 और 2

B  1 only
केवल 1

C  2 and 3 only
केवल 2 और 3

D  1, 2 and 3
1, 2 और 3
Solution : A

Explanation: 
Statement 1 is correct: The moon’s gravitational pull to a great extent and to a
lesser extent the sun’s gravitational pull, are the major causes for the occurrence of
tides. 
Statement 2 is correct: Centrifugal force of the earth is the force that acts to
counterbalance the gravity. Together, the gravitational pull and the centrifugal force
are responsible for creating the two major tidal bulges on the earth. 
Statement 3 is incorrect: Movement of water caused by meteorological effects
such as winds and atmospheric pressure changes are called surges. Surges are
not regular like tides. 
व्याख्या:
कथन 1 सही है : काफी हद तक चं दर् मा का गु रुत्वाकर्षण बल और कुछ हद तक सूर्य का गु रुत्वाकर्षण
बल, ज्वार की घटना के प्रमु ख कारण हैं ।
कथन 2 सही है : पृ थ्वी का अपकेंद्रीय बल वह बल है जो गु रुत्व बल के प्रतिभार के रूप में कार्य
करता है । गु रुत्वाकर्षण बल तथा अपकेंद्रीय बल दोनों मिलकर पृ थ्वी पर दो महत्वपूर्ण ज्वारभाटााओं
को उत्पन्न करने के लिए उत्तरदायी हैं ।
कथन 3 गलत है : मौसम सं बंधी प्रभावों जै से हवा और वायु मंडलीय दाब में परिवर्तन के कारण पानी
की गति को महोर्मि (surge) कहा जाता है । महोर्मि ज्वारभाटाओं की तरह नियमित नहीं होते हैं ।
 51 s

-0.67/2.0 

15
Q. These clouds are black or dark grey in colour. They form at the mid-level
atmosphere or very near to the surface of the earth. They are extremely dense and
opaque to the rays of the sun. 
In the context of types of clouds, which of the following best applies to the above
passage?
Q. ये बादल काले या गहरे भूरे रं ग के होते हैं । ये मध्य-स्तर के वायु मंडल में बनते हैं या पृ थ्वी की
सतह के बहुत निकट होते हैं । ये   बे हद घने और सूर्य की किरणों के लिए अपारदर्शी होते हैं ।
बादलों के प्रकारों के सं दर्भ में , निम्नलिखित में से कौन सा उपर्युक्त विवरणों पर लागू होता है ?

A  Cirrus
पक्षाभ (Cirrus) मे घ

B  Cumulus
कपासी (Cumulus) मे घ

Nimbus
वर्षा (Nimbus) मे घ

D  Stratus
स्तरी (Stratus) मे घ
Solution : C

Explanation: 
Option (a) is incorrect: Cirrus clouds are formed at high altitudes, and they are
generally thin, having a feathery appearance. They are always white in colour. 
Option (b) is incorrect: Cumulus clouds look like cotton wool with a flat base.
They exist in patches. 
Option (c) is correct: Nimbus clouds are black or dark grey in colour and
are extremely dense. They are a shapeless mass of thick vapour. They form at the
middle or very near to the surface of the earth. 
Option (d) is incorrect: Stratus clouds are low-level clouds which are grey or
white in colour, characterized by horizontal layering with a uniform base. 
व्याख्या : 
विकल्प (a) गलत है : पक्षाभ (Cirrus) मे घ अधिक ऊंचाई पर बनते हैं , और आम तौर पर विरल होते
हैं , जिनकी बहुत हल्की उपस्थिति होती है । ये हमे शा सफेद रं ग के होते हैं ।
विकल्प (b) गलत है : कपासी मे घ रूई के समान दिखते हैं और चपटे आधार वाले होते हैं । ये छितरे
एवं इधर - उधर बिखरे होते   हैं ।
विकल्प (c) सही है : वर्षा (Nimbus) मेघ काले या गहरे भूरे रं ग के होते हैं और बे हद घने होते हैं । ये
घने वाष्प के एक आकारहीन द्रव्यमान होते हैं । ये पृ थ्वी की सतह के मध्य या बहुत निकट स्थित होते
हैं ।
विकल्प (d) गलत है : स्तरी (Stratus) मे घ निम्न स्तर के बादल हैं जो भूरे या सफेद रं ग के होते
हैं । एक समान आधार के साथ क्षै तिज परत इसकी विशे षता है ।
 1 m 1 s

2.0/2.0 

16
Q. The ‘Hotspot Volcanism’ is unique because of:
Q. निम्नलिखित में से किस कारण से 'हॉटस्पॉट ज्वालामु खी' (Hotspot Volcanism) विशे ष होता है
?

It allows super-heated molten rock, along with ash and gases to rise through the earth
crust.
यह पृ थ्वी की पपड़ी के माध्यम से राख और गै सों के साथ-साथ अत्यधिक गर्म पिघले हुए चट् टान के
उदय का मार्ग प्रशस्त करता है ।

While tectonic plates are stationary, the mantle plumes that form hotspots are not
B   relatively stationary.
हालां कि टे क्टोनिक प्ले ट स्थिर होती हैं , में टल प्लम (mantle plumes) जो हॉटस्पॉट बनाते हैं ,
अपे क्षाकृत स्थिर नहीं होते हैं ।

It can produce lines of islands, atolls, and seamounts known as hotspot tracks.

यह हॉटस्पॉट ट् रैक (hotspot tracks) के रूप में ज्ञात द्वीपों, प्रवालद्वीपों और अं तःसमु दर् ी पर्वत
(seamounts) की उत्पत्ति का कारण बन सकता है ।

D  It does not occur at the boundaries of tectonic plates.


यह टे क्टोनिक प्ले टों की सीमाओं पर नहीं होता है ।
Solution : D

Explanation: 
All volcanic activities occur along the margin of plate boundaries. Hotspot volcanism
is unique because it does not occur at the boundaries of Earth’s tectonic plates.
Instead, it occurs at abnormally hot centres known as mantle plumes. 
व्याख्या : 
सभी ज्वालामु खीय गतिविधियाँ प्ले ट सीमाओं के किनारों पर होती हैं । हॉटस्पॉट ज्वालामु खी अपने
आप में विशे ष है , क्योंकि यह पृ थ्वी की टे क्टोनिक प्ले टों की सीमाओं पर नहीं होता है । इसके बजाय,
यह असामान्य रूप से गर्म केंद्रों पर होता है जिसे में टल प्लम्स (mantle plumes) कहा जाता है ।
 1 m 13 s

-0.67/2.0 

17
Q. With reference to ‘Compulsory Licensing’ in India, consider the following
statements: 

1. The Government can authorise third parties to manufacture a patented product only
with the consent of the patent owner. 
2. It is allowed by the Agreement on Trade-Related Aspects of Intellectual Property
Rights of WTO. 
3. Indian patent Act of 1970 empowers the government to issue compulsory licensing. 

Which of the above statements is/are correct?


Q. भारत में 'अनिवार्य लाइसें सिं ग' के सं दर्भ में , निम्नलिखित कथनों पर विचार कीजिए:

1. सरकार केवल पे टेंट मालिक की सहमति से पे टेंट उत्पाद के निर्माण हे तु तीसरे पक्ष को अधिकृत कर
सकती है ।
2. विश्व व्यापार सं गठन के बौद्धिक सं पदा अधिकारों के व्यापार-सं बंधित पहलु ओं पर समझौते के
अं तर्गत अनिवार्य लाइसें सिं ग की अनु मति है ।
3. 1970 का भारतीय पे टेंट अधिनियम सरकार को अनिवार्य लाइसें स जारी करने का अधिकार प्रदान
करता है ।

उपर्युक्त कथनों में से कौन सा/से सही है /हैं

A  1 only
केवल 1

B  3 only
केवल 3

C  2 and 3 only
केवल 2 और 3

D  1, 2 and 3
1, 2 और 3
Solution : C

Explanation:

Recently, there were demands from health professionals to issue compulsory


licences for the manufacturing of a generic version of Remdesivir which is being
used to treat Covid-19 patients. 
Statement 1 is incorrect: Compulsory Licensing (CL) allows the government of
India (along with many other governments throughout the world as per their local
law) to give license to third parties to produce and market a patented product or
process without the consent of patent owners. 
Statement 2 is correct: The Agreement on Trade-Related Aspects of Intellectual
Property Rights of WTO allows compulsory licensing. 
Statement 3 is correct: According to Section 92 of the Indian Patent Act 1970,
compulsory licenses can also be issued suo moto by the Controller of Patents in
pursuant to a notification issued by the Central Government. This is done if there is
either a “national emergency” or “extreme urgency” or in cases of “public non-
commercial use”. 
व्याख्या:
हाल ही में , स्वास्थ्य पे शेवरों द्वारा रे मडे सिवीर के जे नेरिक सं स्करण के निर्माण के लिए अनिवार्य
लाइसें स जारी करने की मां ग की गई थी, जिसका उपयोग कोविड-19 रोगियों के इलाज के लिए किया
जा रहा है ।
कथन 1 गलत है : अनिवार्य लाइसें सिं ग (CL) भारत सरकार (दुनिया भर में कई अन्य सरकारों को भी
उनके स्थानीय कानून के अनु सार) को पे टेंट मालिक की सहमति के बिना एक पे टेंट उत्पाद का उत्पादन
और विपणन या उसकी प्रक्रिया का प्रयोग करने के लिए तीसरे पक्ष को लाइसें स दे ने की अनु मति
दे ता है । 
कथन 2 सही है : विश्व व्यापार सं गठन के बौद्धिक सं पदा अधिकारों के व्यापार-सं बंधित पहलु ओं पर
समझौता अनिवार्य लाइसें सिं ग की अनु मति दे ता है ।
कथन 3 सही है : भारतीय पे टेंट अधिनियम 1970 की धारा 92 के अनु सार, केंद्र सरकार द्वारा जारी
अधिसूचना के अनु सार पे टेंट नियं तर् क द्वारा स्वतः ही अनिवार्य लाइसें स जारी किए जा सकते हैं ।
ऐसा ‘राष्ट् रीय आपातकाल’ या अत्यं त आवश्यकता’ अथवा ‘गै र-व्यावसायिक सार्वजनिक उपयोग’ के
मामलों में किया जा सकता है ।
 2 m 0 s

2.0/2.0 

18
Q. Consider the following statements: 

1. In collective farming, farms are owned by several farmers who pool their resources
voluntarily, whereas in cooperative farming farms are owned by a productive
cooperative union. 
2. The Kibbutz is a traditional farming community from Russia. 
3. The Moshav is a type of cooperative agricultural settlement in Israel. 

Which of the statements given above is/are correct?


Q. निम्नलिखित कथनों पर विचार कीजिए:

1. सामूहिक कृषि में , खे तों का स्वामित्व कई किसानों के पास होता है , जो अपने सं साधनों को स्वे च्छा से
मिलकर उपयोग करते हैं , जबकि सहकारी कृषि में खे तों पर एक उत्पादक सहकारी सं घ का स्वामित्व
होता है ।
2. किबूत रूस का एक पारं परिक कृषक समु दाय है ।
3. मोशाव इजरायल में सहकारी कृषि बं दोबस्त का एक प्रकार है । 

उपर्युक्त कथनों में से कौन सा/से सही है /हैं ?



1 only
केवल 1

B  3 only
केवल 3

C  2 and 3 only
केवल 2 और 3

D  1, 2 and 3
1, 2 और 3
Solution : B

Explanation: 
Statement 1 is incorrect: In cooperative farming, farms are owned by several
farmers who pool their resources voluntarily, work together and share the profits or
produce, whereas, in collective farming, farms are owned by a productive
cooperative union, based on social ownership of the means of production including
land and labour. 
Statement 2 is incorrect: Kibbutz is a traditional farming community from Israel.
Kibbutz farmers and their families pool resources and share in the wealth that is
produced as a result of their work. 
Statement 3 is correct: Moshav is a type of cooperative agricultural settlement in
Israel. It is a collection of individual farms that operate with shared harvesting
equipment, storage facilities, and various other implements. 
व्याख्या:
कथन 1 गलत है : सहकारी कृषि में , खे तों पर कई किसानों का अधिकार होता है जो अपने सं साधनों
को स्वे च्छा से मिलकर एक साथ काम करते हैं और लाभ या उत्पादन को परस्पर साझा करते हैं ,
जबकि सामूहिक कृषि में , खे तों पर एक उत्पादक सहकारी सं घ का स्वामित्व होता है , जो भूमि और
श्रम सहित उत्पादन के साधनों के सामाजिक स्वामित्व पर आधारित है ।
कथन 2 गलत है : किबूत इजरायल का एक पारं परिक कृषक समु दाय है । किबूत किसान और उनके
परिवार एक साथ मिलकर सं साधनों का  उपयोग करते हैं और परिणामस्वरूप उत्पन्न उत्पादन को
साझा करते हैं ।
कथन 3 सही है : मोशाव इजरायल में सहकारी कृषि बं दोबस्त का एक प्रकार है । यह व्यक्तिगत खे तों
का एक समूह है जो साझा कृषि उपकरणों, भं डारण सु विधाओं और विभिन्न अन्य उपकरणों के साथ
सं चालित होता है ।
 1 m 30 s


2.0/2.0 

19
Q. With reference to Eskers, which of the following statements is/are correct?

1. They are erosional landforms formed by the action of glaciers. 


2. They are valuable tools for understanding the nature and evolution of glaciers and
ice sheets. 

Select the correct answer using the codes given below: 


Q. एस्कर के सं दर्भ में , निम्नलिखित कथनों में से कौन सा/से कथन सही है /हैं ?

1. ये हिमनदों की क्रिया द्वारा निर्मित अपरदित भू-आकृतियाँ हैं ।


2. ये हिमनदों और बर्फ की चादरों की प्रकृति तथा विकास को समझने के लिए मूल्यवान साधन हैं ।

निम्नलिखित कू टों का उपयोग करके सही उत्तर चु निए:

A  1 only
केवल 1

B  2 only
केवल 2

C  Both 1 and 2
1 और 2 दोनों

D  Neither 1 nor 2
न तो 1, न ही 2
Solution : B

Explanation: 
Statement 1 is incorrect: Eskers are the depositional landforms formed by glaciers,
which are a long, winding ridge of stratified sand, gravel and other coarse materials
like boulders and blocks deposited by glacial meltwater flowing through tunnels
within and underneath glaciers, or through meltwater channels on top of glaciers.
Some minor fractions of rock debris carried into this stream settle in the valley of ice
beneath the glacier. Once the ice melts sinuous ridges become visible which are
then called Eskers. 
Statement 2 is correct: Eskers are valuable tools for understanding the nature and
evolution of glaciers and ice sheets. They record the paths of basal meltwater
drainage near to the ice margin.
व्याख्या:
कथन 1 गलत है : एस्कर हिमनदों द्वारा निर्मित निक्षे पित भू-आकृतियाँ हैं , जो हिमनद के पिघलने से
जल के हिमतल के ऊपर अथवा इसके किनारों या बर्फ के छिद्रों से नीचे प्रवाहित होने के दौरान
बजरी, रे त, कंकड-पत्थर, बड़े गोलाश्म, चट् टानी टु कड़े और छोटा चट् टानी मलबा आदि के निक्षे पण
से लम्बे , सं करे , लहरदार टीले होते हैं । जलधारा अपने साथ बड़े गोलाश्म, चट् टानी टु कड़े और छोटा
चट् टानी मलबा बहाकर लाती है जो हिमनद के नीचे इस बर्फ की घाटी में जमा हो जाते हैं । ये बर्फ
पिघलने के बाद एक वक् राकार कटक के रूप में मिलते हैं , जिन्हें एस्कर कहते हैं ।
कथन 2 सही है : हिमनद और बर्फ की चादरों की प्रकृति तथा विकास को समझने के लिए एस्कर
मूल्यवान साधन हैं । ये  बर्फ के किनारे आधारीय पिघले हुए जल के अपवाह मार्ग की रूपरे खा प्रस्तु त
करते हैं ।
 58 s

-0.67/2.0 

20
Q. It is an area with temperatures of 20-25° celsius with a small diurnal and annual
range. It is composed of acidic soil which is poor in nutrients. Which of the following
forest types is being discussed in the above statements?

Q. यह 20-25 डिग्री से ल्सियस दै निक और वार्षिक तापमान परास वाला एक क्षे तर् है । यह अम्लीय
मिट् टी से निर्मित है जिसमें पोषक तत्वों की कमी है । उपर्युक्त कथन में निम्नलिखित में से किस वन
प्रकार की चर्चा की जा रही है ?

A  Tropical Evergreen forests


उष्णकटिबं धीय सदाबहार वन

B  Tropical Deciduous forest


उष्णकटिबं धीय पर्णपाती वन

C  Tropical Savannah
उष्णकटिबं धीय सवाना

D  Temperate forest
शीतोष्ण वन
Solution : A

Explanation: 
Option (a) is correct: Tropical evergreen forests are characterised by temperatures
of 20-25° C with a small diurnal and annual range. It is composed of acidic soil which
is poor in nutrients. It has a higher canopy and stratification with multi-layered tall
and large trees. They are found in equatorial regions. 
Option (b) is incorrect: Tropical deciduous forests are characterised by a
temperature range of 25-30° C and a seasonal annual rainfall of 1000 mm. The soil
in deciduous forests is rich in nutrients. In these forests, trees are less dense
and of medium height. Many varieties of trees coexist. 
Option (c) is incorrect: Tropical savanna forests are characterised by warm-hot
climates with temperatures ranging from 20-30°C and rainfall in the range of 500-
1,250 mm. It has a soil which is porous with a thin layer of humus. Trees and
large shrubs are absent in this. 
Option (d) is incorrect: Temperate forests are characterised by temperatures
ranging from 20-30°C with evenly distributed rainfall of 750-1,500mm. Well defined
seasons and distinct winter is the characteristic of these forests. It has soil which is
fertile, enriched with decaying litter. They consist of moderately dense broad-
leafed trees with less diversity of plant species. 
व्याख्या: 
विकल्प (a) सही है : उष्णकटिबं धीय सदाबहार जं गलों का तापमान 20-25 डिग्री से ल्सियस होता है
तथा तापमान में दै निक और वार्षिक अं तर  बहुत कम होता है । यह अम्लीय मिट् टी से निर्मित होता है
जिसमें पोषक तत्वों की कमी होती है । ये वन काफी घने और अत्यधिक छायादार होते हैं तथा इनमें
काफी लं बे और बड़े वृ क्ष होते हैं । ये भूमध्यरे खीय क्षे तर् ों में पाए जाते हैं ।  
विकल्प (b) गलत है : उष्णकटिबं धीय पर्णपाती वनों का तापमान परास 25-30 डिग्री से ल्सियस होता
है और यहाँ 1000 मिमी की मौसमी वार्षिक वर्षा होती है । पर्णपाती वनों में  मिट्टी पोषक तत्वों से
भरपूर होती है । इन वनों में , पे ड़ कम घने और मध्यम ऊंचाई के होते हैं । इन वनों में पे ड़ों की कई
किस्में पायी जाती हैं ।
विकल्प (c) गलत है : उष्णकटिबं धीय सवाना वन का तापमान परास 20-30 डिग्री से ल्सियस होता है
और यहाँ 500-1,250 मिमी वर्षा होती है । गर्म-उष्ण जलवायु इन वनों की विशे षता होती है । यहाँ
की मृ दा हयूमस (खाद-मिट्टी) की एक पतली परत के साथ छिद्रयु क्त होती है । इन वनों में पे ड़ और
बड़ी झाड़ियाँ अनु पस्थित होती हैं ।
विकल्प (d) गलत है : शीतोष्ण वन का तापमान परास 20-30 डिग्री से ल्सियस तक होता है और यहाँ
750-1,500 मिमी वर्षा होती है । अच्छी तरह से परिभाषित मौसम और विशिष्ट सर्दी इन वनों की
विशे षता है । यहाँ की मिट् टी सड़नशील घासफू स से समृ द्ध और अत्यधिक उपजाऊ होती है । इन वनों
में मध्यम से घने चौड़े पत्ते वाले वृ क्ष पाए जाते हैं और यहाँ पौधों की प्रजातियों की कम विविधता
होती है ।
 30 s


-0.67/2.0 

21
Q. With reference to seismic waves, which of the following statements is/are correct?

1. Seismic waves are mechanical waves. 


2. Seismic waves propagate fastest through gaseous medium compared to solid and
liquid mediums. 
3. Surface waves have a higher frequency than the body waves, hence arrive earlier
than body waves when an earthquake strikes. 

Select the correct answer using the codes given below: 


Q. भूकंपीय तरं गों के सं दर्भ में , निम्नलिखित कथनों में से कौन सा/से सही है /हैं ?

1. भूकंपीय तरं गें यां त्रिक तरं गें होती हैं ।


2. भूकंपीय तरं गें ठोस और तरल माध्यमों की तु लना में गै सीय माध्यम में ते जी से सं चरित होती हैं ।
3. धरातलीय तरं गों में भूगर्भिक तरं गों की तु लना में उच्च आवृ त्ति होती है , इसलिए भूकंप आने पर
भूगर्भिक तरं गों की तु लना में धरातलीय तरं गे सतह पर पहले पहुंचती हैं ।

निम्नलिखित कू टों का उपयोग करके सही उत्तर चु निए:

A  1 only
केवल 1

B  1 and 3 only
केवल 1 और 3

C  2 and 3 only
केवल 2 और 3

D  1, 2 and 3
1, 2 और 3
Solution : A

Explanation:

Seismic waves are energy waves that travel through the layers of Earth. They are
the result of earthquakes, volcanic eruptions, magma movement, landslides and
even man-made explosions. 
Statement 1 is correct: Seismic waves are mechanical waves which require a
medium for propagation. 
Statement 2 is incorrect: Seismic waves propagate fastest through the solid
medium followed by the liquid and lastly the gaseous medium. 
Statement 3 is incorrect: Body waves and surface waves are the two main types of
seismic waves. Body waves can travel through the earth's inner layers, but surface
waves can only move along the surface of the planet like ripples on water.
Earthquakes radiate seismic energy as both body and surface waves. Body waves
have a higher frequency than the surface waves, hence arrive earlier than surface
waves when an earthquake strikes.
व्याख्या:

भूकंपीय तरं गें ऊर्जा तरं गें हैं जो पृ थ्वी की परतों के माध्यम से सं चरित होती हैं । वे भूकंप, ज्वालामु खी
विस्फोट, मै ग्मा गति, भूस्खलन और यहाँ तक कि मानव निर्मित विस्फोटों के परिणामस्वरुप उत्पन्न
होती हैं ।
कथन 1 सही है : भूकंपीय तरं गें यां त्रिक तरं गें हैं जिनके सं चरण के लिए एक माध्यम की आवश्यकता
होती है ।
कथन 2 गलत है : भूकंपीय तरं गें ठोस माध्यम में सबसे ते जी से सं चरित होती हैं और उसके बाद द्रव
और गै सीय माध्यम में सं चरित होती हैं ।
कथन 3 गलत है : भूगर्भिक तरं गें और धरातलीय तरं गें भूकंपीय तरं गों के दो मु ख्य प्रकार हैं । भूगर्भिक
तरं गें पृ थ्वी की आं तरिक परतों के माध्यम से सं चरित हो सकती हैं , ले किन धरातलीय तरं गें केवल ग्रह
की सतह पर पानी की लहरों की तरह गति कर सकती हैं । भूकंप भूगर्भिक और धरातलीय तरं गों दोनों
के रूप में भूकंपीय ऊर्जा को विकीर्णित करते हैं । भूगर्भिक तरं गों में धरातलीय तरं गों की तु लना में
उच्च आवृ त्ति होती है , इसलिए भूकंप आने पर धरातलीय तरं गों की तु लना में पहले पहुंचते हैं ।
इसलिए, भूकंप आने पर धरातलीय तरं गों की तु लना में सतह पर पहले पहुंचती हैं ।
 58 s

-0.67/2.0 

22
Q. The ‘Mongolian Kanjur’ which was recently in the news, is the text related to:

Q. हाल ही में चर्चा में रहे 'मं गोलियाई कंजूर’ ग्रंथ किससे सं बंधित है ?

A  Islam
इस्लाम

B  Taoism
ताओवाद

Confucianism
कन्फ्यूशियसवा

D  Buddhism
बौद्ध धर्म
Solution : D

Context: The Ministry of Culture has taken up the project of reprinting 108 volumes
of Mongolian Kanjur by March 2022 under the National Mission for Manuscripts
(NMM). 
Explanation:

Mongolian Kanjur is a Buddhist canonical text in 108 volumes and is considered to


be the most important religious text in Mongolia. In the Mongolian language ‘Kanjur’
means ‘Concise Orders’- the words of Lord Buddha in particular. It has been
translated from Tibetan and is written in classical Mongolian. 
संदर्भ: सं स्कृति मं तर् ालय ने राष्ट् रीय पांडुलिपि मिशन (NMM) के तहत मं गोलियाई कंजूर के 108
सं स्करणों को मार्च 2022 तक पु नर्मुद्रण करने की परियोजना शु रू की है ।
व्याख्या:

मं गोलियाई कंजूर 108 खं डों का एक धर्म वै धानिक ग्रंथ है और इसे मं गोलिया का सबसे महत्वपूर्ण
धार्मिक ग्रंथ माना जाता है । मं गोलियाई भाषा में 'कंजूर’ का अर्थ 'सं क्षिप्त आदे श' जो विशे ष रूप से
भगवान बु द्ध के शब्द हैं । इसे तिब्बती भाषा से अनु वादित किया गया है और पारम्परिक मं गोलियाई
भाषा में लिखा गया है ।
 26 s

2.0/2.0 

23
Q. With reference to cool temperate eastern margin climate, which of the following
statements is/are incorrect?

1. It has features of both continental and maritime climate. 


2. The summers are warm and wet whereas winters are cold and dry. 
3. Though rainfall occurs throughout the year, there is a distinct winter maximum due
to the effect of westerlies.  
Select the correct answer using the codes given below: 
Q. ठं डी शीतोष्ण पूर्वी सीमांत जलवायु के सं दर्भ में , निम्नलिखित कथनों में से कौन सा/से  गलत 
है /हैं ?

1. इसमें महाद्वीपीय जलवायु और महासागरीय जलवायु दोनों की विशे षताएँ होती हैं ।
2. ग्रीष्मकाल गर्म और आर्द्र होता है जबकि शीत ऋतु ठं डी और शु ष्क होती हैं ।
3. यद्यपि वर्ष भर वर्षा होती है , ले किन पश्चिमी पवनों के कारण सर्दी अधिकतम होती है ।

निम्नलिखित कू टों का उपयोग करके सही उत्तर चु निए:

A  2 only
केवल 2

B  2 and 3 only
केवल 2 और 3

C  3 only
केवल 3

D  1 and 2 only
केवल 1 और 2
Solution : C

Explanation: 
Statement 1 is correct: Cool temperate eastern margin climate or Laurentian
climate has features of both continental and maritime climate. It is an intermediate
type of climate between the British and the Siberian type of climate.  
Statement 2 is correct: Winters are dry and cold with temperatures well below the
freezing point. Summers are warm and wet like tropics. 
Statement 3 is incorrect: Though rainfall occurs throughout the year, there is a
distinct summer maximum due to easterly winds from the oceans. Winters are
cold and dry due to the westerlies that blow from the interior of the continent. 
व्याख्या:
कथन 1 सही है : ठं डी शीतोष्ण पूर्वी सीमांत जलवायु या लॉरे न्शियन जलवायु में महाद्वीपीय और
समु दर् ी जलवायु दोनों की विशे षताएँ होती हैं । यह ब्रिटिश और साइबे रियाई प्रकार की जलवायु के
बीच का जलवायु का एक मध्यवर्ती प्रकार है ।
कथन 2 सही है : शीत ऋतु शु ष्क और ठं डी होती हैं और तापमान हिमांक बिं दु से नीचे रहता है ।
ग्रीष्मकाल उष्णकटिबं धीय जलवायु की तरह गर्म और आर्द्र होता है ।
कथन 3 गलत है : यद्यपि वर्ष भर वर्षा होती है , ले किन महासागरों से आने वाली पूर्वी पवनों के कारण
अधिकतम गर्मी होती है । महाद्वीप के आं तरिक भाग से आने वाली पश्चिमी पवनों के कारण शीत ऋतु
ठंडी और शु ष्क होती हैं ।
 7 s

24
Q. With reference to the Special Liquidity Scheme for Non-Banking Financial
Companies (NBFCs) and Housing Finance Companies (HFCs), consider the
following statements: 

1. Funds will be provided through budgetary allocations. 


2. NBFCs registered as core investment companies are ineligible for this scheme. 
3. Eligible NBFC should be rated investment grade. 

Which of the above statements are correct?


Q. गै र-बैं किंग वित्तीय कंपनियों (NBFCs) और आवासीय वित्तीय कंपनियों (HFCs) के लिए विशे ष
तरलता योजना के सं दर्भ में , निम्नलिखित कथनों पर विचार कीजिए:

1. धनराशि बजटीय आवं टन के माध्यम से प्रदान की जाएगी।


2. कोर निवे श कंपनियों के रूप में पं जीकृत NBFCs इस योजना हे तु अपात्र हैं ।
3. पात्र NBFC को निवे श ग्रेड का दर्जा दिया जाएगा।

उपरोक्त कथनों में से कौन से सही हैं ?

A  1 and 2 only
केवल 1 और 2

B  2 and 3 only
केवल 2 और 3

C  1 and 3 only
केवल 1 और 3

D  1, 2 and 3
1, 2 और 3
Solution : B

Explanation:

In July 2020, the Government of India has approved a scheme to improve the
liquidity position of NBFCs/HFCs through a Special Purpose Vehicle (SPV) to avoid
any potential systemic risks to the financial sector. 
Statement 1 is incorrect: The Reserve Bank of India(RBI) will provide funds for the
Scheme by subscribing to government-guaranteed special securities issued by the
Special Purpose Vehicle established by the SBICAP. 
Statement 2 is correct: Only the NBFCs and Microfinance Institutions that are
registered with the RBI under the Reserve Bank of India Act, 1934 are eligible for
this liquidity scheme. This scheme excludes those NBFCs which are registered
as Core Investment Companies. Housing Finance Companies that are registered
under the National Housing Bank Act, 1987 also eligible for this scheme. 
Statement 3 is correct: To be eligible for this scheme, a particular NBFC should be
rated investment grade by a SEBI registered rating agency. 
व्याख्या:
जु लाई 2020 में , भारत सरकार ने वित्तीय क्षे तर् में किसी भी सं भावित प्रणालीगत जोखिम से बचने
के लिए एक विशे ष प्रयोजन साधन (Special Purpose Vehicle) के माध्यम से NBFC/HFC की
तरलता की स्थिति में सु धार हे तु एक योजना को मं जरू ी दी है ।
कथन 1 गलत है : भारतीय रिज़र्व बैं क (RBI) SBICAP (भारतीय स्टे ट बैं क कैपिटल मार्के ट् स
लिमिटे ड) द्वारा स्थापित विशे ष प्रयोजन साधन द्वारा जारी सरकारी-गारं टीकृत विशे ष प्रतिभूतियों
की सहायता से योजना के लिए धनराशि प्रदान करे गा।
कथन 2 सही है : केवल भारतीय रिजर्व बैं क अधिनियम, 1934 के तहत RBI के साथ पं जीकृत NBFC
और सूक्ष्म वित्तीय सं स्थान इस तरलता योजना हे तु पात्र हैं । इस योजना में कोर निवेश कं पनी के रूप
में पंजीकृत NBFCs शामिल नहीं हैं । राष्ट् रीय आवास बैं क अधिनियम, 1987 के तहत पं जीकृत
आवासीय वित्तीय कंपनियां भी इस योजना हे तु पात्र हैं ।
कथन 3 सही है : इस योजना हे तु पात्रता के लिए, एक विशे ष NBFC को से बी द्वारा पं जीकृत रे टिंग
एजें सी द्वारा निवे श ग्रेड दिया जाना चाहिए।
 49 s

25
Q. With reference to dust particles, which of the following statements
is/are incorrect?

1. It consists of a mixture of solid and liquid particles suspended in the atmosphere


which vary in composition, source and size. 
2. Higher concentrations of dust particles are found in subtropical and temperate
regions. 
3. Their concentration increases as we go to higher altitudes in the atmosphere. 
Select the correct answer using the codes given below: 
Q. धूल कणों के सं दर्भ में , निम्नलिखित कथनों में से कौन सा/से  गलत है /हैं ?

1. इसमें वायु मंडल में निलं बित ठोस और द्रव कणों का मिश्रण होता है जो सं रचना, स्रोत और आकार
में भिन्न होते हैं ।
2. धूल कणों की उच्च सघनता उपोष्णकटिबं धीय और समशीतोष्ण क्षे तर् ों में पायी जाती है ।
3. जै से-जै से हम वायु मंडल में ऊंचाई पर जाते हैं , उनकी सांदर् ता बढ़ती जाती है ।

निम्नलिखित कू टों का उपयोग करके सही उत्तर चु निए:

A  1 only
केवल 1

B  3 only
केवल 3

C  2 and 3 only
केवल 2 और 3

D  1, 2 and 3
1, 2 और 3
Solution : B

Explanation: 
Statement 1 is correct: It consists of a mixture of solid and liquid particles
suspended in the atmosphere which vary in composition, source and size. 
Statement 2 is correct: Dust particles are found in subtropical and temperate
regions at a higher concentration than the tropical regions due to dry winds in
comparison to equatorial and polar regions which have humidity in winds. 
Statement 3 is incorrect: They are generally concentrated in the lower layers of the
atmosphere. But at times, convection air currents transport them to higher altitudes. 
व्याख्या:
कथन 1 सही है : इसमें वायु मंडल में निलं बित ठोस और द्रव कणों का मिश्रण होता है जो सं रचना,
स्रोत और आकार में भिन्न होते हैं ।
कथन 2 सही है : नमी यु क्त भूमध्यरे खीय और ध्रुवीय क्षे तर् ों की हवाओं के मु काबले शु ष्क हवाओं के
कारण धूल कणों  की सघनता उष्णकटिबं धीय क्षे तर् ों की तु लना में उपोष्णकटिबं धीय और समशीतोष्ण
क्षे तर् ों में अधिक पायी जाती है ।
कथन 3 गलत है : वे सामान्यतः वायु मंडल की निचली परतों में सं केंद्रित होते हैं । ले किन कई
बार,सं वहन वायु धाराओं द्वारा ये कण उच्च ऊँचाइयों तक पहुँच जाते हैं ।
 1 m 21 s

-0.67/2.0 

26
Q. Which of the following are not direct sources for studying the interior of the Earth?

1. Drilling 
2. Meteorites
3. Volcanic eruption
4. Seismic waves

Select the correct answer using the codes given below:

Q. निम्नलिखित में से कौन पृ थ्वी के आं तरिक भाग के अध्ययन हे तु प्रत्यक्ष स्रोत नहीं हैं ?

1. खनन
2. उल्का पिं ड
3. ज्वालामु खी विस्फोट
4. भूकंपीय तरं गे

निम्नलिखित कू टों का उपयोग करके सही उत्तर चु निए:

A  1 and 3 only 
केवल 1 और 3

B  2 and 4 only
केवल 2 और 4

C  1 and 4 only
केवल 1 और 4

D  2 and 3 only
केवल 2 और 3
Solution : B

Explanation:

Most of our knowledge about the interior of the earth is largely based on estimates and
inferences. The source of these estimates and inference are classified into:

 Direct sources of studying the interior of the earth help in obtaining


information through direct observations and analysis of materials, which are
inside the earth like rock materials from mining areas and molten magma from
volcanic eruptions etc.
 Indirect sources include studying similar objects from space like meteorites
that reach the earth, gravitation, magnetic field, and seismic activity.

Statement 1 is incorrect: Drilling of the earth's surface directly exposes the structure


of the earth. Drilling provided information about the interior of the earth through
analysis of materials collected at different depths.

Statement 2 is correct: The origin of the earth and other heavenly bodies have
followed similar mechanisms. Therefore, the material and the structure observed in
the meteorites are similar to that of the earth. This provides us with indirect
information about the interior of the earth.

Statement 3 is incorrect: Volcanic eruptions bring out the matter inside the earth to
the outer surface. By analyzing those materials, based on their physical and chemical
properties, the interior of earth can be studied.

Statement 4 is correct: The study of seismic waves provides a complete picture of


the layered interior of earth without analysing any sample of earth's interior. The
different types of waves helps us in understanding the properties of different materials
in the interior of the earth.

व्याख्या:

पृ थ्वी के आं तरिक भाग के बारे में हमारा अधिकां श ज्ञान आं कलनों और अनु मानों पर आधारित
है । इन आं कलनों और अनु मानों के स्रोत को निम्नलिखित रूप में वर्गीकृत किया गया है :

 पृ थ्वी के आं तरिक भाग के अध्ययन के लिए उपयोग किए जाने वाले  प्रत्यक्ष
स्रोत पृ थ्वी के भीतर मौजूद पदार्थों के प्रत्यक्ष अवलोकन और विश्ले षण के माध्यम से
जानकारी प्राप्त करने में सहायता करते हैं । इन पदार्थों में खनन क्षे तर् ों से प्राप्त
चट् टानें और ज्वालामु खीय विस्फोटों से निकला हुआ पिघला मै ग्मा आदि शामिल है ।
 अप्रत्यक्ष स्रोतों में अं तरिक्ष से आने वाली वस्तु ओं जै से उल्का पिं ड का अध्ययन
शामिल है ,जो पृ थ्वी पर पहुँचता है । अन्य स्रोतों में   गु रुत्वाकर्षण, चु ं बकीय क्षे तर् और
भूकंपीय गतिविधि आदि शामिल हैं ।

कथन 1 गलत है : पृ थ्वी की सतह का खनन पृ थ्वी की सं रचना के बारे में प्रत्यक्ष जानकारी
प्रदान करता है ।भिन्न-भिन्न गहराईयों पर खनन के माध्यम से एकत्रित किए गए पदार्थों के
विश्ले षण से पृ थ्वी के आं तरिक भाग के बारे में जानकारी प्राप्त होती है ।

कथन 2 सही है : पृ थ्वी और अन्य खगोलीय पिं डों की उत्पत्ति समान क्रियाविधि के माध्यम से
हुई है । इसलिए, उल्काओं से प्राप्त सामग्री और उनकी सं रचना पृ थ्वी के समान होती है । यह
हमें पृ थ्वी के आं तरिक भाग के बारे में अप्रत्यक्ष रूप से जानकारी प्रदान करता है ।

कथन 3 गलत है : ज्वालामु खी विस्फोट पृ थ्वी के भीतर मौजूद पदार्थों को बाहरी सतह पर लाते
हैं । उन पदार्थों का विश्ले षण करके, उनके भौतिक और रासायनिक गु णों के आधार पर, पृ थ्वी के
आं तरिक भाग का अध्ययन किया जा सकता है ।
कथन 4 सही है : भूकंपीय तरं गों के अध्ययन से पृ थ्वी के आं तरिक भाग के किसी भी नमूने का
विश्ले षण किए बिना पृ थ्वी के स्तरित आं तरिक भाग की पूरी तस्वीर प्राप्त हो जाती है ।
विभिन्न प्रकार की तरं गें पृ थ्वी के आं तरिक भाग में मौजूद विभिन्न पदार्थों के गु णों को समझने
में हमारी मदद करती हैं ।
 31 s

2.0/2.0 

27
Q. Which of the following are the necessary conditions for the development of
tropical cyclones?

1. Large sea surface with temperature higher than 27° C.


2. Absence of the Coriolis force.
3. Minor vertical wind speed variations.
4. Pre-existing weak low-pressure area.
5. Upper divergence above the sea level system.

Select the correct answer using the codes given below: 

Q. उष्णकटिबं धीय चक् रवातों के विकास के लिए निम्नलिखित में से कौन सी आवश्यक परिस्थितियाँ
हैं ?

1. 27° C से अधिक तापमान के साथ विस्तृ त समु दर् ी सतह।


2. कोरिओलिस बल की अनु पस्थिति।
3. ऊर्ध्वाधर हवा की गति में मामूली बदलाव।
4. पहले से मौजूद दुर्बल निम्न दाब क्षे तर् ।
5. समु दर् तल प्रणाली के ऊपर ऊपरी विचलन।

निम्नलिखित कू टों का उपयोग करके सही उत्तर चु निए:

A  2, 4 and 5 only


केवल 2, 4 और 5 

B  1, 3, 4 and 5 only


केवल 1, 3, 4 और 5

C  1 and 3 only
केवल 1 और 3

D  1, 4 and 5 only


केवल 1, 4 और 5
Solution : B
Explanation:

The necessary conditions for the development of tropical cyclones:

Statement 1 is correct: Large sea surface and temperature higher than 27° C.

Statement 2 is incorrect: Presence of the strong Coriolis force is necessary for the


formation of the tropical cyclones. This is why cyclones are not formed at the Equator
as Coriolis force is zero there.

Statement 3 is correct: Minor vertical wind speed variations.

Statement 4 is correct: Pre-existing weak- low-pressure area.

Statement 5 is correct: Upper divergence above the sea level system. A well-


developed divergence in the upper layers of the atmosphere is necessary so that the
rising air currents within the cyclone continue to be pumped out and a low pressure
maintained at the centre.

व्याख्या:

उष्णकटिबं धीय चक् रवातों के विकास के लिए आवश्यक परिस्थितियाँ :

कथन 1 सही है : विस्तृ त समु दर् ी सतह और 27°C से अधिक तापमान। 

कथन 2 गलत है : उष्णकटिबं धीय चक् रवातों के निर्माण के लिए प्रबल कोरिओलिस बल की
उपस्थिति आवश्यक है । यही कारण है कि भूमध्य रे खा के अं तर्गत आने वाले क्षे तर् ों में
चक् रवात का निर्माण नहीं होता है क्योंकि वहाँ कोरिओलिस बल शून्य होता है ।

कथन 3 सही है : ऊर्ध्वाधर हवा की गति में मामूली बदलाव।

कथन 4 सही है : पहले से मौजूद दुर्बल निम्न दाब क्षे तर् ।

कथन 5 सही है : समु दर् तल प्रणाली के ऊपर ऊपरी विचलन। वायु मं डल की ऊपरी परतों में
अच्छी तरह से विकसित विचलन का होना आवश्यक है ताकि चक् रवात के भीतर की वायु
धाराएं बाहर निकलती रहें और केंद्र में निम्न दाब क्षे तर् बना रहे ।
 44 s

28
Q. With reference to the lava plateaus, which of the following statements
is/are incorrect?

1. They are rich in red and yellow soil. 


2. They are infertile and not suitable for cultivation. 
3. Columbia Plateau in the United States is a lava plateau. 

Select the correct answer using the codes given below: 


Q. लावा पठार के सं दर्भ में , निम्नलिखित कथनों में से कौन सा/से  गलत है /हैं ?

1. ये लाल और पीली मिट् टी से समृ द्ध होते हैं ।


2. ये बं जर होते हैं और कृषि के लिए उपयु क्त नहीं होते हैं ।
3. सं युक्त राज्य अमे रिका में कोलंबिया का पठार एक लावा पठार है ।

निम्नलिखित कू टों का उपयोग करके सही उत्तर चु निए:

A  1 only
केवल 1

B  1 and 2 only
केवल 1 और 2

C  2 and 3 only
केवल 2 और 3

D  1, 2 and 3
1, 2 और 3
Solution : B

Explanation: 
Lava plateaus are the landforms created by volcanic activity. When a volcano erupts,
magma which is primarily made up of basalt, spreads and hardens into layers which
build up over time to create the lava plateau. 
Statement 1 is incorrect: Lava plateaus are rich in black soil. 
Statement 2 is incorrect: Lava plateaus are very fertile as they are rich in black
soil and hence they are good for cultivation. 
Statement 3 is correct: Deccan Plateau in India and the Columbia Plateau in the
United States are examples of lava plateau. 
व्याख्या:
लावा पठार ज्वालामु खीय गतिविधि द्वारा निर्मित भू-आकृतियाँ हैं । जब एक ज्वालामु खी विस्फोटित
होता है , तो मु ख्य रूप से बे साल्ट से निर्मित मै ग्मा फ़ैल जाता है और परतों में जम जाता है जो समय
बीतने के साथ लावा पठार का निर्माण करता है ।
कथन 1 गलत है : लावा पठार काली मिट् टी से समृ द्ध होते हैं ।
कथन 2 गलत है : लावा पठार बहुत उपजाऊ होते हैं क्योंकि वे  काली मिट्टी से समृ द्ध होते हैं और
इसलिए वे कृषि के लिए उपयु क्त हैं ।
कथन 3 सही है : भारत में  दक्कन का पठार और सं युक्त राज्य अमे रिका में  कोलंबिया पठार लावा
पठार के उदाहरण हैं ।
 2 m 5 s

-0.67/2.0 

29
Q. Consider the following pairs:
 
S. no Indigenous people Associated Region
1. Bedouins Amazon Basin
2. Pygmies Horn of Africa
3. Bushmen Kalahari Desert

Which of the above pairs is/are correctly matched?

Q. निम्नलिखित यु ग्मों पर विचार कीजिए:


 
क् रम सं ख्या  आदिवासी सं बद्ध क्षे तर्
1. बद्द ू अमे ज़न घाटी
2. पिग्मी हॉर्न ऑफ़ अफ् रीका
3. बु शमै न कालाहारी मरुस्थल

उपर्युक्त यु ग्मों में से कौन सा/से सु मेलित है /हैं ?

A  1 and 2 only
केवल 1 और 2

B  2 and 3 only
केवल 2 और 3

C  3 only
केवल 3

D  2 only
केवल 2
Solution : C
Explanation: 
Pair 1 is incorrectly matched: Bedouins are a desert-dwelling, Arab nomadic
pastoralist group, who are located in the regions of Sahara and the Arabian
Deserts. 
Pair 2 is incorrectly matched: Pygmies are a group of ethnicities native to Central
Africa, mostly the Congo Basin, traditionally subsisting on a foraging and hunter-
gathering lifestyle. 
Pair 3 is correctly matched: The Bushmen are the hunter-gatherer people who
live in the Kalahari desert. 
व्याख्या:
यु ग्म 1 सुमेलित नहीं है : बद्द मरुस्थल-निवासी,
ू अरब खानाबदोश चरवाहा समु दाय हैं , जो सहारा और
अरब मरुस्थलीय क्षे तर् ों में रहते हैं ।
यु ग्म 2 सुमेलित नहीं है : पिग्मी मध्य अफ् रीका के मूल निवासी नृ जातीय लोगों का एक समूह है । ये
अधिकतर कांगो नदी घाटी में निवास करते हैं । ये पारं परिक रूप से घु मन्तु और शिकारी जीवन शै ली
आधारित जीवन जीते हैं ।
यु ग्म 3 सुमेलित है : बुशमै न शिकारी लोग हैं जो कालाहारी मरुस्थल में रहते हैं ।
 1 m 16 s

-0.67/2.0 

30
Q. The weather of Mussoorie on 7th of February, 2021 is described as having clear
skies, calm air, the relative humidity of 85%, and that the air temperature is at -1° C,
Which of the following is witnessed by the trainees at Lal Bahadur Shastri National
Academy of Administration at 06: 00 AM during their physical training session?

Q. 7 फरवरी, 2021 को मसूरी के मौसम को स्वच्छ आसमान, शांत हवा, 85% की सापे क्षिक आर्द्रता
और -1°C वायु तापमान के रूप में वर्णित किया गया। लाल बहादुर शास्त्री राष्ट् रीय प्रशासनिक
अकादमी में प्रशिक्षु 06: 00 AM पर अपने शारीरिक प्रशिक्षण सत्र के दौरान निम्नलिखित में से
किस के साक्षी होंगे ?

A  Dew
ओस

Fog
कोहरा

C  Mist
धुं ध

D  Frost 
पाला
Solution : D

Explanation:

The ideal conditions for the formation of dew, fog mist and frost are the clear skies,
calm air, high relative humidity, cold and long nights. The difference is where the
temperature of the air is below or above the freezing point of water.

Option (a) is incorrect: Dew is the moisture that is deposited in the form of water
droplets on cooler surfaces of solid objects. For the formation of dew, it is necessary
that the air temperature is above the freezing point.

Option (b) is incorrect: Fog is a cloud with its base at or very near to the ground. For
the formation of fog, it is necessary that the air temperature is above the freezing
point. 

Option (c) is incorrect: The only difference between the mist and fog is that mist
contains more moisture than the fog.

Option (d) is correct: Frost forms on cold surfaces when condensation takes place
below the freezing point. The excess moisture is deposited in the form of minute ice
crystals instead of water droplets. The freezing point of water is 0° C, Here, -1° C is
below the freezing point of water and 7th of February is a cold long night in
Mussoorie because it is winter there and 85% is high relative humidity. Hence,
Frost is the correct answer.

व्याख्या:

ओस, कोहरा, धुं ध और पाले के लिए आदर्श परिस्थितियाँ स्वच्छ आसमान, शांत हवा, उच्च
सापे क्षिक आर्द्रता, ठं डी और लं बी रातें हैं । अं तर केवल हवा के तापमान का पानी के हिमांक
बिं दु से नीचे या ऊपर का होता है ।

विकल्प (a) गलत है : ओस वह आर्द्रता है जो ठोस वस्तु ओं की ठं डी सतहों पर पानी की बूंदों के
रूप में निक्षे पित होती है । ओस के निर्माण के लिए, यह आवश्यक है कि हवा का तापमान हिमांक
बिं दु से ऊपर हो।

विकल्प (b) गलत है : कोहरा एक ऐसा बादल है जिसका आधार भूमि सतह पर या इसके बहुत
करीब होता है । कोहरे के निर्माण के लिए, यह आवश्यक है कि हवा का तापमान हिमांक बिं दु से
ऊपर हो।

विकल्प (c) गलत है : धुं ध और कोहरे के बीच एकमात्र अं तर यह है कि धुं ध में कोहरे की तु लना
में अधिक आर्द्रता होती है ।

विकल्प (d) सही है : जब हिमांक बिं दु से कम ताप पर सं घनन होता है , तब ठं डी सतहों पर पाले
का निर्माण होता है । अतिरिक्त आर्द्रता पानी की बूंदों के बजाय बर्फ के सूक्ष्म क्रिस्टल के रूप
में निक्षे पित हो जाती है । पानी का हिमांक बिं दु 0°C होता है और प्रश्न में दिया गया वायु
तापमान -1°C है जो पानी के हिमांक बिं दु से कम है और 7 फरवरी को मसूरी में ठं डी लं बी रात
होती है , क्योंकि इस समय वहाँ सर्दी रहती है और 85% सापे क्षिक आर्द्रता बहुत उच्च आर्द्रता
है । इसलिए, पाला सही उत्तर है ।
 37 s

-0.67/2.0 

31
Q. With reference to the interior of the Earth, which of the following statements is/are
correct?

1. The oceanic crust is granitic in nature.


2. The core is denser than mantle whereas mantle is denser than the crust.
3. The uppermost portion of the core is called the asthenosphere.

Select the correct answer using the codes given below:

Q. पृ थ्वी के आं तरिक भाग के सं दर्भ में , निम्नलिखित कथनों में से कौन सा/से सही है /हैं ?

1. महासागरीय पर्पटी की प्रकृति ग्रेनाइटिक होती है ।


2. क् रोड मैं टल की तु लना में सघन होता है , जबकि मैं टल पर्पटी की तु लना में सघन होता है ।
3. क् रोड के ऊपरी भाग को दुर्बल मं डल(asthenosphere) कहा जाता है ।

निम्नलिखित कू टों का उपयोग करके सही उत्तर चु निए:

A  1 and 2 only
केवल 1 और 2

B  2 only
 केवल 2

C  2 and 3 only
केवल 2 और 3

D  1 and 3 only
केवल 1 और 3
Solution : B

Explanation:

Statement 1 is incorrect: The oceanic crust is basaltic in nature and continental


crust is granitic in nature. Also, the thickness of the crust varies under the oceanic and
continental areas.

Statement 2 is correct: As we go inside from the crust to the core the material
becomes denser. Hence, the core is denser than mantle and mantle is denser than the
crust. The density of materials at the centre of the earth is around 13g/cm3. The density
of materials in the mantle is 3. 4 g/cm3. The mean density of materials in the crust is 2.
7 g/cm3.

Statement 3 is incorrect: The uppermost portion of the mantle is called the


asthenosphere. It is the main source of magma.

व्याख्या:

कथन 1 गलत है : महासागरीय पर्पटी प्रकृति में बेसाल्टिक होती है और महाद्वीपीय पर्पटी


प्रकृति में ग्रेनाइटिक होती है । इसके अलावा, महासागरीय और महाद्वीपीय क्षे तर् ों में पर्पटी
की मोटाई अलग-अलग होती है ।

कथन 2 सही है : जै से-जै से हम पर्पटी से क् रोड के अं दर जाते हैं , पदार्थ सघन होता जाता है ।
इसलिए, क् रोड मैं टल की तु लना में सघन होता है और मैं टल पर्पटी की तु लना में सघन होता
है । पृ थ्वी के केंद्र में पदार्थों का घनत्व लगभग 13g/cm3 होता है । मैं टल में पदार्थों का घनत्व
3.4 g/cm3 होता है । पर्पटी में पदार्थों का औसत घनत्व 2.7 g/cm3 होता है ।

कथन 3 गलत है : मैं टल के ऊपरी भाग को दुर्बल मं डल कहा जाता है । यह मै ग्मा का मु ख्य
स्रोत है ।
 1 m 26 s

-0.67/2.0 

32
Q. The term ‘AIM-iCREST’ recently in the news, is related to:

Q. हाल ही में चर्चा में रहा 'एआईएम-आईसीआरईएसटी' (AIM-iCREST) पद किससे सं बंधित है ?

A  Vaccine development for the Neglected Tropical Diseases (NTD) in India.


भारत में उपे क्षित उष्णकटिबं धीय रोगों (NTD) के लिए टीका विकास से ।

Promotion of electric vehicles manufacturing in India.
भारत में इले क्ट्रिक वाहन निर्माण को बढ़ावा दे ने से ।

C  Creation of high performing startups in India.


भारत में उच्च प्रदर्शन वाले स्टार्टअप के सृ जन से ।

D  Indigenous defence equipment manufacturing in India.


भारत में स्वदे शी रक्षा उपकरण विनिर्माण से ।
Solution : C

Explanation:

NITI Aayog’s Atal Innovation Mission (AIM), has launched AIM iCREST – an
Incubator Capabilities enhancement program for a Robust Ecosystem focused on
creating high performing Startups.
व्याख्या:

नीति आयोग के अटल नवाचार मिशन (AIM) ने एक मजबूत पारिस्थितिकी तं तर् के लिए एक
् कार्यक् रम एआईएम-आईसीआरईएसटी (AIM-iCREST) शु रू किया है जो
इनक्यूबेटर क्षमता वृ दधि
उच्च प्रदर्शन वाले स्टार्टअप बनाने पर केंद्रित है ।
 46 s

2.0/2.0 

33
Q. With reference to humidity, which of the following statements are correct?

1. The ability of air to hold water vapour depends entirely on its temperature. 
2. The mass of water vapor present in a unit volume of air is known as absolute
humidity. 
3. Humidity often mentioned in the weather reports, in percentage terms, is absolute
humidity. 

Select the correct answer using the codes given below: 


Q. आर्द्रता के सं दर्भ में , निम्नलिखित कथनों में से कौन से कथन सही हैं ?

1. जल वाष्प धारण करने की वायु की क्षमता सं पर्ण


ू तया उसके तापमान पर निर्भर करती है ।
2. वायु की इकाई आयतन में उपस्थित जलवाष्प का द्रव्यमान निरपे क्ष आर्द्रता के रूप में जाना जाता
है ।
3. मौसम रिपोर्ट में प्रायः प्रतिशत में उल्लिखित आर्द्रता निरपे क्ष आर्द्रता होती है ।
निम्नलिखित कू टों का उपयोग करके सही उत्तर चु निए:

A  1 and 2 only
केवल 1 और 2

B  1 and 3 only
केवल 1 और 3

C  2 and 3 only
केवल 2 और 3

D  1, 2 and 3
1, 2 और 3
Solution : A

Explanation:

Humidity is the concentration of water vapour present in the air.


Statement 1 is correct: The ability of air to hold water vapour depends entirely on
its temperature. For instance, warm air can hold more moisture than cold air. 
Statement 2 is correct: Absolute Humidity is the actual amount of water vapour
present in the atmosphere. It is the mass of water vapour per unit volume of air and
expressed in grams per cubic metre. 
Statement 3 is incorrect: The humidity often mentioned in the weather reports
is relative humidity, not absolute humidity.

Figure: Weather Report for Delhi


In the above image, 58% Relative humidity means air contains 58% of water
vapour compared to how much it can hold at that temperature. 
व्याख्या:
आर्द्रता वायु में उपस्थित जल वाष्प की मात्रा होती है ।
कथन 1 सही है : जल वाष्प धारण करने की वायु की क्षमता सं पर्णू तया उसके तापमान पर निर्भर करती
है । उदाहरण के लिए, गर्म हवा ठं डी हवा की तु लना में अधिक नमी धारण कर सकती है ।
कथन 2 सही है : निरपे क्ष आर्द्रता वायु मंडल में उपस्थित जलवाष्प की वास्तविक मात्रा होती है । यह
वायु के प्रति इकाई आयतन में उपस्थित जल वाष्प का द्रव्यमान होता है और इसे ग्राम प्रति घन
मीटर में व्यक्त किया जाता है ।
कथन 3 गलत है : मौसम रिपोर्ट में उल्लिखित आर्द्रता सापे क्ष आर्द्रता होती है , न कि निरपे क्ष
आर्द्रता।

चित्र: दिल्ली के लिए मौसम रिपोर्ट


उपर्युक्त चित्र में , 58% सापे क्ष आर्द्रता का अर्थ है  कि समान ताप पर वायु द्वारा धारण की जा सकने
वाली जल वाष्प की अधिकतम मात्रा की तु लना में  वायु में 58% जल वाष्प उपस्थित है ।
 0 s

2.0/2.0 

34
Q. Which of the following are the components of the Human Development Index
(HDI) published by the United Nations Development Programme (UNDP)?

1. Life expectancy at birth


2. Mean years of schooling
3. Infant Mortality Rate
4. Expected years of Schooling
Select the correct answer using the codes given below:

Q. सं युक्त राष्ट् र विकास कार्यक् रम (UNDP) द्वारा प्रकाशित मानव विकास सूचकांक (HDI) के
निम्नलिखित में से कौन से घटक हैं ?

1. जन्म के समय जीवन प्रत्याशा


2. शिक्षा का औसत वर्ष
3. शिशु मृ त्यु दर
4. स्कू ली शिक्षा के अपे क्षित वर्ष

निम्नलिखित कू टों का उपयोग करके सही उत्तर चु निए:

A  2 and 3 only
केवल 2 और 3 

B  1, 2 and 4 only


केवल 1, 2 और 4

C  3 and 4 only
केवल 3 और 4

D  1, 3 and 4 only


केवल 1, 3 और 4
Solution : B

Explanation: 

The HDI was created to emphasize that people and their capabilities should be the
ultimate criteria for assessing the development of a country, not economic growth
alone.

The Human Development Index (HDI) is a summary measure of average achievement


in key dimensions of human development: a long and healthy life,
being knowledgeable and having a decent standard of living. The HDI is the
geometric mean of normalized indices for each of the three dimensions.

It is published by the United Nations Development Programme (UNDP). India


stands at 131st position out of 189 countries in 2019. (As per the Human Development
Report 2020).
Statement 1 is correct: Life Expectancy at birth is the only indicator which is used in
measuring the dimension of a long and healthy life.

Statement 2 and 4 correct: Mean years of schooling and expected years of schooling


are the indicators used in measuring the dimension of being Knowledgeable.

Statement 3 is incorrect: Infant Mortality Rate is not the indicator used in measuring


the dimension of a long and healthy life.

व्याख्या:

किसी दे श के विकास का आकलन करने के लिए उस दे श के लोगों और उनकी क्षमताओं को


अं तिम मानदं ड माना जाना चाहिए, न कि केवल आर्थिक विकास को। इसी बात को ध्यान में
रखते हुए मानव विकास सूचकांक (HDI) को डिज़ाइन किया गया था। मानव विकास सूचकांक
(HDI) मानव विकास के प्रमु ख आयामों (एक लंबा और स्वस्थ जीवन, ज्ञानवान होना और
जीवन स्तर का एक उचित मानक) में औसत उपलब्धि का एक सारां श है । HDI तीनों आयामों
में से प्रत्ये क के लिए सामान्यीकृत सूचकांकों का ज्यामितीय माध्य है ।

यह सं युक्त राष्ट् र विकास कार्यक् रम (UNDP) द्वारा प्रकाशित किया जाता है । मानव विकास
रिपोर्ट 2020 के अनु सार, भारत 2019 में 189 दे शों में से 131 वें स्थान पर है ।

कथन 1 सही है : जन्म के समय जीवन प्रत्याशा एकमात्र सं केतक है जिसका उपयोग लं बे
और स्वस्थ जीवन के आयाम को मापने में किया जाता है ।

कथन 2 और 4 सही: स्कू ली शिक्षा के औसत वर्ष और स्कू ली शिक्षा के अपे क्षित वर्ष ऐसे
सं केतक हैं , जिसका उपयोग ज्ञानवान होने सं बंधी आयाम को मापने में किया जाता है ।

कथन 3 गलत है : शिशु मृ त्यु दर एक लं बे और स्वस्थ जीवन के आयाम को मापने में उपयोग
किया जाने वाला सं केतक नहीं है ।
 1 m 21 s

2.0/2.0 

35
Q. With reference to the Consumer Protection Act, 2019, consider the following
statements:

1. It provides for product liability in India.


2. It covers e-commerce retailers. 
3. It has a provision for appeal against the settlement through mediation.

Which of the statements given above is/are correct?

Q.  उपभोक्ता सं रक्षण अधिनियम, 2019 के सं दर्भ में , निम्नलिखित कथनों पर विचार कीजिए:

1. यह भारत में उत्पाद दायित्व (Product Liability) का प्रावधान करता है ।


2. इसके अं तर्गत ई-कॉमर्स रिटे लर्स शामिल हैं ।
3. इसके अं तर्गत मध्यस्थता द्वारा किए गए समझौते के विरूद्ध अपील का प्रावधान है ।

उपर्युक्त कथनों में से कौन सा/से सही है / हैं ?

A  1 and 2 only
केवल 1 और 2

B  2 only
केवल 2

C  3 only
केवल 3

D  1, 2 and 3
1, 2 और 3
Solution : A

Explanation:

The Consumer Protection Act, 2019 came into force from 20th July 2020.

Statement 1 is correct: The Act introduces the concept of product which creates


liability for any harm caused by a defective product bought from manufacturer, seller
or service providers.

Statement 2 is correct: The new Act provides protection to the buyers of e-


commerce retailers/platforms in addition to traditional sellers.

Statement 3 is incorrect: There is an elaborate provision for mediation under the


Consumer Protection Act, 2019. There will be no appeal against settlement
through mediation.

व्याख्या:

उपभोक्ता सं रक्षण अधिनियम, 2019, 20 जु लाई 2020 से प्रभावी हुआ है ।

कथन 1 सही है : इस अधिनियम के अं तर्गत उत्पाद की अवधारणा का प्रावधान किया गया है
जो निर्माता, विक् रे ता या से वा प्रदाताओं से खरीदे गए दोषपूर्ण उत्पाद के कारण होने वाले
किसी भी नु कसान के लिए दायित्व का निर्धारण करता है ।

कथन 2 सही है : नए अधिनियम में पारं परिक विक् रे ताओं के अलावा ई-कॉमर्स खु दरा
विक् रे ताओं / प्ले टफार्मों के खरीदारों को भी सु रक्षा प्रदान करने का प्रावधान है ।

कथन 3 गलत है : उपभोक्ता सं रक्षण अधिनियम, 2019 के अं तर्गत मध्यस्थता के लिए एक


विस्तृ त प्रावधान किया गया है । मध्यस्थता के माध्यम से किए गए समझौते   के विरूद्ध कोई
अपील नहीं होगी।
 1 m 38 s

36
Q. Arrange the following discontinuities in the interior layers of the earth from top to
bottom:

1. Lehmann
2. Conrad
3. Mohorovic
4. Repiti

Select the correct answer: 

Q. पृ थ्वी की आं तरिक परतों में विद्यमान निम्नलिखित असं बद्धताओं (Discontinuities) को ऊपर से
नीचे के क् रम में व्यवस्थित कीजिए:

1. ले हमै न
2. कॉनराड
3. मोहरोविक
4. रे पिटी

निम्नलिखित क् रमों में से सही उत्तर का चयन कीजिए :

A  2-3-4-1
2-3-4-1

B  1-2-4-3
1-2-4-3

C  3-2-1-4
3-2-1-4

D  3-4-2-1
3-4-2-1
Solution : A
Explanation:

Seismic discontinuities are the regions in the earth’s interior where seismic waves
behave differently compared to the surrounding areas as these discontinuities have
marked differences in terms of physical or chemical properties.

1. Conrad: The discontinuity between the continental crust and oceanic crust is


termed as the Conrad Discontinuity. Thus it acts as a transition zone between
SiAl (Silica-Alumina or Continental Crust) and SiMa (Silica-Magnesia or
Oceanic Crust).
2. Mohorovic: Also known as Moho discontinuity, it separates the crust and
mantle.
3. Repiti: It separates the upper mantle from the lower mantle. Due to the
presence of this discontinuity, there is a sharp increase in the velocity of
seismic waves.
4. Lehmann: It separates the outer core from the inner core

             Figure: earth’s internal


discontinuity.

व्याख्या:

भूकंपीय असं बद्धताएँ पृ थ्वी के वै से आं तरिक क्षे तर् हैं जहाँ आस-पास के क्षे तर् ों की तु लना में
भूकंपीय तरं गों का व्यवहार अलग होता है क्योंकि इन असं बद्धताओं के  भौतिक अथवा
रासायनिक गु णों में भिन्नता होती है ।

1. कॉनराड: महाद्वीपीय भूपर्पटी और महासागरीय पर्पटी के बीच के असं बद्धता को कॉनराड


असं बद्धता(Conrad Discontinuity) कहा जाता है । इस प्रकार यह SiAl (सिलिका-
एल्यूमिना या महाद्वीपीय भूपटल) और SiMa (सिलिका-मै ग्ने शिया या महासागरीय
पर्पटी) के मध्य एक सं क्रमण क्षे तर् के रूप में कार्य करता है ।
2. मोहरोविक: इसे मोहो असं बद्धता (Moho Discontinuity) के रूप में भी जाना जाता है ,
यह क् रस्ट और में टल को पृ थक करता है ।
3. रे पिटी: यह ऊपरी में टल को निचले में टल से पृ थक करता है । इस असं बद्धता की
् होती है ।
उपस्थिति के कारण भूकंपीय तरं गों के वे ग में ते ज वृ दधि
4. ले हमैन: यह बाहरी कोर को आं तरिक कोर से पृ थक करता है ।

                   चित्र: पृ थ्वी की आंतरिक


असंबद्धता
 1 m 2 s

-0.67/2.0 

37
Q. Consider the following statements regarding the Anti-Tank Spike Missile, which
was recently in news:

1. It is developed by the Defence Research and Development Organisation (DRDO). 


2. It has a "fire and forget" mode of operation.
3. It is a long-range missile.

Which of the statements given above is/are correct?

Q. हाल ही में चर्चा में रहे एं टी टैं क मिसाइल स्पाइक के बारे में निम्नलिखित कथनों पर विचार
कीजिए:

1. यह रक्षा अनु संधान और विकास सं गठन (Defence Research and Development


Organisation-DRDO) द्वारा विकसित किया गया है ।
2. यह  "दागो और भूल जाओ" (Fire-and-Forget) कार्य प्रणाली पर आधारित है ।
3. यह लं बी दरू ी की मिसाइल है ।

उपर्युक्त कथनों में से कौन सा/से सही है / हैं ?

A  1 only
केवल 1 

B  2 and 3 only
केवल 2 और 3
C  1 and 3 only
केवल 1 और 3

D  1, 2 and 3 
1, 2 और 3
Solution : B

Explanation:

The Government of India recently procured 4th generation Anti-Tank Spike Long-
Range Guided-missile which is developed and manufactured by the Rafael Advanced
Defence Systems of Israel.

Statement 1 is incorrect: Spike missile is developed and manufactured by the Rafael


Advanced Defence Systems of Israel and not by DRDO.

Statement 2 is correct: It can be operated in fire-and-forget mode or fire, observe and


update mode using the fiber-optic data link.

Statement 3 is correct: It is a long-range missile with 4 km range.

व्याख्या:

हाल ही में , भारत सरकार ने चौथी पीढ़ी के एं टी-टैं क स्पाइक लॉन्ग-रें ज गाइडे ड-मिसाइल
(Anti-Tank Spike Long-Range Guided-missile) की खरीद की है जो कि इज़राइल के राफेल
एडवां स्ड डिफेंस सिस्टम (Rafael Advanced Defence System) द्वारा विकसित एवं निर्मित है ।

कथन 1 गलत है : स्पाइक मिसाइल का विकास एवं निर्माण रक्षा अनुसंधान और विकास संगठन
(Defence Research and Development Organisation-DRDO) ने नहीं, अपितु  इज़राइल
के राफेल एडवांस्ड डिफेंस सिस्टम  ने  किया है  ।

कथन 2 सही है : इसे फाइबर-ऑप्टिक डे टा लिं क का उपयोग करके 'दागो और भूल जाओ'
(Fire and Forget) अथवा या 'दागो, दे खो और फिर निशाना साधो' (Fire, Observe and
Update) मोड में सं चालित किया जा सकता है ।

कथन 3 सही है : यह एक लं बी दरू ी की मिसाइल है जिसकी मारक क्षमता चार किलोमीटर तक
है ।
 1 m 26 s

2.0/2.0 

38
Q. With reference to the Coriolis Force, consider the following statements:
1. Its magnitude is directly proportional to the angle of latitude. 
2. It acts parallel to isobars at the equator.
3. Geostrophic winds are parallel to the isobars due to absence of coriolis force.

Select the correct answer using the codes given below: 

Q. कोरिओलिस बल (Coriolis Force) के सं दर्भ में निम्नलिखित कथनों पर विचार कीजिए:

1. इसका परिमाण अक्षां श के कोण के प्रत्यक्ष आनु पातिक होता है ।


2. यह भूमध्य रे खा पर समदाब रे खा (Isobar) के समानांतर कार्य करता है ।
3. भूविक्षे पी पवनें (Geostrophic Winds) कोरिओलिस बल की अनु पस्थिति के कारण समदाब रे खा के
समानांतर होती हैं ।

निम्नलिखित कू टों का उपयोग करके सही उत्तर चु निए:

A  1 only
केवल 1

B  2 and 3 only
केवल 2 और 3

C  1 and 3 only
केवल 1 और 3

D  1, 2 and 3
1, 2 और 3
Solution : A

Explanation:

Statement 1 is correct: Coriolis force is directly proportional to the angle of latitude.


Higher the latitude, higher the Coriolis force. It is zero at the equator and maximum at
poles.

Statement 2 is incorrect: Coriolis force acts perpendicular (not parallel) to isobars


at the equator and thus the low pressure gets filled at the equator. This is the reason
why cyclones do not develop at the equator.

Statement 3 is incorrect: Geostrophic winds are formed in the upper atmosphere.


Due to the absence of friction force, pressure gradient force balances the Coriolis
force. Hence these winds flow parallel to the isobars.

व्याख्या:

कथन 1 सही है : कोरिओलिस बल अक्षां श के कोण के प्रत्यक्ष आनु पातिक होता है । जितना
उच्च अक्षां श होगा उतना ही अधिक कोरिओलिस बल होगा। यह भूमध्य रे खा पर शून्य और
ध्रुवों पर अधिकतम होता है ।
कथन 2 गलत है : भूमध्य रे खा पर कोरिओलिस बल समदाब रे खा के लंबवत (समानांतर
नहीं) कार्य करता है और इस तरह वहां कम दबाव का क्षे तर् बन जाता है । यही कारण है कि
भूमध्य रे खा पर चक् रवात उत्पन्न नहीं होते हैं ।

कथन 3 गलत है : भूविक्षे पी पवनों (Geostrophic winds) का निर्माण ऊपरी वातावरण में होता
है । घर्षण बल की अनु पस्थिति के कारण दबाव ढाल बल (Pressure Gradient Force)
कोरिओलिस बल को सं तुलित करता है । इसलिए ये पवनें समदाब रे खा के समानांतर बहती हैं ।
 53 s

39
Q. With reference to the Permanent Court of Arbitration (PCA), consider the
following statements: 

1. It was established under the United Nations Convention on the Law of the Sea
(UNCLOS).
2. Its award is binding without any appellate mechanism.
3. India is a member of PCA. 

Which of the statements given above is/are correct?

Q. मध्यस्थता के स्थायी न्यायालय (Permanent Court of Arbitration-PCA) के सं दर्भ में


निम्नलिखित कथनों पर विचार कीजिए:

1. इसकी स्थापना सं युक्त राष्ट् र समु दर् ी कानून सं धि (United Nations Convention on the Law of
the Sea-UNCLOS) के तहत की गई थी।
2. इसका निर्णय बिना किसी अपीलीय तं तर् के बाध्यकारी होता है ।
3. भारत PCA का सदस्य है ।

उपर्युक्त कथनों में से कौन सा/से सही है /हैं ?

A  1 and 2 only
केवल 1 और 2

3 only

केवल 3
 

C  2 and 3 only
केवल 2 और 3

D  1, 2 and 3
1, 2 और 3
Solution : C
Explanation:

In 2012, Enrica Lexie case, the Permanent Court of Arbitration in its judgment has
said that two Italian marines who shot dead two Indian fishermen, cannot be tried in
an Indian court.

Statement 1 is incorrect: The PCA was established in 1899 by the first Hague Peace
Conference under Articles 20 to 29 of the 1899 Hague Convention for the Pacific
Settlement of International Disputes.

However, when signing, ratifying, or acceding to UNCLOS, a State may make a


declaration choosing one or more of the following means for settling such disputes:

 The International Tribunal for the Law of the Sea (ITLOS) in Hamburg,
Germany;
 The International Court of Justice in The Hague, The Netherlands;
 Ad hoc arbitration (in accordance with Annex VII of UNCLOS); or
 A “special arbitral tribunal” constituted for certain categories of disputes
(established under Annex VIII of UNCLOS).

Statement 2 is correct: Its decision is binding on the parties, and there is no


mechanism for appeal.

Statement 3 is correct: India is a member state of PCA. 

व्याख्या:

2012 में , एन्रीका ले क्सी वाद में मध्यस्थता के स्थायी न्यायालय (Permanent Court of
Arbitration-PCA) ने अपने फैसले में कहा था कि दो इतालवी नौसै निकों जिन्होंने दो भारतीय
मछुआरों की गोली मारकर हत्या कर दी थी, उनपर भारतीय अदालत में मु कदमा चलने की
अनु मति नहीं दी जा सकती।

कथन 1 गलत है : PCA की स्थापना 1899 में प्रथम हे ग शां ति सम्मे लन द्वारा अंतर्राष्ट्रीय
विवादों के प्रशांत निपटान हे तु हे ग अभिसमय (Hague Convention for the Pacific
Settlement of International Disputes) के अनु च्छे द 20 से 29 के तहत की गई थी।

हालाँ कि, UNCLOS पर हस्ताक्षर अथवा उसका अनु समर्थन करते समय एक दे श इस तरह के
विवादों को निपटाने के लिए निम्नलिखित में से एक या अधिक साधन चु नने की घोषणा कर
सकता है :

 इं टरने शनल ट्रिब्यूनल फॉर द लॉ ऑफ द सी (International Tribunal for the Law of


the Sea-ITLOS), है म्बर्ग, जर्मनी;
 अं तर्राष्ट् रीय न्यायालय (International Court of justice-ICJ), हे ग, नीदरलैं ड; 
 तदर्थ मध्यस्थता (UNCLOS के अनु लग्नक VII के अनु सार); या
 एक "विशे ष मध्यस्थ न्यायाधिकरण" जिसका गठन विवादों की कुछ श्रेणियों
(UNCLOS के अनु लग्नक VIII के तहत स्थापित) के लिए किया गया है ।

कथन 2 सही है : इसका निर्णय सभी पक्षकारों पर बाध्यकारी होता है जिसके विरुद्ध अपील के
लिए कोई तं तर् नहीं है ।

कथन 3 सही है : भारत PCA का सदस्य है ।


 1 m 50 s

2.0/2.0 

40
Q. With regards to Gross National Happiness (GNH), which of the following are its
pillars?

1. Sustainable and equitable socio-economic development.


2. Good governance.
3. Environmental conservation.
4. Preservation and promotion of culture.

Select the correct answer using the codes given below:

Q. सकल राष्ट् रीय खु शहाली (GNH) के सं बंध में , निम्नलिखित में से कौन से इसके स्तं भ हैं ?

1. सतत और न्यायसं गत सामाजिक-आर्थिक विकास


2. सु शासन
3. पर्यावरण सं रक्षण
4. सं स्कृति का सं रक्षण और सं वर्धन

निम्नलिखित कू टों का उपयोग करके सही उत्तर चु निए:

A  2 and 4 only
केवल 2 और 4

B  2, 3 and 4 only


केवल 2, 3 और 4 

C  1 and 3 only
केवल 1 और 3

D  1, 2, 3 and 4
1, 2, 3 और 4
Solution : D

Explanation:
Gross National Happiness (GNH) is a measure of economic and moral progress that
the country of Bhutan introduced in the 1970s as an alternative to Gross Domestic
Product (GDP). Rather than focusing strictly on quantitative economic measures,
gross national happiness takes into account an evolving mix of quality-of-life factors.

Statements 1, 2, 3 and 4 are correct: The ‘four pillars of GNH’, are:

1. Sustainable and equitable socio-economic development; 


2. Environmental conservation; 
3. The preservation and promotion of culture; and
4. Good governance

व्याख्या:

सकल राष्ट् रीय खु शहाली (GNH) आर्थिक और नै तिक प्रगति का एक मापक है जिसे 1970 के
दशक में भूटान ने सकल घरे लू उत्पाद (GDP) के विकल्प के रूप में पे श किया था। मात्रात्मक
आर्थिक उपायों पर दृढ़ता से ध्यान केंद्रित करने के बजाय, सकल राष्ट् रीय खु शहाली में जीवन
के गु णवत्ता वाले कारकों के एक विकसित मिश्रण को ध्यान में रखा जाता है ।

कथन 1, 2, 3 और 4 सही हैं : 'सकल राष्ट् रीय खु शहाली (GNH) के चार स्तं भ' हैं :

1. सतत और न्यायसं गत सामाजिक-आर्थिक विकास;


2. पर्यावरण सं रक्षण;
3. सं स्कृति का सं रक्षण और सं वर्धन; तथा
4. सु शासन

 48 s

2.0/2.0 

41
Q. Consider the following pairs:
 
S. No Name of Mars Mission Country
1 HOPE Japan
2 Mars Orbiter Mission India
3 Tianwen-1 China

Which of the pairs given above are correctly matched?

Q. निम्नलिखित यु ग्मों पर विचार कीजिए:


 
क् रम सं ख्या मं गल मिशन का नाम दे श
1 होप (HOPE) जापान
2 मार्स ऑर्बिटर मिशन भारत
3 तियानवे न-1 चीन

उपर्युक्त यु ग्मों में से कौन से सु मेलित हैं ?

A  1 and 2 only
केवल 1 और 2

B  2 and 3 only
केवल 2 और 3

C  1 and 3 only
केवल 1 और 3

D  1, 2 and 3
1, 2 और 3
Solution : B

Explanation:

Pair 1 is incorrectly matched: HOPE mission is the Mars mission of the United Arab
Emirates. It was launched in July 2020. It is the first interplanetary mission for any
country in the Arab World.

Pair 2 is correctly matched: The Mars Orbiter Mission, also called Mangalyaan, was
launched by the Indian Space Research Organisation (ISRO), India.

Pair 3 is correctly matched: Tianwen-1 is an interplanetary mission to Mars by the


China National Space Administration (CNSA).

व्याख्या:

यु ग्म 1 सुमेलित नहीं है : होप (HOPE) मिशन सं युक्त अरब अमीरात का मं गल मिशन है । यह
जु लाई 2020 में प्रक्षे पित किया गया था। यह अरब जगत में किसी भी दे श का पहला
अन्तर्ग्रहीय मिशन है ।

यु ग्म 2 सुमेलित है : मार्स ऑर्बिटर मिशन, जिसे मं गलयान भी कहा जाता है , भारतीय अं तरिक्ष
अनु संधान सं गठन (इसरो), भारत द्वारा प्रक्षे पित किया गया था।

यु ग्म 3 सुमेलित है :तियानवे न-1 चीनी राष्ट् रीय अं तरिक्ष प्रशासन (CNSA) द्वारा मं गल ग्रह
के लिए एक अन्तर्ग्रहीय मिशन है ।
 1 m 2 s

2.0/2.0 

42
Q. With reference to India’s in-Orbit space debris monitoring and surveillance
system, consider the following statements: 

1. It has been developed by Indian Space Research Organisation (ISRO). 


2. It uses a constellation of nanosatellites in Low Earth Orbit (LEO) to carry out the
surveillance. 
3. It is India’s first in-orbit space debris monitoring and tracking system. 

Which of the above statements is/are correct?


Q. भारत के इन-ऑर्बिट स्पे स डे बर् ीज मॉनिटरिं ग एं ड सर्विलांस सिस्टम (in-Orbit space debris
monitoring and surveillance system) के सं दर्भ में , निम्नलिखित कथनों पर विचार कीजिए: 

1. इसे भारतीय अं तरिक्ष अनु संधान सं गठन (ISRO) द्वारा विकसित किया गया है ।
2. सर्विलांस के लिए इस प्रणाली में निम्न भू-कक्षा (Low Earth Orbit- LEO) में स्थित लघु उपग्रहों
के समूह का उपयोग होता है ।
3. यह भारत का प्रथम इन-ऑर्बिट स्पे स डे बर् ीज मॉनिटरिं ग एं ड ट् रैकिंग सिस्टम है ।

उपर्युक्त कथनों में से कौन सा/से सही है /हैं ?

A  1 only
केवल 1

B  2 and 3 only
केवल 2 और 3

C  2 only
केवल 2

D  1, 2 and 3
1, 2 और 3
Solution : B

Explanation: 
Statement 1 is incorrect: It has been developed by Digantara, India’s first air and
space surveillance company. Digantara is a private company based in Bangalore. 
Statement 2 is correct: This surveillance system will provide global real-time earth
coverage by deploying a constellation of cost-efficient nanosatellites in Low Earth
Orbit (LEO). 
Statement 3 is correct: The surveillance system is India’s first in-orbit space debris
monitoring and tracking system. 
व्याख्या: 
कथन 1 गलत है : इसे भारत की पहली वायु और अं तरिक्ष निगरानी कंपनी दिगांतरा द्वारा विकसित
किया गया है ।दिगांतरा बैं गलोर में स्थित एक निजी कंपनी है ।
कथन 2 सही है : यह सर्विलांस सिस्टम निम्न भू-कक्षा (Low Earth Orbit- LEO) में लागत-कुशल
लघु उपग्रहों के समूह को स्थापित करके वै श्विक रियल टाइम अर्थ कवरे ज (global real-time earth
coverage) प्रदान करे गी।
कथन 3 सही है : यह सर्विलांस सिस्टम भारत का प्रथम इन-ऑर्बिट स्पे स डे बर् ीज मॉनिटरिं ग एं ड
ट् रैकिंग सिस्टम है ।
 19 s

-0.67/2.0 

43
Q. Consider the following statements: 

1. If two countries lie along the same latitude, then the sun rises at the same time
irrespective of their longitudes. 
2. Unlike latitudes, there is no specific central longitude. 

Which of the statements given above is/are correct?


Q. निम्नलिखित कथनों पर विचार कीजिए:

1. यदि दो दे श एक ही अक्षां श पर स्थित होते हैं ,तो उनके दे शांतरों के अलग होने के बावजूद सूर्य के
उगने का समय एक ही होता है ।
2. अक्षां शों के विपरीत, कोई विशिष्ट प्रधान दे शांतर नहीं होता है ।

उपर्युक्त कथनों में से कौन सा/से सही है /हैं ?


 
A  1 only
केवल 1

B  2 only
केवल 2

C  Both 1 and 2
1 और 2 दोनों
D  Neither 1 nor 2
न तो 1, न ही 2
Solution : B

Explanation: 
Statement 1 is incorrect: The time of the sunrise and sunset depends on a number
of variables which include longitude, latitude, altitude and location in relation to a
specific time zone. It does not depend on a single variable. Locations on similar
latitudes have similar day lengths. 
Statement 2 is correct: Unlike latitudes, there is no specific central longitude. As a
reference, the meridian line through Greenwich, England is considered as the Prime
Meridian at 0° longitude and it divides the Earth into Eastern and Western
hemispheres. 
व्याख्या:
कथन 1 गलत है : सूर्योदय और सूर्यास्त का समय कई कारकों पर निर्भर करता है जिसमें किसी
विशिष्ट समय क्षे तर् का दे शांतर, अक्षां श, ऊंचाई और अवस्थिति शामिल हैं ।यह किसी एक कारक पर
निर्भर नहीं करता है । समान अक्षां शों के स्थानों में दिन की लं बाई समान होती है ।
कथन 2 सही है : अक्षां शों के विपरीत, कोई विशिष्ट प्रधान दे शांतर नहीं होता है ।उदाहरण के रूप में ,
इं ग्लैं ड के ग्रीनविच से गु जरने वाली मध्यान्ह रे खा को 0° दे शांतर पर प्रधान मध्यान्ह माना जाता है
और यह पृ थ्वी को पूर्वी और पश्चिमी गोलार्ध में विभाजित करती है ।
 16 s

44
Q. With reference to the origin and evolution of the universe, which of the following
statements is/are correct?

1. Edwin Hubble provided evidence that the universe is expanding. 


2. Hoyle’s concept of steady-state further clarified that the universe is expanding at a
steady and consistent rate. 

Select the correct answer using the codes given below: 


Q. ब्रह्मांड की उत्पत्ति और विकास के सं दर्भ में , निम्नलिखित कथनों में से कौन सा/से सही है /हैं ?

1. एडविन हबल ने सिद्ध किया कि ब्रह्मांड का विस्तार हो रहा है ।


2. आगे चलकर हॉयल की स्थिर-अवस्था अवधारणा (concept of steady-state) से स्पष्ट हुआ कि
ब्रह्मांड का विस्तार स्थिर और क् रमिक दर से हो रहा है ।
निम्नलिखित कू टों का उपयोग करके सही उत्तर का चयन कीजिए:

A  1 only
केवल 1

B  2 only
केवल 2

C  Both 1 and 2
1 और 2 दोनों

D  Neither 1 nor 2
न तो 1, न ही 2
Solution : A

Explanation: 
Statement 1 is correct: The origin of the Big Bang theory can be credited to Edwin
Hubble. Hubble provided evidence that the universe is continuously expanding. He
also discovered that a galaxy's velocity is proportional to its distance. Galaxies that
are twice as far from us move twice as fast. 
Statement 2 is incorrect: In contrast to the concept given by Edwin Hubble, Hoyle’s
concept of steady-state considers that the universe to be roughly the same at any
point of time. However, with more research and evidence, the scientific community at
present favours the argument of expanding universe. 
व्याख्या:
कथन 1 सही है : बिग बैं ग सिद्धांत की उत्पत्ति का श्रेय एडविन हबल को दिया जा सकता है ।हबल ने
सिद्ध किया कि ब्रह्मांड का लगातार विस्तार हो रहा है । उन्होंने यह भी पता लगाया कि एक
आकाशगं गा का वे ग उसकी दरू ी के समानु पाती होता है ।आकाशगं गाएँ हमसे जितनी ही अत्यधिक दरू
हैं उतनी ही ते जी से हमसे दरू होती जा रही हैं । 
कथन 2 गलत है : एडविन हबल द्वारा दी गई अवधारणा के विपरीत, हॉयल की स्थिर-अवस्था
अवधारणा के अनु सार ब्रह्मांड हर समय लगभग एक ही अवस्था में होती है ।हालां कि, अधिक शोध
और साक्ष्य के साथ, वर्तमान में वै ज्ञानिक समु दाय ब्रह्मांड के विस्तार के तर्क का समर्थन करते हैं ।
 42 s

45
Q. Which of the following features is found to be common on the mouths of large
rivers such as Ganga, Congo, Hudson, Mississippi?
Q. गं गा, कां गो, हडसन, मिसिसिपी जै सी बड़ी नदियों के मु हाने पर निम्नलिखित में से कौन सी
विशे षता पाई जाती है ?

Seamounts

समु दर् ी टीला
(Seamounts)

B  Oceanic trenches
महासागरीय खाइयाँ

C  Submarine canyons
अन्तःसमु दर् ी घाटियाँ

D  Guyots
निमग्न द्वीप (Guyots)
Solution : C

Explanation: 
Option (c) is correct: The submarine canyon is a steep-sided valley cut into the
seabed of the continental slope, sometimes extending well onto the continental shelf,
having nearly vertical walls. They occur around all the coasts of the world. Many
canyons are associated with the mouths of large rivers such as Ganga, Congo,
Hudson, Mississippi.
Option (a), (b) and (d) are incorrect: Seamounts, oceanic trenches and guyots are
the features of ocean bottom relief and hence are not a result of the action of rivers.

 Seamounts are underwater mountains that rise hundreds or thousands of feet


above the seafloor. They are generally extinct volcanoes that created piles of
lava when active. 
 Guyots are the flat-topped seamounts.
 Oceanic trenches are very deep topographic depressions of relatively narrow
width. These oceanographic features are the deepest parts of the ocean floor. 

व्याख्या:
विकल्प (c) सही है : अन्तःसमु दर् ी घाटियाँ महाद्वीपीय ढलान वाली घाटियाँ होती हैं ,जो महाद्वीपीय
ढलान के किनारे पर स्थित होती हैं । कभी-कभी यह महाद्वीपीय जल सीमा पर अच्छी तरह से
विस्तारित होती है और इसकी दीवारें लगभग ऊर्ध्वाधर होती हैं ।ये विश्व के सभी तटों के आसपास
होती हैं ।कई घाटियाँ गं गा, कां गो, हडसन, मिसिसिपी जै सी बड़ी नदियों के मु हाने से सं बंधित हैं ।
विकल्प (a), (b) और (d) गलत हैं : समु दर् ी टीले , महासागरीय खाइयाँ और निमग्न द्वीप (Guyots)
महासागरीय नितल  उच्चावच (ocean bottom relief) की विशे षताएँ हैं और इस प्रकार ये नदी
गतिविधियों के परिणाम नहीं हैं ।

 समुद ्री टीला (seamounts) पानी के अं दर स्थित पहाड़ हैं जो समु दर् तल से सै कड़ों या
हजारों फीट ऊपर उठे होते हैं ।ये आमतौर पर निष्क्रिय ज्वालामु खी हैं जिनका निर्माण
इनके सक्रिय होने के दौरान लावा के ढे र पर हुआ है ।
 निमग्न द्वीप(Guyots) सपाट शीर्ष वाले समु दर् ी टीले (seamounts) होते हैं ।
 महासागरीय खाइयां अपे क्षाकृत सं कीर्ण चौड़ाई की अत्यधिक गहरी स्थलाकृतिक
खाइयां होती हैं ।ये महासागरीय विशे षताएँ महासागरीय तल के सबसे गहरे भाग हैं ।

 57 s

-0.67/2.0 

46
Q. With reference to the Gold mines of the world, which of the following statements
is/are correct?

1. Grasberg mine is located in Indonesia. 


2. Muruntau mine is located in Australia. 
3. Boddington mine is located in Uzbekistan. 

Q. Select the correct answer using the codes given below: 


दुनिया में सोने की खानों के सं दर्भ में , निम्नलिखित कथनों में से कौन सा/से सही है /हैं ?

1. ग्रासबर्ग खदान इं डोने शिया में स्थित है ।


2. मु रुन्तौ ऑस्ट् रेलिया में स्थित है ।
3. बोडिं गटन खदान उज्बे किस्तान में स्थित है ।

निम्नलिखित कू टों  का उपयोग करके सही उत्तर का चयन कीजिए:

A  1 only
केवल 1

B  2 only
केवल 2

C  2 and 3 only
केवल 2 और 3

D  1, 2 and 3
1, 2 और 3
Solution : A
Explanation: 
Statement 1 is correct: Grasberg mine is located in Indonesia. It has the world's
single largest known gold reserve and the second largest copper reserves. 
Statement 2 is incorrect: Muruntau mine is located in Uzbekistan. 
Statement 3 is incorrect: Boddington mine is located in Australia. 
व्याख्या:
कथन 1 सही है : ग्रासबर्ग खदान इं डोने शिया में स्थित है । यह दुनिया का एकमात्र सबसे बड़ा ज्ञात
ू रा सबसे बड़ा तांबे का भं डार है ।
सोने का भं डार और दस
कथन 2 गलत है : मु रुन्तौ खदान उज्बे किस्तान में स्थित है ।
कथन 3 गलत है : बोडिं गटन खदान ऑस्ट् रेलिया में स्थित है ।
 39 s

2.0/2.0 

47
Q. Consider the following pairs:
 
  Shifting Cultivation Name Region
1. Jhuming North East India
2. Milpa Indonesia
3. Ladang Central America
4. Caingin Philippines
5. Dahiyaar Bundelkhand, India 
6. Kumari            Western Ghats, India

Which of the above given pairs are incorrectly matched?

Q. निम्नलिखित यु ग्मों पर विचार कीजिए:


 
  स्थानांतरणीय कृषि का नाम क्षे तर्
1. झम ू उत्तर पूर्व भारत
2. मिल्पा इं डोने शिया
3. लदां ग मध्य अमे रिका
4. कैंगिन फिलीपींस
5. दहियार बु ं देलखं ड, भारत
6. कुमारी पश्चिमी घाट, भारत
उपर्युक्त यु ग्मों में से किनका गलत मिलान किया गया है ?

A  1, 2 and 4 only


केवल 1, 2 और 4

B  2 and 3 only
केवल 2 और 3 

C  5 and 6 only
केवल 5 और 6 

D  3, 4, 5 and 6 only


केवल 3, 4, 5 और 6
Solution : B

Explanation:

Shifting cultivation is widely practised by many tribes in the tropics, especially in


Africa, south and central America and south-east Asia. The vegetation is usually
cleared by fire, and the ashes adds to the fertility of the soil. Shifting cultivation is
thus, also called slash and burn agriculture. It is prevalent in tropical regions with
different names.

Pair 1 is correctly matched: In North-East India it is known as Jhuming Cultivation.

Pair 2 is incorrectly matched: In Central America and Mexico it is known as Milpa.

Pair 3 is incorrectly matched: In Indonesia and Malaysia it is known as Ladang..

Pair 4 is correctly matched: In the Philippines, it is known as Caingin

Pair 5 is correctly matched: In Bundelkhand, Central India it is known as Dahiyaar


and Vevar Cultivation.

Pair 6 is correctly matched: In Western Ghats (in Kerala), India it is known as


Kumari

व्याख्या:

स्थानां तरणीय कृषि व्यापक रूप से उष्णकटिबं धीय क्षे तर् ों, विशे ष रूप से अफ् रीका, दक्षिण और
मध्य अमे रिका तथा दक्षिण-पूर्व एशिया में कई जनजातियों द्वारा की जाती है । स्थानांतरणीय
कृषि के लिए भूमि पर मौजूद वनस्पति को आमतौर पर आग द्वारा साफ किया जाता है और इस
् करती है । इस प्रकार, स्थानांतरणीय कृषि को
प्रकार निर्मित राख मिट् टी की उर्वरता में वृ दधि
कर्तन दहन कृषि (slash and burn agriculture) भी कहते हैं । यह उष्णकटिबं धीय क्षे तर् ों में
विभिन्न नामों से प्रचलित है ।
ू कृषि के नाम से जाना जाता है ।
यु ग्म 1 सुमेलित है : उत्तर-पूर्व भारत में इसे झम

यु ग्म 2 सुमेलित नहीं है : मध्य अमे रिका और मै क्सिको में इसे मिल्पा के रूप में जाना जाता है ।

यु ग्म 3 सुमेलित नहीं है : इं डोने शिया और मले शिया में इसे लदां ग के नाम से जाना जाता है ।

यु ग्म 4 सुमेलित है : फिलीपींस में , इसे कैंगिन के रूप में जाना जाता है ।

यु ग्म 5 सुमेलित है : बुं देलखं ड, मध्य भारत में इसे दहियार और वे वर कृषि के नाम से जाना
जाता है ।

यु ग्म 6 सुमेलित है : भारत के पश्चिमी घाट (केरल) में इसे कुमारी के नाम से जाना जाता है ।
 
 2 m 44 s

2.0/2.0 

48
Q. With reference to Madden-Julian Oscillation (MJO), which of the following
statements is/are correct?

1. MJO is an eastward-moving "pulse" of cloud and rainfall near the tropic of cancer.
2. Effects of MJO are most evident over the Indian Ocean and western equatorial the
Pacific Ocean.
3. MJO influences the El Nino Southern Oscillation (ENSO) cycle.

Select the correct answer using the codes given below:

Q. मै डेन-जूलियन ऑसीले शन(Madden-julian Oscillation-MJO) के सं दर्भ में , निम्नलिखित


कथनों में से कौन सा/से कथन सही है /हैं ?

1. MJO कर्क रे खा के नज़दीक पूर्व की ओर गतिशील बादलों और वर्षा का ‘कंपन(pulse)’ है ।


2. MJO के प्रभाव हिं द महासागर और प्रशांत महासागर के पश्चिमी भूमध्य रे खा पर सर्वाधिक  स्पष्ट
3. MJO अल नीनो दक्षिणी दोलन (ENSO) चक् र को प्रभावित करता है ।

निम्नलिखित कू टों का उपयोग करके सही उत्तर का चयन कीजिए:

A  1 and 2 only
केवल 1 और 2

B  2 and 3 only
केवल 2 और 3

3 only
केवल 3

D  1, 2 and 3
1, 2 और 3 
Solution : B

Explanation:

Statement 1 is incorrect: The Madden-Julian Oscillation (MJO can be characterized


as an eastward-moving "pulse" of cloud and rainfall near the equator that typically
recurs every 30 to 60 days.

Statement 2 is correct: Effects of MJO are most evident over the Indian Ocean and
western equatorial the Pacific Ocean. With the movement of the pulse an area of
enhanced tropical rainfall is first apparent over the western Indian Ocean, which
spreads eastward into the warm waters of the tropical Pacific Ocean.

Statement 3 is correct: There is evidence that the MJO influences the El Nino


Southern Oscillation (ENSO) cycle. Though,  it does not cause El Nino or La Nina,
but it can contribute to the speed of development and intensity of El Nino and La
Nina.

व्याख्या :

कथन 1 गलत है : मै डेन-जूलियन ऑसिले शन (MJO) की विशे षता है   भूमध्य रे खा के


निकट पूर्व की ओर गतिशील बादलों और वर्षा का ‘कं पन(pulse)’ जिसकी पु नरावृ त्ति आमतौर
पर हर 30 से 60 दिनों में   होती है ।

कथन 2 सही है : MJO  के प्रभाव हिं द महासागर और प्रशांत महासागर के पश्चिमी भूमध्य
रे खा पर सर्वाधिक स्पष्ट होते हैं । कंपन की गति के साथ, पश्चिमी हिन्द  महासागर पर सं वर्धित
उष्णकटिबं धीय वर्षा का क्षे तर् स्पष्ट है , जो उष्णकटिबं धीय प्रशांत महासागर के गर्म पानी में
पूर्व की ओर फैलता है ।

कथन 3 सही है : इस बात के प्रमाण हैं कि MJO अल नीनो दक्षिणी दोलन (ENSO) चक् र को
प्रभावित करता है । हालां कि, यह एल नीनो या ला नीना का कारण नहीं बनता है , ले किन यह
अल नीनो और ला नीना के विकास और तीव्रता की गति में योगदान कर सकता है ।
 0 s

2.0/2.0 

49
Q. Consider the following pairs regarding types of mass movements:
 
S. No Mass movement Description
1. Slump Imperceptibly slow, steady, downward movement of slope forming soil or rock.
2. Lahars It is a mudflow that flows down a composite volcano.
3. Mudslide The saturated flow of soil and debris on hillsides during the time of high precipitatio

Which of the above pairs is/are correctly matched?

Q. वृ हत सं चलन के सं दर्भ में निम्नलिखित यु ग्मों पर विचार कीजिए:


 
क् रम संख्या संचलन विवरण
1. स्लं प (Slump) स्पष्ट रूप से धीमी, स्थिर, नीचे की ओर गतिशील, मिट् टी या चट् टान से निर्मि
2. लाहर (Lahars) यह कीचड़ का प्रवाह है जो एक समग्र ज्वालामु खी के नीचे बहता है ।
3. मडस्लाइड (Mudslide) अत्यधिक वर्षा के दौरान पहाड़ियों पर मिट् टी और मलबे का सं तृप्त प्रवाह।

उपर्युक्त यु ग्मों में से कौन सा/से सु मेलित है / हैं ?

A  1 and 2 only
केवल 1 और 2

B  2 and 3 only
केवल 2 और 3

C  3 only
केवल 3

D  2 only
केवल 2
Solution : B

Explanation:

Pair 1 is incorrectly matched: Slump is a movement of large blocks of soil along a


curved surface of an undercut slope. Creep is an imperceptibly slow, steady,
downward movement of slope forming soil or rock.

Pair 2 is correctly matched: A lahar is mudflow that flows down a composite


volcano when it erupts, carrying melted ice, volcanic ash and other debris.

Pair 3 is correctly matched: Mudslide is a saturated flow of soil and debris on hill


slopes during the time of high precipitation. These typically occur where cliffs are
made up of boulder clay.
व्याख्या:

यु ग्म 1 सुमेलित नहीं  है : स्लं प मिट् टी के बड़े ब्लॉकों का एक ढलान की घु मावदार सतह पर
सं चलन है । क् रीप मिट् टी या चट् टान का एक धीमा, स्थिर, नीचे की ओर हो रहा सं चलन है ।

यु ग्म 2 सुमेलित है : लाहर कीचड़ का प्रवाह है जिसमें एक समग्र ज्वालामु खी के फटने के बाद
पिघली हुई बर्फ , ज्वालामु खी की राख और अन्य मलबे का मिश्रण नीचे की ओर बहता है ।

यु ग्म 3 सुमेलित है : अत्यधिक वर्षा के समय पहाड़ी ढलानों पर मिट् टी और मलबे का सं तृप्त
प्रवाह मडस्लाइड कहलाता है । ये आम तौर पर वहाँ होते हैं जहाँ चट् टानें गोलाश्म मृ त्तिका
(Boulder Clay) से बनी होती हैं ।
 1 m 7 s

2.0/2.0 

50
Q. With reference to the ‘Accelerate Vigyan’ scheme, consider the following
statements:

1. It is launched by the Council of Scientific and Industrial Research (CSIR).


2. It acts as a single platform for research internships and workshops across the
country.

Which of the statements given above is/are correct?

Q. ‘एक्सीले रेट विज्ञान’ (Accelerate Vigyan) योजना के सं दर्भ में निम्नलिखित कथनों पर विचार
कीजिए:

1. यह वै ज्ञानिक और औद्योगिक अनु संधान परिषद (Council of Scientific and Industrial


Research-CSIR) द्वारा प्रारं भ किया गया है ।
2. यह दे श भर में अनु संधान इं टर्नशिप और कार्यशालाओं हे तु एक एकल मं च के रूप में कार्य करता है ।

उपर्युक्त कथनों में से कौन सा/से सही है /हैं ?

A  1 only
केवल 1

B  2 only
केवल 2

C  Both 1 and 2
1 और 2 दोनों

Neither 1 nor 2
न तो 1, न ही 2
Solution : B

Explanation:

Statement 1 is incorrect: ‘Accelerate Vigyan’ has been launched by the Science and


Engineering Research Board (SERB), a statutory body under the Department of
Science and Technology, Government of India.

Statement 2 is correct: It provides a single platform for research internships, capacity


building programs and workshops across the country.

व्याख्या:

कथन 1 गलत है : ‘एक्सीले रेट विज्ञान’ (Accelerate Vigyan) योजना विज्ञान और


इंजीनियरिं ग अनुसंधान बोर्ड (Science and Engineering Research Board-SERB) द्वारा
प्रारं भ की गयी है , जो विज्ञान और प्रौद्योगिकी विभाग, भारत सरकार के तहत एक वै धानिक
निकाय है ।

कथन 2 सही है : यह दे श भर में अनु संधान इं टर्नशिप, क्षमता निर्माण कार्यक् रमों और
कार्यशालाओं के लिए एक एकल मं च प्रदान करती है ।
 
 36 s

2.0/2.0 

51
Q. Consider the following statements: 

1. A population pyramid is used to depict the age-sex structure of the population. 


2. The bell-shaped population pyramid represents a constant population. 
3. Japan has a bell-shaped population pyramid. 

Which of the statements given above is/are correct?


Q. निम्नलिखित कथनों पर विचार कीजिए:

1. जनसं ख्या पिरामिड का उपयोग जनसं ख्या की आयु -लिं ग सं रचना को दर्शाने के लिए किया जाता है ।
2. घं टे के आकार का जनसं ख्या पिरामिड एक स्थिर जनसं ख्या को दर्शाता है ।
3. जापान में एक घं टे के आकार वाली जनसं ख्या पिरामिड है ।

उपर्युक्त कथनों में से कौन सा/से सही है /हैं ?



1 only
केवल 1

B  1 and 2 only
केवल 1 और 2

C  2 and 3 only
केवल 2 और 3

D  1, 2 and 3
1, 2 और 3
Solution : B

Explanation: 
Statement 1 is correct: A population pyramid is used to depict the age-sex
structure of the population.
Statement 2 is correct: The shape of the population pyramid reflects the
characteristics of the population. The triangular-shaped pyramid represents an
expanding population. The bell-shaped pyramid represents a constant
population. A narrow base and tapered at top pyramid represent the declining
population. 
Statement 3 is incorrect: Japan has a narrow base and tapered at the top pyramid
which represents the declining population. Australia has a bell-shaped population
pyramid. 

Figure: Population pyramid of


Japan
Figure: Population pyramid of
Australia.
व्याख्या:
कथन 1 सही है : जनसं ख्या पिरामिड का उपयोग जनसं ख्या की आयु -लिं ग सं रचना को दर्शाने के लिए
किया जाता है ।
कथन 2 सही है : जनसं ख्या पिरामिड का आकार जनसं ख्या की विशे षताओं को दर्शाता है ।त्रिकोणीय
आकार का पिरामिड जनसं ख्या के विस्तार को दर्शाता है ।घंटे के आकार का पिरामिड एक स्थिर
जनसंख्या को दर्शाता है ।पिरामिड का सं कीर्ण आधार और शं कु के आकार वाला शीर्ष,घटती जनसं ख्या
को दर्शाता है ।
कथन 3 गलत है : जापान में पिरामिड का सं कीर्ण आधार और शं कु के आकार वाला शीर्ष जै सी सं रचना
दिखती है  जो घटती जनसंख्या को दर्शाता है ।ऑस्ट् रेलिया में घं टे के आकार वाला पिरामिड दिखाई
दे ता है ।
चित्र: जापान का जनसंख्या पिरामिड।

चित्र: ऑस्ट्रेलिया का जनसंख्या


पिरामिड।
 1 m 7 s

-0.67/2.0 

52
Q. Consider the following statements with reference to Polar Vortex: 
1. They are observed only in winter. 
2. They are confined to the troposphere only. 
3. A strong polar vortex favours a strong jet stream.

Which of the statements given above is/are correct?


Q. ध्रुवीय भं वर (Polar Vortex) के सं दर्भ में निम्नलिखित कथनों पर विचार कीजिए:

1. ये केवल सर्दियों में ही दिखाई दे ते हैं ।


2. ये केवल क्षोभमं डल में ही होते हैं ।
3. एक शक्तिशाली ध्रुवीय भं वर एक शक्तिशाली जे ट स्ट् रीम के लिए अनु कूल होता है ।

उपर्युक्त कथनों में से कौन सा/से सही है /हैं ?

A  1 only
केवल 1

B  1 and 3 only
केवल 1 और 3

C  3 only
केवल 3

D  2 and 3 only
केवल 2 और 3
Solution : C

Explanation: 
The polar vortex is a large area of low pressure and cold air surrounding both of the
Earth’s poles. The term "vortex" refers to the counterclockwise flow of air that helps
keep the colder air near the Poles. 
Statement 1 is incorrect: Polar vortex always exists near the poles. It is more
prominent in winter but becomes weaker in summer. 
Statement 2 is incorrect: The polar vortex is not just confined to the Troposphere
but extends beyond that to the stratosphere. Sometimes its reach is even upto the
mesosphere. 
Statement 3 is correct: A strong polar vortex favours a strong jet stream.
Conversely, when the polar vortex weakens, the jet stream also tends to weaken and
become distorted. Jet streams follow the boundaries between hot and cold air. Since
a strong polar vortex increases the temperature difference between hot and cold air,
the jet streams are strong. 
व्याख्या:
ध्रुवीय भं वर पृ थ्वी के दोनों ध्रुवों के आसपास कम दबाव और ठं डी हवा का एक बड़ा क्षे तर् है ।शब्द
"भं वर" हवा के वामावर्त प्रवाह को सं दर्भित करता है जो ध्रुवों के पास ठं डी हवा को बनाए रखने में
मदद करता है ।
कथन 1 गलत है : ध्रुवीय भं वर हमे शा ध्रुवों के पास मौजूद होता है । सर्दियों में इसकी प्रधानता
होती है , ले किन गर्मियों में यह कमजोर हो जाता है ।
कथन 2 गलत है : ध्रुवीय भं वर केवल क्षोभमं डल तक ही सीमित नहीं होता है , बल्कि समताप मं डल
तक विस्तृ त होता है ।कभी-कभी इसका विस्तार मध्यमं डल (Mesosphere) तक भी होता है ।
कथन 3 सही है : एक शक्तिशाली ध्रुवीय भं वर एक शक्तिशाली जे ट स्ट् रीम के अनु कूल होता है ।
इसके विपरीत, जब ध्रुवीय भं वर कमजोर होता है , तो जे ट स्ट् रीम भी कमजोर हो जाती है और इसका
रूप बदल जाता है ।जे ट धाराएँ गर्म और ठं डी हवा के बीच में चलती है ।चूंकि एक शक्तिशाली ध्रुवीय
भं वर गर्म और ठं डी हवा के बीच के तापमान के अं तर को बढ़ाता है , अतः जे ट धाराएं मजबूत होती हैं ।
 1 m 1 s

-0.67/2.0 

53
Q. Consider the following statements:

1. The troposphere at any latitude is thicker in summer than in winter.


2. With an increase in altitude, the temperature starts increasing in the mesosphere.
3. Noctilucent clouds are found in the Stratosphere only.

Which of the statements given above is/are correct?

Q. निम्नलिखित कथनों पर विचार कीजिए:

1. किसी भी अक्षां श पर क्षोभमं डल सर्दियों की तु लना में गर्मियों में अधिक मोटा होता है ।
् के साथ, मध्यमं डल (mesosphere) में तापमान बढ़ने लगता है ।
2. ऊंचाई में वृ दधि
3. निशादीप्त (Noctilucent)  बादल केवल समताप मं डल में पाए जाते हैं ।

उपर्युक्त कथनों में से कौन सा/से सही है /हैं ?

A  1 only
केवल 1

B  3 only
केवल 3

C  1 and 2 only
केवल 1 और 2
D  1, 2 and 3
1, 2 और 3
Solution : A

Explanation:

Statement 1 is correct: The Troposphere is the lowermost layer of the atmosphere.


Its average height is 13 Kilometers. The troposphere at any latitude is thicker in
summer than in winter due to higher insolation.

Statement 2 is incorrect: The Mesosphere lies above the stratosphere which extends


up to 80 Kilometers. In this layer, the temperature starts decreasing with an increase
in altitude and reaches up to -100 degree celsius at 80 Kilometers.

Statement 3 is incorrect: Noctilucent clouds are extremely rare very high clouds seen
in the night sky, usually on clear, summer nights. Noctilucent clouds form in the
highest reaches of the atmosphere – the mesosphere – as much as 50 miles (80 km)
above the Earth’s surface.

व्याख्या:

कथन 1 सही है : क्षोभमंडल वायु मं डल की सबसे निचली परत है । इसकी औसत ऊँचाई 13


किलोमीटर होती है । किसी भी अक्षां श पर क्षोभमं डल अधिक सूर्यताप (insolation) होने के
कारण सर्दियों की तु लना में गर्मी में अधिक मोटा होता है , अर्थात इसका क्षे तर् बढ़ जाता है ।

कथन 2 गलत है : मध्यमंडल  समताप मं डल के ऊपर स्थित है जो 80 किलोमीटर तक फैला


् के साथ तापमान घटने  लगता है और 80 किलोमीटर तक
हुआ है । इस परत में , ऊंचाई में वृ दधि
-100 डिग्री से ल्सियस तक पहुँच जाता है ।

कथन 3 गलत है : रात्रिकालीन आकाश में निशादीप्त (Noctilucent)  बादल बहुत दुर्लभ हैं ,
जो आमतौर पर बहुत ऊँचाई पर होते हैं एवं साफ़ , गर्मियों की रातों में दिखाई दे ते हैं ।
निशादीप्त (Noctilucent)  बादल वायु मं डल के उच्चतम खं ड अर्थात मध्यमं डल - पृ थ्वी की
सतह से 50 मील (80 किमी) ऊपर में बनते हैं ।
 0 s

54
Q. Consider the following statements: 

1. A double-landlocked country means that the given country has only landlocked
neighbours. 
2. Uzbekistan is the only double landlocked country in the world. 
Which of the statements given above is/are correct?
Q. निम्नलिखित कथनों पर विचार कीजिए:

1. दोहरे स्थल-अवरुद्ध (double-landlocked) दे श का अर्थ है कि उस दे श के पड़ोसी केवल स्थल-


अवरुद्ध दे श ही हैं ।
2. उज्बे किस्तान दुनिया का एकमात्र दोहरा स्थल-अवरुद्ध दे श है ।

उपर्युक्त कथनों में से कौन सा/से सही है /हैं ?

A  1 only
केवल 1

B  2 only
केवल 2

C  Both 1 and 2
1 और 2 दोनों

D  Neither 1 nor 2
न तो 1, न ही 2
Solution : A

Explanation: 
Statement 1 is correct: A double-landlocked country means that the given country
has only landlocked neighbours. 
Statement 2 is incorrect: There are only two double-landlocked countries in the
world, Liechtenstein in Europe and Uzbekistan in Asia. 
व्याख्या: 
कथन 1 सही है : दोहरे स्थल-अवरुद्ध दे श का अर्थ है कि उस दे श के पड़ोसी केवल स्थल-
अवरुद्ध (landlocked) दे श ही हैं ।
कथन 2 गलत है : दुनिया में केवल दो दोहरे स्थल-अवरुद्ध (double-landlocked) दे श हैं ,यूरोप
में  लिकटें स्टीन (Liechtenstein) और एशिया में  उजबे किस्तान।
 46 s

2.0/2.0 

55
Q. Regarding International Date Line, consider the following statements:
1. It was defined at the 1884 Prime Meridian Conference.
2. As one travels from west of the line to its east, he/she gains a calendar day.
3. It passes through Siberia in the North and New Zealand in the South.

Which of the statements given above is/are correct?

Q. अं तर्राष्ट् रीय तिथि रे खा के सं दर्भ में निम्नलिखित कथनों पर विचार कीजिए:

1. इसे 1884 के प्राइम मे रिडियन सम्मे लन में परिभाषित किया गया था।
2. जब कोई इस रे खा के पश्चिम से पूर्व की ओर यात्रा करता है उसे एक अतिरिक्त कैलें डर दिवस
प्राप्त होता है ।
3. यह उत्तर में साइबे रिया और दक्षिण में न्यूजीलैं ड से गु जरती है ।

उपर्युक्त कथनों में से कौन सा/से सही है /हैं ?

A  1 and 2 only
केवल 1 और 2

B  2 only
केवल 2

C  1 and 3 only
केवल 1 और 3

D  1, 2 and 3
1, 2 और 3
Solution : B

Explanation:

International Date Line, also called Date Line, is an imaginary line extending


between the North Pole and the South Pole. It demarcates each calendar day from the
next.

Statement 1 is incorrect:  The line is one of convenience and it is not recognized as a


legal entity. It has no international status, and was not defined at the 1884 Prime
Meridian conference in Washington, D.C.

Statement 2 is correct: When you cross the International Date Line from west to
east, you subtract a day thus you gain a day. Suppose you started on 8th of February
from the West of the IDL, you arrive on the East of the IDL on 7th of February, thus
gaining a calendar day.

Statement 3 is incorrect: It corresponds along most of its length to the 180th


meridian of longitude but deviates eastward through the Bering Strait to avoid
dividing Siberia and then deviates westward to include the Aleutian Islands with
Alaska. South of the Equator, another eastward deviation allows certain island groups
to have the same day as New Zealand.

व्याख्या:

अंतर्राष्ट्रीय तिथि रे खा, जिसे  तिथि रे खा भी कहा जाता है , उत्तरी और दक्षिणी ध्रुव के मध्य
एक काल्पनिक रे खा है । यह प्रत्ये क कैलें डर दिवस का अगले दिन से निर्धारण करती है ।

कथन 1 गलत है : यह रे खा केवल सु विधा के लिए है और इसे कानूनी इकाई के रूप में मान्यता
प्राप्त नहीं है । इसकी कोई अं तर्राष्ट् रीय अवस्थिति नहीं है और इसे वाशिं गटन, डी.सी. में
1884 के प्राइम मे रिडियन सम्मे लन में परिभाषित नहीं किया गया था।

कथन 2 सही है : जब आप अं तर्राष्ट् रीय तिथि रे खा को पश्चिम से पूर्व की ओर पार करते हैं , तो
आप एक दिन घटाते हैं , इस प्रकार आप एक अतिरिक्त दिन प्राप्त करते हैं । मान लीजिए कि
आपने 8 फरवरी को अं तर्राष्ट् रीय तिथि रे खा के पश्चिम से शु रुआत की है , तो आप 7 फरवरी
को अं तर्राष्ट् रीय तिथि रे खा के पूर्व में आएँ गे। इस प्रकार एक अतिरिक्त कैलें डर दिवस प्राप्त
होता है ।

कथन 3 गलत है : यह अपनी लं बाई के अधिकां श भाग तक दे शांतर के 180 वें मध्याह्न रे खा के
अनु रूप है , परं तु पूर्व में यह बे रिंग जल सन्धि से गु जरती है और साइबे रिया को विभाजित नहीं
करते हुए पश्चिम की ओर मु ड़ जाती है और अलास्का के साथ अले उतियन द्वीपों को शामिल
करती है । भूमध्य रे खा के दक्षिण में इसमें एक और पूर्ववर्ती विचलन होता है जिससे कुछ द्वीप
समूहों में भी न्यूजीलैं ड के समान दिवस सं भव होते हैं ।
 41 s

56
Q. Which of the following countries have a coastline with the Mediterranean Sea?

1. Burkina Faso
2. Morocco
3. Eritrea
4. Libya

Select the correct answer using the codes given below:

Q. निम्नलिखित दे शों में से किस की तटरे खा भूमध्य सागर के साथ लगती है ?

1. बु र्कि ना फासो
2. मोरक्को
3. इरिट्रिया
4. लीबिया

निम्नलिखित कू टों का उपयोग करके सही उत्तर चु निए:

A  1, 2 and 3 only


केवल 1, 2 और 3

B  2 and 4 only
केवल 2 और 4

C  3 and 4 only
केवल 3 और 4
D  1, 2 and 4 only
केवल 1, 2 और 4
Solution : B

Explanation:

21 countries have coastlines on the Mediterranean Sea. They are Albania, Algeria,
Bosnia and Herzegovina, Croatia, Cyprus, Egypt, France, Greece, Israel, Italy,
Lebanon, Libya, Malta, Monaco, Montenegro, Morocco, Slovenia, Spain, Syria,
Tunisia, and Turkey.
African countries bordering Mediterranean Sea are Morocco, Algeria, Tunisia,
Libya and Egypt.

Figure:
Mediterranean Sea

व्याख्या:

21 दे शों की तटरे खा भूमध्य सागर से लगती है । ये दे श हैं :अल्बानिया, अल्जीरिया, बोस्निया


और हर्ज़िगोविना, क् रोएशिया, साइप्रस, मिस्र, फ् रांस, ग्रीस, इजरायल, इटली, ले बनान,
लीबिया, माल्टा, मोनाको, मोंटे ने गर् ो, मोरक्को, स्लोवे निया, स्पे न, सीरिया, ट्यूनीशिया और
तु र्की।

भूमध्य सागर के साथ सीमा साझा करने वाले अफ् रीकी दे श मोरक्को, अल्जीरिया,
ट्यूनीशिया, लीबिया और मिस्र हैं ।
चित्र: भूमध्य सागर
 
 1 m 19 s

2.0/2.0 

57
Q. Consider the following pairs:
 
S. no Mining Region Mineral
1. Chuquicamata, Chile Copper
2. Greenbushes, Australia Lithium
3. Ruhr, Germany Coal

Which of the above given pairs is/are correctly matched?

Q. निम्नलिखित यु ग्मों पर विचार कीजिए:


 
क् रम सं ख्या खनन क्षे तर् खनिज
1. चिक्विकामाटा, चिली तांबा
2. ग्रीनबु श (Greenbushes), ऑस्ट् रेलिया लिथियम
3. रूर (Ruhr), जर्मनी कोयला

उपर्युक्त यु ग्मों में से कौन सा/से सु मेलित है /हैं ?

A  1 only
केवल 1

2 only
केवल 2

C  2 and 3 only
केवल 2 और 3

D  1, 2 and 3
1, 2 और 3
Solution : D

Explanation: 
Pair 1 is correctly matched: Chuquicamata is the largest open-pit copper mine in
the world. It is in Chile. 
Pair 2 is correctly matched: Greenbushes mine in Western Australia is the biggest
operating lithium mine in the world. 
Pair 3 is correctly matched: Ruhr region of Germany is known for coalfields. 
व्याख्या: 
यु ग्म 1 सुमेलित है : चिक्विकामाटा दुनिया की सबसे बड़ी खु ली गड्ढे वाली तांबे की खान है । यह
चिली में है ।
यु ग्म 2 सुमेलित है : पश्चिमी ऑस्ट् रेलिया की ग्रीनबु श खदान दुनिया की सबसे बड़ी लिथियम खदान
है जो अभी सं चालन में है ।
यु ग्म 3 सुमेलित है : जर्मनी का रुर क्षे तर् कोयला के लिए प्रसिद्ध है ।
 1 m 9 s

2.0/2.0 

58
Q. With reference to mining, which of the following statements are incorrect: 

Q. खनन के सं बंध में ,निम्नलिखित कथनों में से कौन सा गलत  है :

A  Opencast mining is supposedly the cheapest and easiest way of mining.


विवृ त खनन (Opencast mining) को खनन का सबसे सस्ता और सरल तरीका माना जाता है ।

B  Rat hole mining is practised in areas where the coal seam layer is very thin.
रै ट होल खनन उन क्षे तर् ों में किया जाता है जहाँ कोयले की परत बहुत पतली होती है ।

Shaft method of mining is used for deep-seated minerals.
खनन की कू पकी विधि का प्रयोग गहराई में स्थित खनिजों के लिए किया जाता है ।

Strip mining is mostly used to extract deep bedded deposits.



विपट् टी खनन (strip mining) का उपयोग ज्यादातर गहरे सं स्तरित निक्षे प को निकालने के लिए
किया जाता है ।
Solution : D

Explanation:

Option (a) is correct: Opencast mining is the cheapest and easiest method of mining
as it involves less capital investment.

Option (b) is correct: In places where the coal seam is extremely thin (like in
Meghalaya), Rat hole mining is practised, since no other method would be
economically viable. This is because the removal of rocks from the hilly terrain and
putting up pillars inside the mine to prevent collapse would be costlier.

Option (c) is correct: Shaft mining refers to the method of excavating a vertical or


near-vertical tunnel from the top down, where there is initially no access to the
bottom. Shaft mining is the process where miners dig straight down until they reach
their desired depth. Then the mine begins to branch out in all directions. Miners will
enter or exit a mine through a lift or elevator installed where the initial vertical tunnel
was originally.

Option (d) is incorrect: Strip mining is mostly used to extract shallow, "bedded"


deposits, where a mineral layer is covered by a layer of soft topsoil and weathered
rocks.

व्याख्या:

विकल्प (a) सही है : विवृ त खनन, खनन का सबसे सस्ता और आसान तरीका है ,क्योंकि इसमें
कम पूँजी निवे श की आवश्यकता होती है ।

विकल्प (b) सही है : उन स्थानों पर जहाँ कोयले की परत बहुत पतली (जै से मे घालय में ) होती
है , वहाँ रै ट होल खनन किया जाता है , क्योंकि वहाँ कोई अन्य विधि आर्थिक रूप से व्यवहार्य
नहीं होगी। ऐसा इसलिए है ,क्योंकि पहाड़ी इलाकों से चट् टानों को हटाना और खानों को ढहने
से रोकने के लिए इसके भीतर स्तं भ का निर्माण करना अत्यं त महं गा होता है ।

विकल्प (c) सही है : कू पकी खनन या कू पकी गर्तन एक ऊर्ध्वाधर या लगभग ऊर्ध्वाधर सु रंग
की शीर्ष से नीचे की ओर खु दाई की विधि को सं दर्भित करता है , जहाँ प्रारं भ में तल तक कोई
पहुंच नहीं होती है । कू पकी खनन एक ऐसी विधि है जिसमें खनिक वां छित गहराई तक पहुंचने
तक सीधे खु दाई करते हैं । इसके बाद जब खदान सभी दिशाओं में शाखित हो जाती है , तब
खनिक खदानों में लगी लिफ्ट या सीढ़ियों के माध्यम से खदान में प्रवे श या उससे बाहर निकल
जाते हैं ।

विकल्प (d) गलत है : विपट् टी खनन का उपयोग ज्यादातर उथले , "सं स्तरित" निक्षे प को
निकालने के लिए किया जाता है , जहाँ खनिज की परत मु लायम ऊपरी मिट् टी और अपक्षीण
चट् टानों की परत द्वारा ढँ की होती है ।
 56 s

-0.67/2.0 

59
Q. With reference to insolation, which of the following statements is/are correct?

1. Topographical variations play a major role in distribution of Insolation.


2. In the northern hemisphere south facing slopes are warmer, dryer than north facing
slopes.

Select the correct answer using the codes given below:

Q. सूर्यताप (Insolation) के सं दर्भ में निम्नलिखित में से कौन सा/से कथन सही है /हैं ?

1. स्थलाकृतिक भिन्नता सूर्यताप के वितरण में एक प्रमु ख भूमिका निभाती है ।


2. उत्तरी गोलार्ध में दक्षिण की ओर की ढलानें उत्तर की ढलानों की तु लना में गर्म और शु ष्क होती हैं ।

निम्नलिखित कू टों का उपयोग करके सही उत्तर चु निए:

A  1 only
केवल 1

B  2 only
केवल 2

C  Both 1 and 2
1 और 2 दोनों

D  Neither 1 nor 2
न तो 1, न ही 2
Solution : C

Explanation:

Statement 1 is correct: Topographical variations are major factors modifying the


distribution. Elevation, surface orientation creates strong local gradients of insolation.

Statement 2 is correct: In the northern hemisphere a south-facing slope(more open to


sunlight) will be generally warmer, this can be evident from thick vegetation covering
southern slopes of himalaya; than northern slopes which are cold, heavily glaciated.
व्याख्या:

कथन 1 सही है : स्थलाकृतिक भिन्नताएँ वितरण का निर्धारण करने वाले प्रमु ख कारक हैं ।
ऊंचाई एवं सतह अभिविन्यास सूर्यताप की सशक्त स्थानीय प्रवणता का निर्माण करते हैं ।

कथन 2 सही है : उत्तरी गोलार्ध में एक दक्षिण मु खी ढलान (जिसपर सूरज की रोशनी अधिक
पड़ती हो) आम तौर पर गर्म होगा। यह उत्तरी ढलान, जो ठं डे हैं और जहाँ व्यापक सं ख्या में
हिमनद हैं , की तु लना में हिमालय के दक्षिणी ढलानों पर पाई जाने वनस्पतियों के घने आवरण
से स्पष्ट हो सकता है ।
 
 34 s

60
Q. With reference to El Nino, which of the following statements are correct?

1. El Nino is a climate pattern that describes the unusual warming of surface waters in
the Central Equatorial Pacific Ocean. 
2. In an El Nino event, the productivity of the coastal ecosystem in the Eastern
Equatorial Pacific Ocean is negatively affected. 
3. Trade winds are weaker during El Nino-Southern Oscillation (ENSO). 

Select the correct answer using the codes given below: 


Q. एल नीनो के सं दर्भ में निम्नलिखित कथनों में से कौन सा/से सही है /हैं ?

1. एल नीनो एक जलवायविक प्रक्रिया है जो केंद्रीय भूमध्यरे खीय प्रशांत महासागर में सतह के जल
के असामान्य तापन का वर्णन करती है ।
2. एल नीनो की घटना में पूर्वी भूमध्यरे खीय प्रशांत महासागर में तटीय पारिस्थितिकी तं तर् की
उत्पादकता नकारात्मक रूप से प्रभावित होती है ।
3. एल नीनो-दक्षिणी दोलन (El Nino-Southern Oscillation-ENSO) के दौरान व्यापारिक पवनें
कमजोर होती हैं ।

निम्नलिखित कू टों का उपयोग करके सही उत्तर चु निए:

A  1 and 2 only
केवल 1 और 2

B  2 and 3 only
केवल 2 और 3

C  3 only
केवल 3

1, 2 and 3
1, 2 और 3
Solution : B

Explanation: 
Statement 1 is incorrect: El Nino is a climate pattern that describes the unusual
warming of surface water in the Eastern Equatorial Pacific Ocean. El Nino
Modoki is characterized by changes in sea-surface temperatures in the Central
Equatorial Pacific Ocean. 
Statement 2 is correct: The thick layer of warm water accumulated due to unusual
warming of surface water does not allow normal upwelling to occur. Without an
upwelling, the euphotic zone (layer closer to the surface that receives enough light
for photosynthesis to occur) can no longer support its normally productive coastal
ecosystem in eastern equatorial Pacific Ocean. 
Statement 3 is correct: Trade winds are weaker during El Nino-Southern Oscillation
(ENSO). These changes in wind speed and air pressure cause warm surface water
to move eastward along the Equator, from the western Pacific to the coast of
northern South America.
व्याख्या:
कथन 1 गलत है : एल नीनो एक जलवायविक प्रक्रिया है जिसमें  पूर्वी भूमध्यरे खीय प्रशांत
महासागर में सतही जल का असामान्य तापन होता है । केंद्रीय भूमध्यरे खीय प्रशांत महासागर में
समु दर् की सतह के तापमान में परिवर्तन एल नीनो मोदोकी (El Nino Modoki) की विशे षता है ।
कथन 2 सही है : सतह के जल के असामान्य रूप से गर्म होने के कारण गर्म पानी की मोटी परत जल
का सामान्य उत्प्लावन नहीं होने दे ती है । उत्प्लावन के बिना व्यं जना क्षे तर् (euphotic zone) (सतह
के करीब की परत जो प्रकाश सं श्ले षण के लिए पर्याप्त प्रकाश प्राप्त करता है ) पूर्वी भूमध्यरे खीय
प्रशांत महासागर में उत्पादक तटीय पारिस्थितिकी तं तर् का सहयोग नहीं कर पाता है ।
कथन 3 सही है : एल नीनो-दक्षिणी दोलन (El Nino-Southern Oscillation--ENSO) के दौरान
व्यापारिक पवनें कमजोर होती हैं । हवा की गति और दबाव में इन परिवर्तनों के कारण गर्म सतह का
पानी भूमध्य रे खा के साथ-साथ पश्चिमी प्रशांत क्षे तर् से उत्तरी दक्षिण अमे रिका के तट तक चला
जाता है ।
 1 m 12 s

2.0/2.0 

61
Q. With reference to the Population distribution in the world, which of the following
statements is/are correct?

1. The equatorial regions are highly populated regions of the world. 


2. The maritime climate favours higher population growth than continental interiors.
3. Desert regions have populations only near oasis.

Select the correct answer using the codes given below: 

Q. विश्व में जनसं ख्या वितरण के सं दर्भ में , निम्नलिखित कथनों में से कौन सा/से सही है /हैं ?

1. भूमध्यरे खीय क्षे तर् दुनिया के अत्यधिक आबादी वाले क्षे तर् हैं ।
2. समु दर् ी जलवायु महाद्वीपीय आं तरिकों की तु लना में अधिक जनसं ख्या वृ दधि
् के लिए उपयु क्त होती
है ।
3. रे गिस्तानी क्षे तर् ों में आबादी केवल मरूद्यान के पास निवास करती है ।

निम्नलिखित कू टों का उपयोग करके सही उत्तर चु निए:

A  1 only
केवल 1 

B  2 only
 केवल 2

C  1 and 3 only
केवल 1 और 3

D  1, 2 and 3
1, 2 और 3
Solution : B

Explanation:

Statement 1 is incorrect: Equatorial region is not a highly populated region of the


world because of its harsh climate. The tropical and temperate climate is suitable for
high population growth.

Statement 2 is correct: Maritime climate favours high population growth than


interiors because of Moderate climate.

Statement 3 is incorrect: Desert regions have  populations not just  near the oasis but
also where economic opportunities are available (Like Dubai).

व्याख्या:

कथन 1 गलत है : भूमध्यरे खीय क्षे तर् अपनी कठोर जलवायु के कारण दुनिया का अत्यधिक

आबादी वाला क्षे तर् नहीं है । उष्णकटिबं धीय और समशीतोष्ण जलवायु उच्च जनसं ख्या वृ दधि
के लिए उपयु क्त है ।

कथन 2 सही है : समु दर् ी जलवायु शीतल-मृ दु जलवायु की वजह से आं तरिक भाग की तु लना में
् के लिए उपयु क्त होती है ।
उच्च जनसं ख्या वृ दधि

कथन 3 गलत है : रे गिस्तानी क्षे तर् ों में आबादी न केवल मरूद्यान के आस-पास निवास करती है ,
बल्कि उन स्थानों में भी निवास करती है जहाँ आर्थिक अवसर उपलब्ध होते हैं , जै से दुबई।
 31 s

-0.67/2.0 

62
Q. With reference to the G4 Flu virus, which was recently in news, consider the
following statements:

1. It has been descended from the H1N1 virus.


2. It has the potential to grow and multiply humans airways.
3. It can bind to human-type receptors like SARS-Cov-2 virus.

Which of the statements given above is/are correct?

Q. हाल ही में चर्चा में रहे G4 फ्लू विषाणु के सं दर्भ में , निम्नलिखित कथनों पर विचार कीजिए:

1. यह H1N1 विषाणु का वं शज है ।
् करने की क्षमता है ।
2. इसमें मानव श्वसन मार्ग में विकसित होने और गु णक वृ दधि
3. यह सार्स-कोव-2 विषाणु (SARS-Cov-2) की तरह मानव-प्रकार के रिसे प्टर्स के साथ बं धित हो
सकता है ।

उपर्युक्त कथनों में से कौन सा/से सही है /हैं ?

A  1 only
केवल 1

B  2 and 3 only
केवल 2 और 3

C  1 and 3 only
केवल 1 और 3

D  1, 2 and 3
1, 2 और 3
Solution : D
Explanation:

Statement 1 is correct: G4 flu virus had descended from the H1N1 strain that was
responsible for the 2009 swine flu pandemic.

Statement 2 is correct: It has the potential to copy itself in human airway epithelial
cells.

Statement 3 is correct: It has the capability of binding to human-type receptors (like


the SARS-CoV-2 virus).

व्याख्या:

कथन 1 सही है : G4 फ्लू विषाणु H1N1 स्ट् रेन से उत्पन्न हुआ था जो 2009 के स्वाइन फ्लू
महामारी का कारण था।

् करने की
कथन 2 सही है : इसमें मानव श्वसन मार्ग उपकला कोशिकाओं में खु द में गु णक वृ दधि
क्षमता है ।

कथन 3 सही है : इसमें (SARS-CoV-2 विषाणु की तरह) मानव-प्रकार के रिसे प्टर्स के साथ
बं धित होने की क्षमता है ।
 5 s

63
Q. Which of the following seas are part of the Northern Sea Route?

1. North Sea
2. Barents Sea
3. Kara Sea
4. Baltic Sea

Select the correct answer using the codes given below:

Q. निम्नलिखित में से कौन से सागर उत्तरी समु दर् ी मार्ग के भाग हैं ?

1. उत्तरी सागर
2. बे रिंट सागर
3. कारा सागर
4. बाल्टिक सागर

निम्नलिखित कू टों का उपयोग करके सही उत्तर चु निए:

A  1 and 2 only
केवल 1 और 2
B  2 and 3 only
केवल 2 और 3

C  1 and 4 only
केवल 1 और 4

D  2, 3 and 4 only


केवल 2, 3 और 4
Solution : B

Explanation:

The Northern Sea Route (NSR) is a shipping lane between the Atlantic Ocean and the
Pacific Ocean along the Russian coast of Siberia and the Far East, crossing five Arctic
Seas: the Barents Sea, the Kara Sea, the Laptev Sea, the East Siberian Sea and the
Chukchi Sea.

व्याख्या:

उत्तरी समु दर् ी मार्ग (NSR) साइबे रिया और सु दरू पूर्व के रूसी तट से गु जरता हुआ अटलां टिक
महासागर और प्रशांत महासागर के बीच एक नौ-परिवहन मार्ग है । यह पाँच आर्क टिक सागरों
से होकर गु जरता है : बे रिंट सागर, कारा सागर, लै पटे व सागर, पूर्व साइबे रियाई सागर और
चु कची सागर।
 2 m 1 s

2.0/2.0 

64
Q. Consider the following statements regarding the solar system: 

1. Mercury takes the shortest time to complete one revolution around the sun. 
2. Neptune takes the longest time to complete one rotation on its own axis. 
3. Unlike other planets, Uranus orbits around the sun in a clockwise direction from east
to west. 

Which of the following statements is/are correct?


Q. सौर प्रणाली के बारे में निम्नलिखित कथनों पर विचार कीजिए:

1. बु ध को सूर्य के चारों ओर एक चक्कर पूरा करने में सबसे कम समय लगता है ।


2. ने प्च्यून को अपनी धु री पर एक चक्कर पूरा करने में सबसे अधिक समय लगता है ।
3. अन्य ग्रहों के विपरीत, यूरेनस पूर्व से पश्चिम की ओर दक्षिणावर्त दिशा में सूर्य की परिक् रमा करता
है ।

निम्नलिखित कथनों में से कौन सा/से सही है /हैं ?

A  1 only
केवल 1

B  2 only
केवल 2
C  1 and 3 only
केवल 1 और 3

D  1, 2 and 3
1, 2 और 3
Solution : C

Explanation: 
Statement 1 is correct: Mercury takes the shortest time to complete one revolution
around the sun. (88 days)
Statement 2 is incorrect: Venus takes the longest time to complete one rotation on
its own axis. (243 days)
Statement 3 is correct: Unlike other planets, Uranus orbits around the sun in a
clockwise direction from east to west. 
व्याख्या: 
कथन 1 सही है : बु ध को सूर्य के चारों ओर एक चक्कर पूरा करने में सबसे कम समय लगता है ।(88
दिन)
कथन 2 गलत है : शु क्र अपनी धु री पर एक चक्कर पूरा करने में सबसे अधिक समय ले ता है । (243
दिन)
कथन 3 सही है : अन्य ग्रहों के विपरीत, यूरेनस पूर्व से पश्चिम की ओर दक्षिणावर्त दिशा में सूर्य की
परिक् रमा करता है ।
 7 s

65
Q. Consider the following statements:

1. Seasonal migration of animal livestock from lowland pastures to mountainous


regions is known as Livestock ranching.
2. Commercial grazing of livestock over an extensive area which is associated with a
very large land requirement is known as Transhumance.

Which of the statements given above is/are correct?

Q. निम्नलिखित कथनों पर विचार कीजिए:

1. तराई क्षे तर् ों से पर्वतीय क्षे तर् ों में पशु धन के मौसमी पलायन को पशु धन पालन (Livestock
ranching) के रूप में जाना जाता है ।
2. एक विस्तृ त क्षे तर् पर पशु धन की व्यावसायिक चराई जो एक व्यापक भूमि की आवश्यकता से
सं बंधित होती है , को ऋतु प्रवास (Transhumance) के रूप में जाना जाता है ।

उपर्यक्त कथनों में से कौन सा/से सही है /हैं ?

A  1 only
केवल 1

B  2 only
केवल 2

C  Both 1 and 2
1 और 2 दोनों

Neither 1 nor 2

न तो 1 और न ही
2
Solution : D

Explanation:

Statement 1 is incorrect: Seasonal migration of animal livestock from lowland


pastures to mountainous regions is known as transhumance.

Statement 2 is incorrect: Commercial grazing of livestock over an extensive area


which is associated with a very large land requirement is known as Livestock
Ranching.

व्याख्या:

कथन 1 गलत है : तराई क्षे तर् ों से पर्वतीय क्षे तर् ों में पशु धन के मौसमी पलायन को ऋतु प्रवास
(Transhumance) के रूप में जाना जाता है ।

कथन 2 गलत है : एक विस्तृ त क्षे तर् पर पशु धन की व्यावसायिक चराई जो एक व्यापक भूमि की
आवश्यकता से सं बंधित होती है , को पशु धन पालन (Livestock ranching) के रूप में जाना
जाता है ।
 55 s

2.0/2.0 

66
Q. With reference to local winds, which of the following statements is/are correct?
1. Solano is a hot wind which blows in Southern California. 
2. Mistral is a dry, cold wind which blows from the Alps to France. 
3. Santa Ana is a hot, moist wind which blows from the Sahara to the Iberian
Peninsula. 

Select the correct answer using the codes given below: 


Q. स्थानीय हवाओं के सं दर्भ में , निम्नलिखित कथनों में से कौन सा/से सही है /हैं ?

1. सोलानो (Solano) एक गर्म हवा है जो दक्षिणी कैलिफ़ोर्निया में बहती है ।


2. मिस्ट् रल एक शु ष्क, ठं डी हवा है जो आल्प्स से फ् रांस तक बहती है ।
3. सांता एना (Santa Ana) एक गर्म, नम हवा है जो सहारा से इबे रियन प्रायद्वीप तक बहती है ।

निम्नलिखित कू टों का उपयोग करके सही उत्तर का चयन कीजिए:

A  1 only
केवल 1

B  2 only
केवल 2

C  3 only
केवल 3

D  1, 2 and 3
1, 2 और 3
Solution : B

Explanation: 
Statement 1 is incorrect: Solano is a hot, moist wind which blows from the Sahara
to the Iberian Peninsula. 
Statement 2 is correct: Mistral is a dry, cold wind which blows from the Alps to
France. 
Statement 3 is incorrect: Santa Ana is a hot wind which blows in Southern
California. 
व्याख्या:
कथन 1 गलत है : सोलानो एक गर्म, नम हवा है जो सहारा से इबे रियन प्रायद्वीप तक बहती है ।
कथन 2 सही है : मिस्ट् रल एक शु ष्क, ठं डी हवा है जो आल्प्स से फ् रांस तक बहती है ।
कथन 3 गलत है : सांता एना एक गर्म हवा है जो दक्षिणी कैलिफ़ोर्निया में बहती है ।
 4 s

67
Q. With reference to Footloose Industries, which of the following statements is/are
correct?

1. They are labour-intensive industries.


2. One of the important factors in their location is road connectivity.
3. They are generally non-polluting in nature.

Select the correct answer using the codes given below:

Q. फुटलूज़ उद्योग के सं दर्भ में , निम्नलिखित में से कौन सा/से कथन सही है /हैं ?

1. ये श्रम प्रधान उद्योग हैं ।


2. उनकी अवस्थिति का एक महत्वपूर्ण कारक सड़क सं पर्क है ।
ू णकारी होते हैं ।
3. ये सामान्यतः प्रकृति में गै र-प्रदष

निम्नलिखित कू टों का उपयोग करके सही उत्तर चु निए:

A  1 and 2 only
केवल 1 और 2

B  2 only
केवल 2

C  2 and 3 only
केवल 2 और 3

D  3 only
केवल 3
Solution : C

Explanation:

Footloose industries can be located in a wide variety of places. They are not
dependent on any specific raw material, weight losing or otherwise. They largely
depend on component parts which can be obtained anywhere.

Statement 1 is incorrect: Footloose industries are not labour-intensive industries


because these industries produce their products in small quantities and thus do not
require large labour forces.

Statement 2 is correct: One of the important factors in their location is accessibility


by road network.
Statement 3 is correct: Footloose industries are generally non-polluting industries as
there is no significant manufacturing process involved.

व्याख्या:

फुटलूज़ उद्योग व्यापक विविधता वाले स्थानों में स्थित होते हैं । ये किसी विशिष्ट कच्चे माल,
वज़न में समय के साथ कमी वाले पदार्थ अथवा अन्य, पर निर्भर नहीं रहते हैं । ये घटक भागों
पर निर्भर रहते हैं जो कहीं से भी प्राप्त किए जा सकते हैं ।

कथन 1 गलत है : फुटलूज़ उद्योग श्रम प्रधान उद्योग नहीं हैं , क्योंकि इन उद्योगों में उत्पादों
का उत्पादन कम मात्रा में होता है और इस प्रकार उन्हें अधिक श्रम बल की आवश्यकता
नहीं होती है ।

कथन 2 सही है : इनकी स्थापना के महत्वपूर्ण कारकों में से एक सड़कों के जाल द्वारा सु गम्यता
होती है ।

कथन 3 सही है : फुटलूज़ उद्योग सामान्यतः गै र-प्रदष ू णकारी उद्योग हैं , क्योंकि इसमें कोई
महत्वपूर्ण विनिर्माण प्रक्रिया शामिल नहीं होती है ।
 46 s

2.0/2.0 

68
Q. With reference to Air masses, which of the following statements are correct?

1. They retain the characteristics they acquire from the source region.
2. When continental air mass moves over the ocean, it helps to increase evaporation.
3. When air mass moves over a colder surface, it leads to precipitation. 

Select the correct answer using the codes given below:

Q. वायु राशियों के सं दर्भ में , निम्नलिखित कथनों में से कौन सा/से सही है /हैं ?

1. ये उन विशे षताओं को बनाए रखते हैं जो स्रोत क्षे तर् से हासिल करते हैं ।
2. जब महाद्वीपीय वायु राशियाँ समु दर् के ऊपर बढती हैं , तो यह वाष्पीकरण को बढ़ाने में मदद करती
है ।
3. जब वायु राशियाँ एक ठं डी  सतह की ओर बढ़ती हैं , तो इससे वर्षा होती है ।

निम्नलिखित कू टों का उपयोग करके सही उत्तर का चयन कीजिए:

A  1 and 2 only
केवल 1 और 2

2 and 3 only
केवल 2 और 3

C  1 and 3 only
केवल 1 और 3

D  1, 2 and 3
1, 2 और 3
Solution : A

Explanation: 

Statement 1 is correct: Air masses retain the characteristics they acquire from the
source region even after travelling long distances. They are essentially conservative
in character. 

Statement 2 is correct: Maritime air masses have moist characteristics and produce a


large amount of precipitation. Continental air masses are dry and produce limited
amounts of precipitation. When continental air mass moves into the ocean, it helps
to increase evaporation. In doing so, it develops some properties of the maritime air
mass. 

Statement 3 is incorrect: When air mass moves over a cold surface, its lower layer
becomes cold. This creates a condition of stability. It prevents upward movement of
air which inhibits condensation and precipitation. Tropical air masses experience
this phenomenon.

व्याख्या:

कथन 1 सही है : वायु राशियों में लं बी दरू ी की यात्रा के बाद भी स्रोत क्षे तर् से प्राप्त
विशे षताएँ बनी रहती हैं । ये अनिवार्य रूप से  प्रकृति में अपरिवर्तनीय होती हैं ।

कथन 2 सही है : समु दर् ी वायु राशियों में नमी वाली विशे षताएं होती हैं और इससे बड़ी मात्रा
में वर्षा होती है । महाद्वीपीय वायु राशियाँ शु ष्क होती हैं और इससे सीमित मात्रा में वर्षा होती
है । जब महाद्वीपीय वायु राशियाँ महासागर की ओर जाती हैं , तो यह  वाष्पीकरण को बढ़ाने
में  मदद करती है । ऐसी स्थिति में इसमें समु दर् ी वायु राशियों की कुछ विशे षताएँ आ जाती हैं ।

कथन 3 गलत है : जब वायु राशियाँ एक ठं डी सतह की ओर जाती हैं , तो इसकी निचली परत
ठं डी हो जाती है । इससे स्थिरता की स्थिति बनती है । यह हवा के ऊपर की ओर गति को रोकता
है जो संघनन और वर्षा को रोकता है । उष्णकटिबं धीय वायु राशियों में ऐसी स्थिति दे खी जाती
है ।
 22 s

69
Q. With reference to ‘Plea bargaining’ in the codes of Criminal Procedure, 1973,
consider the following statements: 

1. Plea bargaining can be initiated only by the accused. 


2. It is not available for the offences committed against a woman. 
3. It will not apply to offences that attract a prison term of above seven years. 

Which of the above statements is/are correct?


Q. आपराधिक प्रक्रिया सं हिता,1973 के तहत ‘प्ली बार्गेनिं ग’ (Plea bargaining) के सं दर्भ में ,
निम्नलिखित कथनों पर विचार कीजिए:

1. प्ली बार्गेनिं ग की पहल केवल अभियु क्तों द्वारा की जा सकती है ।


2. किसी महिला के खिलाफ किए गए अपराधों के लिए यह उपलब्ध नहीं है ।
3. यह उन अपराधों पर लागू नहीं होगा जिसमें सात साल से अधिक की जे ल की सजा का प्रावधान है ।

उपर्युक्त कथनों में से कौन सा/से सही है /हैं ?

A  1 only
केवल 1

B  1 and 2 only
केवल 1 और 2

C  2 and 3 only
केवल 2 और 3

D  1, 2 and 3
1, 2 और 3
Solution : D

Explanation: 
Context: Recently many members of the Tablighi Jamaat belonging to different
countries have obtained release from court cases by means of plea bargaining. 
Plea bargaining refers to a person charged with a criminal offence negotiating with
the prosecution for a lesser punishment than what is provided in law by pleading
guilty to a less serious offence. 
Statement 1 is correct: In India, a plea bargaining process can be initiated only by
the accused, not by the prosecutor. The applicant should state that it is a
voluntary preference and that he/she has understood the nature and extent of
punishment provided in law for the offence. 
Statement 2 is correct: It is not available for those that involve offences affecting
the “socio-economic conditions” of the country, or committed against a woman or
a child below the age of 14. 
Statement 3 is correct: Only those who have been charged for an offence that does
not attract the death sentence, life sentence or a prison term above seven
years can make use of the plea bargaining scheme. 
व्याख्या: 
संदर्भ : हाल ही में विभिन्न दे शों से सं बंधित तब्लीगी जमात के कई सदस्यों को ‘प्ली बारगे निंग’
(Plea Bargaining) प्रक्रिया के जरिए अदालती मामलों से रिहाई/मु क्ति मिली है ।।
प्ली बार्गेनिं ग से तात्पर्य एक ऐसी प्रक्रिया से है जिसमें किसी कम गं भीर प्रकृति के  दं डनीय अपराध
हे तु आरोपित व्यक्ति अपने अपराध को स्वीकार कर ‘प्ली बारगे निंग’ की प्रक्रिया के जरिए कानून के
तहत निर्धारित सज़ा से कम सज़ा प्राप्त करने के लिए अभियोजन पक्ष से समझौता करता है ।
कथन 1 सही है : भारत में , प्ली बार्गेनिं ग पहल केवल अभियु क्तों द्वारा की जा सकती है न कि
अभियोजन पक्ष द्वारा।आवे दक को यह बताना चाहिए कि यह एक स्वै च्छिक निर्णय है और उसने
अपराध के लिए कानून में प्रदान की गई सजा की प्रकृति और सीमा को समझा है ।
कथन 2 सही है : यह उन लोगों के लिए उपलब्ध नहीं है , जिन्होंने दे श की "सामाजिक-आर्थिक
स्थिति" को प्रभावित करने वाला अपराध किया हो, या महिला या 14 साल से कम उम्र के बच्चे के
खिलाफ अपराध किया हो।
कथन 3 सही है : यह केवल उन लोगों पर लागू होता है जिन्होंने अपराध किया हो,परं तु मौत की
सजा, आजीवन कारावास या सात साल से अधिक की कारावास की सजा पाने वाले व्यक्ति प्ली
बार्गेनिं ग का उपयोग नहीं कर सकते हैं ।
 1 m 17 s

-0.67/2.0 

70
Q. With reference to the Katabatic winds, which of the following statements
is/are incorrect?

1. It is a low-density air which rises above from lower altitude to higher altitude.
2. It is a low-density air which rises above from lower altitude to higher altitude.
3. It is observed in the oceanic atmosphere.

Select the correct answer using the codes given below:

Q. अवरोही पवन के सं दर्भ में , निम्नलिखित कथनों में से कौन सा/से  गलत  है /हैं ?

1. यह निम्न घनत्व वाली पवन है जो निम्न उन्नतां श से उच्च उन्नतां श की ओर बहती है ।


2. इसके प्रवाह के लिए तीव्र वाष्पीकरण उत्तरदायी होता है ।
3. यह महासागरीय वातावरण में बहती है ।

निम्नलिखित कू टों का उपयोग करके सही उत्तर चु निए:

A  1 only
केवल 1

B  1 and 3 only
केवल 1 और 3

C  2 only
केवल 2

D  1, 2 and 3
1, 2 और 3
Solution : D

Explanation:

Statement 1 is incorrect: It is a wind that carries high-density air from a higher


elevation down a slope.

Statement 2 is incorrect: It flows as drainage down the slope using the gravitational
force.

Statement 3 is incorrect: It is observed in mountains and valley regions or even in


plateau regions. In the oceanic atmosphere, we find land and sea breezes.

व्याख्या:

कथन 1 गलत है : यह उच्च घनत्व वाली पवन है जो उच्च उन्नतां श से निम्न उन्नतां श की ओर
बहती है ।

कथन 2 गलत है : यह गु रुत्वाकर्षण बल की सहायता से अपवाह की तरह ढलान के साथ नीचे
की ओर बहती है ।

कथन 3 गलत है : यह पर्वतों और घाटी क्षे तर् ों या पठारी क्षे तर् ों में बहती है । महासागरीय
वातावरण में , स्थलीय और समु दर् ी हवाएँ बहती हैं ।
 
 0 s

71
Q. With reference to erosional landforms, which of the following statements is/are
correct?

1. A canyon is almost equal in width at its top as well as its bottom. 


2. Potholes are formed in the lower course of the river due to the turbulent flow of
water. 
3. Entrenched meanders are symmetrical and Ingrown meanders are asymmetrical. 

Select the correct answer using the codes given below: 


Q. अपरदनकारी भू-आकृतियों (erosional landforms) के सं दर्भ में , निम्नलिखित कथनों में से कौन
सा/से कथन सही है /हैं ?

1. एक घाटी(canyon) के शीर्ष और तली की चौड़ाई लगभग बराबर होती है ।


2. पानी के उग्र प्रवाह के कारण नदी के निचले प्रवाह मार्ग में खड्ड(Potholes) का निर्माण होता है ।
3. गभीरीभूत विसर्प(Entrenched meanders) समरूप होते हैं और अं त:कर्तित विसर्प(Ingrown
meanders) विषम होते हैं ।

निम्नलिखित कू टों का उपयोग करके सही उत्तर का चयन कीजिए:

A  1 only
केवल 1

B  2 only
केवल 2

C  3 only
केवल 3

D  2 and 3 only
केवल 2 और 3
Solution : C

Explanation: 
Statement 1 is incorrect: A gorge is a deep valley with very steep and straight
sides. It is almost equal in width at its top as well as its bottom. A canyon is
characterized by steep step-like side slopes and might be as deep as a gorge.
Hence a canyon is wider at its top than at its bottom. 
Statement 2 is incorrect: Potholes are cylindrical holes drilled into the river-bed
that vary in depth and diameter by the action of flowing water. They are found in
the upper course of a river due to the turbulent flow of water where it has enough
potential energy to erode vertically. 
Statement 3 is correct: Entrenched meanders are symmetrical valley sides
formed due to rapid down-cutting of the river. On the other hand, Ingrown
meanders are asymmetrical in nature. They are formed when the river cuts at a less
rapid pace.  
व्याख्या: 
कथन 1 गलत है : गॉर्ज (gorge) एक गहरी घाटी होती है जिसकी ढ़ाल अत्यधिक तीव्र और किनारा
लं बवत होता है ।इसके शीर्ष और तली की चौड़ाई लगभग बराबर होती है ।एक घाटी(canyon) की
विशे षता होती है कि यह तीव्र ढ़ाल के साथ सीढीनु मा किनारे वाला और गॉर्ज के समान गहरा हो
सकता है ।अतः घाटी(canyon) अपने निचले भाग की तु लना में अपने शीर्ष पर अधिक चौड़ी होती
है ।
कथन 2 गलत है : खड्ड(Potholes) बे लनाकार छिद्र होते हैं जो नदी-तल में बनते हैं तथा बहते पानी
के कारण गहराई और व्यास में भिन्न होते हैं ।ये पानी के उग्र प्रवाह के कारण नदी के ऊपरी प्रवाह
मार्ग में पाए जाते हैं , जहाँ लं बवतरूप से अपरदन हे तु इनमें पर्याप्त स्थितिज ऊर्जा होती है ।
कथन 3 सही है : गभीरीभूत विसर्प(Entrenched meanders) समरूप घाटी के किनारे होते हैं
ू री ओर अंत:कर्तित विसर्प(Ingrown
जिनका निर्माण नदी के तीव्र कटाव के कारण होता है । दस
meanders) की प्रकृति विषम होती है ।इनका निर्माण तब होता है जब नदी कम तीव्र गति से
कटाव करती है ।
 0 s

72
Q. Arrange the following cities from East to West:

1. Damascus
2. Ankara
3. Baghdad
4. Riyadh

Select the correct answer using the codes given below:

Q. निम्नलिखित शहरों को पूर्व से पश्चिम दिशा की ओर व्यवस्थित कीजिए:

1. दमिश्क
2. अं कारा
3. बगदाद
4. रियाद

निम्नलिखित कू टों का उपयोग करके सही उत्तर चु निए:



4-3-2-1
 


3-4-2-1


3-4-1-2


4-3-1-2
Solution : D

Explanation:

All the options are Capitals of the countries in Western Asia.


Damascus is the capital of Syria.
Ankara is the capital of Turkey.
Baghdad is the capital of Iraq.
Riyadh is the capital of Saudi Arabia.

व्याख्या:

सभी विकल्प पश्चिमी एशिया में स्थित दे शों की राजधानियाँ हैं ।


दमिश्क सीरिया की राजधानी है ।
अंकारा तु र्की की राजधानी है ।
बगदाद इराक की राजधानी है ।
रियाद सऊदी अरब की राजधानी है ।
 0 s

73
Q. The term ‘Bathynomus raksasa’, recently seen in the news is related to:

Q. ‘बाथिनोमस रक्सासा (Bathynomus Raksasa)’ शब्द हाल ही में चर्चा में रहा है ,जो
निम्नलिखित में से किससे सं बंधित है ?

Traditional religious ritual practiced by the tribes in Arunachal Pradesh.



अरुणाचल प्रदे श में जनजातियों द्वारा किया जाने वाला पारं परिक धार्मिक
अनु ष्ठान।

B  Microscopic algae that caused mass deaths of elephants in Botswana.


अतिसूक्षम शै वाल जो बोत्सवाना में हाथियों की सामूहिक मौत का कारण बना।

C  Super giant isopod species found in the Indian Ocean.


हिं द महासागर में पाई गयी अति विशाल आइसोपॉड की प्रजातियाँ ।
D  Tree frog that is endemic to western Ghats.
वृ क्ष में ढक (Tree frog) जो पश्चिमी घाट के लिए स्थानिक हैं ।
Solution : C

Explanation: 
Context: Recently, scientists have reported the discovery of the first super giant
isopod species in the eastern Indian Ocean named ‘Bathynomus raksasa’. 
It has been described as the ‘cockroach of the sea’. It acts as a scavenger i.e., it
eats the remains of dead marine animals, such as whales and fish, but can also go
for long periods without food, a trait that it shares with the cockroach. 
व्याख्या:
संदर्भ : हाल ही में ,वै ज्ञानिकों ने पूर्वी हिं द महासागर में  ‘बाथिनोमस रक्सासा (Bathynomus
Raksasa)’ नाम के पहले अति विशाल आइसोपॉड की प्रजातियों की खोज की सूचना दी है ।इसे
समु दर् के 'कॉकरोच' के रूप में वर्णित किया गया है ।यह सफाई करने वाले जीव के रूप में कार्य करता
है , अर्थात यह मृ त समु दर् ी जानवरों जै से व्हे ल और मछली के अवशे षों को खाता है , जै से व्हे ल और
मछली,ले किन लं बी अवधि तक यह भोजन के बिना भी रह सकते हैं ,एक ऐसी विशे षता जो कॉकरोच
के सदृश है ।
 0 s

74
Q. The Amazon rainforest which is sometimes described as the lungs of the earth
spreads over which of the following countries?

1. Peru
2. Chile
3. Bolivia
4. Guyana

Select the correct answer using the codes given below:

Q. अमे ज़ॅन वर्षावन जिसे कभी-कभी 'पृ थ्वी के फेफड़े ' के रूप में वर्णित किया जाता है , निम्नलिखित में
से किन दे शों में फैला हुआ है ?

1. पे रू
2. चिली
3. बोलीविया
4. गु याना

निम्नलिखित कू टों का उपयोग करके सही उत्तर चु निए:


A  1, 3 and 4 only
केवल 1, 3 और 4

B  2 and 3 only
केवल 2 और 3

C  1 and 4 only
केवल 1 और 4

1, 2, 3 and 4

1, 2, 3 और 4
 
Solution : A

Explanation:

The Amazon spans across eight countries: Brazil, Bolivia, Peru, Ecuador, Colombia,
Venezuela, Guyana, Suriname and French Guiana, an overseas territory of France.

The landscape contains:

 One in ten known species on Earth.


 1.4 billion acres of dense forests, half of the planet's remaining tropical forests.
 4,100 miles of winding rivers.
 2.6 million square miles in the Amazon basin, about 40% of South America.

                         Figure: Amazon
forest

व्याख्या:

अमे ज़ॅन वर्षावनों का विस्तार आठ दे शों में है : ब्राजील, बोलीविया, पे रू, इक्वाडोर, कोलं बिया,
वे नेजुएला, गु याना, सूरीनाम और फ् रें च गु याना, जो फ् राँस का एक विदे शी क्षे तर् है ।
इस क्षे तर् में शामिल है :

   पृ थ्वी पर विद्यमान दस प्रजातियों में से एक।


 1.4 अरब एकड़ में घने जं गल, पृ थ्वी के शे ष उष्णकटिबं धीय वनों का आधा भाग।
 4,100 मील लं बी नदियाँ ।
 दक्षिण अमे रिका के अमे ज़ॅन बे सिन का 2.6 मिलियन वर्ग मील, लगभग 40 प्रतिशत।

                                      चित्र: अमेज़ॅन
वन 
 0 s

75
Q. Consider the following pairs:
 
S. no.  Layer of Earth  Composition 
1. Oceanic crust  Silicon and Aluminium
2. Mantle Silicon and Magnesium
3. Core Nickel and Iron

Which of the above pairs is/are correctly matched?

Q. निम्नलिखित यु ग्मों पर विचार कीजिए:


 
क् रम सं ख्या पृ थ्वी की परत सं रचना 
1. महासागरीय भूपर्पटी  सिलिकॉन एवं एल्यूमीनियम
2. में टल सिलिकॉन एवं मै ग्नीशियम
3. कोर निकेल एवं आयरन

उपर्युक्त यु ग्मों में से कौन सा/से सु मेलित है / हैं ?



1 and 2 only
केवल 1 और 2

B  3 only
केवल 3

C  2 and 3 only
केवल 2 और 3

D  1, 2 and 3
1, 2 और 3
Solution : C

Explanation:

Pair 1 is incorrectly matched: Oceanic crust is made up of Silicon and Magnesium.


Continental crust is made up of Silicon and Aluminium.

Pair 2 is correctly matched: Mantle and Oceanic crust are made up of Silicon and
Magnesium.

Pair 3 is correctly matched: Core is made up of Nickel and Iron.

व्याख्या:

यु ग्म 1 सुमेलित नहीं है : महासागरीय भूपर्पटी का निर्माण सिलिकॉन और मै ग्नीशियम से हुआ


है । महाद्वीपीय  भूपर्पटी का निर्माण सिलिकॉन और एल्यूमीनियम से हुआ है ।

यु ग्म 2 सुमेलित है : में टल एवं महासागरीय भूपर्पटी का निर्माण सिलिकॉन और मै ग्नीशियम से


हुआ है ।

यु ग्म 3 सुमेलित है : कोर का निर्माण निकेल और आयरन से हुआ है । 


 0 s

76
Q. With reference to ‘Environmental Determinism’, which of the following statements
is/are incorrect?

1. It is the belief that human beings determine environmental development.


2. It says that the environment sets certain limitations, but culture is determined by
social conditions.
3. Level of technology is low and human society is primitive in environmental
determinism.
Select the correct answer using the codes given below:

Q. 'पर्यावरण नियतिवाद (Environmental Determinism)' के सं दर्भ में , निम्नलिखित कथनों में से


कौन सा/से  गलत  है /हैं ?

1. यह ऐसी धारणा है जिसमें यह माना जाता है कि मानव पर्यावरणीय विकास का निर्धारण करता है ।
2. इसके अनु सार पर्यावरण कुछ सीमाएँ निर्धारित करता है , ले किन सं स्कृति सामाजिक परिस्थितियों से
निर्धारित होती है ।
3. प्रौद्योगिकी का स्तर निम्न है और मानव समाज पर्यावरण निर्धारण में प्राचीन है ।

निम्नलिखित कू टों का उपयोग करके सही उत्तर चु निए:

A  1 only
केवल 1

B  1 and 2 only
केवल 1 और 2

C  2 only
केवल 2

D  1, 2 and 3
1, 2 और 3
Solution : D

Explanation:

Statement 1 is incorrect: Environmental determinism is the belief that the


environment and its physical factors such as landforms and climate, determines the
patterns of human culture and societal development.

Statement 2 is incorrect: Possibilism is the theory that the environment sets certain
limitations, but culture is otherwise determined by social condition.

Statement 3 is incorrect: In this type of interaction the level of technology is very


low and the human development is primitive.

व्याख्या:

कथन 1 गलत है : पर्यावरणीय नियतिवाद ऐसी धारणा है जिसमें यह माना जाता है कि पर्यावरण
और इसके भौतिक कारक जै से कि भू-आकृतियाँ और जलवायु , मानव सं स्कृति तथा सामाजिक
विकास के स्वरुप को निर्धारित करते हैं ।

कथन 2 गलत है : संभववाद सिद्धांत के अनु सार, पर्यावरण कुछ सीमाएँ निर्धारित करता है ,
ले किन सं स्कृति सामाजिक परिस्थितियों से निर्धारित होती है ।
कथन 3 गलत है : इस प्रकार की अं तःक्रिया में , प्रौद्योगिकी का स्तर बहुत निम्न होता है
और मानव विकास प्राचीन है ।
 
 0 s

77
Q. The military conflict over the Nagorno-Karabakh region, recently in the news is
between which of the following countries?

Q. हाल ही में चर्चा में रहा नागोर्नो-काराबाख (Nagorno-Karabakh) क्षे तर् को ले कर हुआ सै न्य
सं घर्ष   निम्नलिखित में से किन दे शों के बीच हुआ है ?

A  Armenia and Azerbaijan


आर्मेनिया और अज़रबै जान

B  Armenia and Russia


आर्मेनिया और रूस

C  Azerbaijan and Georgia


अज़रबै जान और जॉर्जिया

D  Armenia and Georgia


आर्मेनिया और जॉर्जिया
Solution : A

Explanation:

It is the military conflict between Armenia and Azerbaijan. Armenia, Azerbaijan


and Russia have signed an agreement to end military conflict over the disputed
enclave of Nagorno-Karabakh. The region is internationally recognised as
Azerbaijani but has been run by ethnic Armenians since 1994.
Under the deal, Azerbaijan will hold on to areas of Nagorno-Karabakh that it has
taken during the conflict. Armenia has also agreed to withdraw from several other
adjacent areas over the next few weeks.

             Figure: Nagorno-Karabakh region


व्याख्या:

यह सै न्य सं घर्ष आर्मे निया और अजरबैजान के बीच हुआ है । आर्मेनिया, अजरबै जान और रूस ने
नागोर्नो-काराबाख के विवादित क्षे तर् को ले कर चल रहे  सै न्य संघर्ष को समाप्त करने के लिए
एक समझौते पर हस्ताक्षर किए हैं । इस क्षे तर् को अं तर्राष्ट् रीय स्तर पर अज़रबै जान का माना
जाता है , परं तु 1994 से इस क्षे तर् का नियंतर् ण नृजातीय अर्मे नियाई लोगों के पास है ।

इस समझौते के तहत, अजरबै जान का नागोर्नो-काराबाख के उन क्षे तर् ों पर कब्जा बना रहे गा
जिन पर उसने सं घर्ष के दौरान अधिकार कर लिया था। आर्मेनिया भी कई अन्य क्षे तर् ों से वापस
हटने के लिए सहमत हो गया है ।

                 चित्र: नागोर्नो-काराबाख क्षे तर्


 0 s

78
Q. With regards to Quaternary activities, which of the following statements is/are
correct?

1. Quaternary activities are performed by the highest level of decision-makers.


2. These are referred to as White-collared jobs.
3. Judges come under the quaternary sector. 

Select the correct answer using the codes given below: 

Q. चतु र्विध गतिविधियों (Quaternary Activities) के सं बंध में निम्नलिखित में से कौन सा/से कथन
सही हैं / हैं ?

1. चतु र्विध गतिविधियाँ उच्चतम स्तर के निर्णयकर्त्ताओं द्वारा निष्पादित की जाती हैं ।
2. इन्हें सफेद कॉलर (White-Collared) नौकरियों के रूप में जाना जाता है ।
3. न्यायाधीश चतु र्विध क्षे तर् के अं तर्गत आते हैं ।

निम्नलिखित कू टों का उपयोग करके सही उत्तर चु निए:

A  1 only
केवल 1

2 only
केवल 2

C  1 and 3 only
केवल 1 और 3

D  1, 2 and 3
1, 2 और 3
Solution : B

Explanation:

Statement 1 is incorrect: Quaternary and Quinary activities are types of tertiary


activities. Quaternary activities centre around research, development and may be
seen as an advanced form of services involving specialised knowledge and technical
skills. While Quinary activities are performed by the highest level of decision-
makers.

Statement 2 is correct: Activities under the Quaternary sector are called as White-


collared jobs, and those under the Quinary sector are called as Gold-collared jobs.

Statement 3 is incorrect: Judges are part of the highest level of decision-makers.


They come under Quinary activity.

व्याख्या:

कथन 1 गलत है : चतु र्विध और क्रियात्मक गतिविधियाँ (Quaternary and Quinary


Activities)  तृ तीयक गतिविधियों के प्रकार हैं । चतु र्वि ध गतिविधियाँ अनु संधान, विकास से
सं बंधित होती हैं जिन्हें विशे ष ज्ञान और तकनीकी कौशल वाली से वाओं का उन्नत रूप कहा जा
सकता है । जबकि क्रियात्मक गतिविधियाँ उच्चतम स्तर के निर्णयकर्त्ताओं द्वारा निष्पादित की
जाती हैं ।

कथन 2 सही है : चतु र्विध क्षे तर् के अं तर्गत आने वाली गतिविधियों को व्हाइट-कॉलर वाली
नौकरियों के रूप में जाना जाता है और क्रियात्मक क्षे तर् के अं तर्गत आने वाली गतिविधियों
को गोल्ड-कॉलर वाली नौकरियों के रूप में जाना जाता है ।

कथन 3 गलत है : न्यायाधीश उच्चतम निर्णयकर्त्ताओं की श्रेणी में आते हैं । वे क्रियात्मक
गतिविधियों के अं तर्गत आते हैं ।
 0 s

79
Q. Arrange the following Indian Ocean island nations from North to South:

1. Comoros
2. Mauritius
3. Maldives
4. Seychelles

Select the correct answer using the codes given below:

Q. निम्नलिखित हिं द महासागरीय द्वीपीय राष्ट् रों को उत्तर से दक्षिण दिशा की ओर व्यवस्थित
कीजिए:

1. कोमोरोस
2. मॉरीशस
3. मालदीव
4. से शेल्स

निम्नलिखित कू टों का उपयोग करके सही उत्तर चु निए:


3-4-2-1


4-3-2-1


4-3-1-2


3-4-1-2
Solution : D

Explanation:
व्याख्या:

 0 s

80
Q. Consider the following statements: 

1. The level of ocean water is higher near the equator than in the middle latitudes. 
2. Oceanic circulation helps ports of Denmark and Norway remain ice-free during
winters. 
3. The oceanic circulation pattern roughly corresponds to the earth’s atmospheric
circulation pattern. 

Which of the statements given above is/are correct?


Q. निम्नलिखित कथनों पर विचार कीजिए:

1. समु दर् के पानी का स्तर भूमध्य रे खा के पास मध्य अक्षां शों की तु लना में ऊँचा होता है ।
2. महासागरीय प्रवाह के कारण डे नमार्क और नॉर्वे के बं दरगाह सर्दियों के दौरान बर्फ मु क्त रहते हैं ।
3. महासागरीय परिसं चरण पै टर्न (oceanic circulation pattern) आमतौर पर पृ थ्वी के वायु मंडलीय
परिसं चरण पै टर्न (atmospheric circulation patter) से मे ल खाता है ।

उपर्युक्त कथनों में से कौन सा/से सही है /हैं ?



1 and 2 only
केवल 1 और 2

B  2 and 3 only
केवल 2 और 3

C  1 and 3 only
केवल 1 और 3

D  1, 2 and 3
1, 2 और 3
Solution : D

Explanation: 
Statement 1 is correct: Heating by solar energy causes the water to expand. Hence
the level of ocean water is about 8 cm higher near the equator than in the middle
latitudes. 
Statement 2 is correct: Ocean currents act like a conveyor belt by transporting
warm water from the equator toward the poles and cold water back to the tropics.
The warm water is brought to the coast of western Europe and Norway by the North
Atlantic drift, which is a branch of the Gulf Stream. This helps ports of Denmark and
Norway remain ice-free during winters. 
Statement 3 is correct: The oceanic circulation pattern roughly corresponds to
the earth’s atmospheric circulation pattern. At middle latitudes, the air
circulation over the oceans is mainly anti-cyclonic. The oceanic circulation
pattern also roughly corresponds to the same (due to pressure belts, wind
systems and coriolis force). 
व्याख्या: 
कथन 1 सही है : सौर ऊर्जा द्वारा उत्पन्न ऊष्मा से पानी का फैलाव होता है ।इसलिए समु दर् के पानी
का स्तर भूमध्य रे खा के पास मध्य अक्षां श की तु लना में लगभग 8 से मी ऊँचा होता है ।
कथन 2 सही है : महासागरीय धाराएँ भूमध्य रे खा से गर्म जल को ध्रुवों की ओर ले जाकर तथा ठं डे 
जल को वापस उष्ण कटिबं ध की ओर ले जाकर एक वाहक पे टी की तरह कार्य करती हैं ।गर्म पानी
उत्तरी अटलां टिक बहाव,जो गल्फ स्ट् रीम की एक शाखा है , द्वारा पश्चिमी यूरोप और नॉर्वे के तट पर
तक पहुँचता है ।इससे सर्दियों के दौरान डे नमार्क और नॉर्वे के बं दरगाहों को बर्फ से मु क्त रखने में मदद
मिलती है ।
कथन 3 सही है : महासागरीय परिसं चरण पै टर्न आमतौर पर पृ थ्वी के वायु मंडलीय परिसं चरण पै टर्न से
मे ल खाता है ।मध्य अक्षां शों पर,महासागरों के ऊपर हवा का परिसं चरण मु ख्य रूप से
एं टीसाइक्लोनिक होता है ।महासागरीय परिसं चरण पै टर्न भी लगभग उसी के अनु रूप होता है (दबाव
पे टी, पवन प्रणाली और कोरिओलिस बल के कारण)।
 0 s

81
Q. Regarding the Red Coral Kukri, a very rare species which was spotted recently,
consider the following statements:

1. It is a nocturnal non-venomous reptile.


2. It is endemic to the corals of the Gulf of Mannar region.

Which of the statements given above is/are correct?

Q. हाल ही में दे खी गई एक बहुत ही दुर्लभ प्रजाति 'लाल कोरल कुकरी' के बारे में , निम्नलिखित
कथनों पर विचार कीजिए:

1. यह एक निशाचर विष रहित सरीसृ प है ।


2. यह मन्नार क्षे तर् की खाड़ी के प्रवालों के लिए स्थानिक है ।

उपर्युक्त कथनों में से कौन सा/से सही है /हैं ?

A  1 only
केवल 1

B  2 only
केवल 2

C  Both 1 and 2
1 और 2 दोनों

D  Neither 1 nor 2
न तो 1, न ही 2
Solution : A

Explanation:

Recently, a "very rare" species named Red Coral Kukri snake was spotted in the
Dudhwa National Park in Uttar Pradesh's Lakhimpur Kheri.

Statement 1 is correct: It is a nocturnal non-venomous reptile which feeds on insects


and worms.

Statement 2 is incorrect: It was spotted in the Dudhwa National Park in Uttar


Pradesh's Lakhimpur Kheri and hence is not endemic to the Gulf of Mannar region.
व्याख्या:

हाल ही में , उत्तर प्रदे श के लखीमपु र खीरी स्थित दुधवा राष्ट् रीय उद्यान में लाल कोरल कुकरी
नामक साँप की एक "बहुत दुर्लभ" प्रजाति दे खी गई थी।

कथन 1 सही है : यह एक निशाचर विष रहित सरीसृ प है जो कीड़े -मकोड़े और कृमि को खाता है ।

कथन 2 गलत है : इसे उत्तर प्रदे श के लखीमपु र खीरी स्थित दुधवा राष्ट् रीय उद्यान में दे खा
गया था और इसलिए यह मन्नार क्षे तर् की खाड़ी के लिए स्थानिक नहीं है ।
 0 s

82
Q. With reference to the Jet Streams, consider the following statements:

1. They are high altitude, high-speed air currents confined to mid-latitudes.


2. Each large meander, or wave, within the jet stream is known as a Rossby wave.
3. They affect the formation and duration of the Indian monsoon.

Which of the statements given above is/are correct?

Q. जे ट स्ट् रीम के सं दर्भ में निम्नलिखित कथनों पर विचार कीजिए:

1. ये अधिक ऊंचाई एवं उच्च गति वाली वायु धाराएँ हैं जो मध्य अक्षां शों में पाई जाती हैं ।
2. जे ट स्ट् रीम की प्रत्ये क बड़ी धारा या लहर को रॉस्बी लहर (Rossby wave) के रूप में जाना जाता
है ।
3. ये भारतीय मानसून की उत्पत्ति और अवधि को प्रभावित करते हैं ।

उपर्युक्त कथनों में से कौन सा/से सही है / हैं ?

A  1 only
केवल 1

B  1 and 2 only
केवल 1 और 2

C  2 and 3 only
केवल 2 और 3

D  1 and 3 only
केवल 1 और 3
Solution : C

Explanation:
Statement 1 is incorrect: Jet streams are high speed, narrow, meandering air currents
in the atmosphere usually found at the tropopause. Some of the Jet streams are also
located at the lower levels of the atmosphere. They are located across the globe.

Statement 2 is correct: Jet streams flow in meander path, each large meander, or


wave within the jet stream is known as a Rossby Wave.

Statement 3 is correct: The tropical easterly jet stream affects the formation and
duration of Indian and African summer monsoons. During summer this system occurs
over Southeast Asia, India, the Arabian Sea and tropical Africa.

व्याख्या:

कथन 1 गलत है : जे ट स्ट् रीम वायु मं डल की उच्च गति वाली, सं कीर्ण वायु धाराएँ हैं जो
आमतौर पर क्षोभसीमा (Tropopause) पर पाई जाती हैं । कुछ जे ट स्ट् रीम धाराएँ वायु मं डल के
निचले स्तरों में भी स्थित होती हैं । ये दु निया भर में पाई जाती हैं ।

कथन 2 सही है : जे ट स्ट् रीम लहरदार पथ में प्रवाहित होती है । जे ट स्ट् रीम की प्रत्ये क बड़ी
लहर या तरं ग को रॉस्बी लहर (Rossby Wave) के रूप में जाना जाता है ।

कथन 3 सही है : उष्णकटिबं धीय पूर्वी जे ट स्ट् रीम भारतीय और अफ् रीकी ग्रीष्कालीन मानसून
के उद्भव और अवधि को प्रभावित करता है । यह प्रक्रिया ग्रीष्म ऋतु में दक्षिण पूर्व
एशिया, भारत, अरब सागर और उष्णकटिबं धीय अफ् रीका में होती है ।
 0 s

83
Q. With regard to Phacoliths, which one of the following statements is correct?

Q. फैकोलिथ के सं बंध में ,निम्नलिखित कथनों में से कौन सा सही है ?

A  A dome-shaped intrusive body connected with a pipe-like conduit.


गु ं बद के आकार का एक अं तर्वेधी पिं ड जो पाइप जै सी नलिका के साथ जु ड़ा हुआ होता है ।

B  A saucer-shaped intrusion with concave facing towards the sky.


तश्तरी के आकार का एक अं तर्वेधी पिं ड जिसका खोखला भाग आकाश की ओर होता है ।

C  A wavy mass of intrusive igneous rocks at the base of syncline or anticline.


अभिनति या अपनत के आधार पर अं तर्वेधी आग्ने य चट् टानों का एक लहरदार द्रव्यमान।

A large mass of intrusive igneous rock that forms from cooled magma deep in the Earth's
crust.
अं तर्वे धी आग्ने य चट् टानों का एक बड़ा द्रव्यमान जो पृ थ्वी की पर्पटी में गहराई पर मै ग्मा के ठं डे
होने से निर्मित होता है ।
Solution : C

Explanation:

Option (a) is incorrect: Laccoliths are dome-shaped intrusive bodies. They have an


even base which is connected with a pipe-like conduit.

Option (b) is incorrect: Lopoliths are formed, due to movement of lava upwards,


whereby it intrudes in the horizontal cracks in a saucer shape, with a concave face
towards the sky.

Option (c) is correct: Phacoliths are the wavy mass of intrusive igneous rocks at the
base of syncline or anticline.

Option (d) is incorrect: Batholiths are the large mass of intrusive igneous rock that
forms from cooled magma deep in the Earth's crust.

व्याख्या:

कथन (a) गलत है : लै कोलिथ गु ं बद के आकार के अं तर्वे धी पिं ड होते हैं । उनमें एक समतल
आधार होता है जो एक पाइप जै सी नलिका के साथ जु ड़ा हुआ होता है ।

कथन (b) गलत है : लोपोलिथ का निर्माण ऊपर की ओर मै ग्मा की गति के कारण होता है , जहाँ
यह आसमान की ओर एक खोखली सं रचना के साथ एक तश्तरी के आकार में क्षै तिज दरारों में
अन्तर्वेधित हो जाता है ।

कथन (c)  सही है : फैकोलिथ अभिनति या अपनत के आधार पर अं तर्वे धी आग्ने य चट् टानों का
एक लहरदार द्रव्यमान होता है ।

कथन (d) गलत है : बै थोलिथ अं तर्वे धी आग्ने य चट् टानों का एक बड़ा द्रव्यमान होता है जो
पृ थ्वी की पर्पटी में गहराई पर मै ग्मा के ठं डे होने से निर्मित होता है ।
 0 s

84
Q. With reference to National Disaster Response Fund (NDRF), consider the
following statements:

1. It was constituted under the Disaster Management Act of 2005. 


2. It is placed under the Contingency fund of India. 
3. It is audited by the Comptroller and Auditor General (CAG) of India. 

Which of the above statements is/are correct?


Q. राष्ट् रीय आपदा प्रतिक्रिया कोष (National Disaster Response Fund-NDRF) के सं दर्भ में
निम्नलिखित कथनों पर विचार कीजिए:

1. इसका गठन 2005 के आपदा प्रबं धन अधिनियम (Disaster Management Act) के अं तर्गत हुआ
था।
2. इसे भारत की आकस्मिकता निधि के अं तर्गत शामिल किया गया है ।
3. इसका ले खा परीक्षण भारत के नियं तर् क और महाले खा परीक्षक (Comptroller and Auditor
General-CAG) द्वारा किया जाता है ।

उपर्युक्त कथनों में से कौन सा/से सही है /हैं ?

A  1 only
केवल 1

B  2 and 3 only
केवल 2 और 3

C  1 and 3 only
केवल 1 और 3

D  1, 2 and 3
1, 2 और 3
Solution : C

Explanation: 
Context: Recently, the Central Government has allowed contributions to National
Disaster Response Fund (NDRF) from any person or institution in the as per Section
46(1)(b) of the Disaster Management (DM) Act, 2005. 
Statement 1 is correct: It is constituted under Section 46 of the Disaster
Management Act, 2005. 
Statement 2 is incorrect: It is placed in the ‘Public Account’ of the Government of
India under the head of ‘reserve funds not bearing interest’. 
Statement 3 is correct: The Comptroller and Auditor General (CAG) of India audits
the accounts of NDRF. 
व्याख्या:
हाल ही में , केंद्र सरकार ने आपदा प्रबं धन (Disaster Management) अधिनियम, 2005 की धारा
46 (1) (B) के अनु सार किसी भी व्यक्ति या सं स्थान को  राष्ट् रीय आपदा प्रतिक्रिया कोष
(National Disaster Response Fund-NDRF) में योगदान की अनु मति प्रदान कर दी है ।
कथन 1 सही है : इसका गठन आपदा प्रबंधन अधिनियम  (Disaster Management Act), 2005
की धारा 46 के तहत किया गया है ।
कथन 2 गलत है : इसे भारत सरकार की ‘लोक लेखा’ (Public Account) में 'ब्याज रहित सं रक्षित
निधि' के अं तर्गत शामिल किया गया है ।
कथन 3 सही है : भारत का नियं तर् क और महाले खा परीक्षक (Comptroller and Auditor General-
CAG) NDRF के खातों का ले खा परीक्षण करता है ।
 0 s

85
Q. Consider the following statements: 

1. The area between Tropic of Cancer and Tropic of Capricorn is known as the Torrid
zone. 
2. The linear distance of a degree of latitude at the pole is longer than that at the
equator. 
3. The longitudes 180° East and 180° West are diametrically opposite to each other. 

Which of the following statements is/are correct?


Q. निम्नलिखित कथनों पर विचार कीजिए:

1. कर्क रे खा और मकर रे खा के बीच के क्षे तर् को उष्ण कटिबं ध (Torrid Zone) के रूप में जाना जाता है ।
2. ध्रुव पर अक्षां श के एक अं श की रे खीय दरू ी भूमध्य रे खा की तु लना में अधिक लं बी होती है ।
3. दे शांतर 180° पूर्व और 180° पश्चिम एक दस ू रे के विपरीत हैं ।

उपर्युक्त कथनों में से कौन सा/से सही है /हैं ?

A  1 only
केवल 1

B  1 and 2 only
केवल 1 और 2

C  2 and 3 only
केवल 2 और 3

D  1, 2 and 3
1, 2 और 3
Solution : B

Explanation: 
Statement 1 is correct: The area between Tropic of Cancer and Tropic of Capricorn
is known as the Torrid zone. The mid-day sun is exactly overhead at least once a
year on all latitudes in this zone. 
Statement 2 is correct: The linear distance of a degree of latitude at the pole is
longer than that at the equator. This is because the earth is flattened at the poles
which increases the linear distance with every degree of latitude traversed. 
Statement 3 is incorrect: The longitudes 180° East and 180° West are on the same
line. The Prime Meridian (0° longitude) and 180° longitude are diametrically opposite
to each other. 
व्याख्या:
कथन 1 सही है : कर्क रे खा और मकर रे खा के बीच के क्षे तर् को उष्ण कटिबं ध (Torrid Zone) के रूप में
जाना जाता है । इस क्षे तर् के सभी अक्षां शों पर वर्ष में कम से कम एक बार मध्य-दिवस का सूरज ठीक
सिर के ऊपर होता है ।
कथन 2 सही है : ध्रुव पर अक्षां श के एक अं श की रे खीय दरू ी भूमध्य रे खा की तु लना में अधिक लं बी
होती है । इसका कारण यह है कि पृ थ्वी ध्रुवों पर चपटी होती है   जिस कारण अक्षां श के प्रत्ये क अं श
के साथ रै खिक दरू ी बढ़ जाती है ।
कथन 3 गलत है : दे शांतर 180° पूर्व और 180° पश्चिम एक ही रे खा पर हैं । प्राइम मे रिडियन (0°
ू रे के विपरीत हैं ।
दे शांतर) और 180° दे शांतर एक दस
 0 s

86
Q. With reference to the ocean currents in the Indian Ocean, which of the following
statements are correct?

1. Conditions in the Northern Indian Ocean are not favourable for the development of
permanent ocean currents. 
2. If winds stop blowing, then ocean currents in the Indian ocean will cease to exist. 
3. The currents in the Northern Indian ocean change their flow direction due to
monsoon winds. 

Select the correct answer using the codes given below: 


Q. हिं द महासागर में महासागरीय धाराओं के सं दर्भ में निम्नलिखित में से कौन से कथन सही हैं ?

1. उत्तरी हिं द महासागर में परिस्थितियाँ स्थायी महासागरीय धाराओं की उत्पत्ति के अनु कूल नहीं हैं ।
2. अगर पवनें बहना बं द हो जाती हैं तो हिं द महासागर में समु दर् ी धाराएँ मौजूद नहीं रहें गी।
3. उत्तरी हिं द महासागर में धाराएँ मानसूनी हवाओं के कारण अपनी प्रवाह की दिशा बदल ले ती हैं ।

निम्नलिखित कू टों का उपयोग करके सही उत्तर चु निए:



1 and 2 only
केवल 1 और 2

B  1 and 3 only
केवल 1 और 3

C  2 and 3 only
केवल 2 और 3

D  1, 2 and 3
1, 2 और 3
Solution : B

Explanation: 
Statement 1 is correct: Land-Water Distribution (Dominance of Land) in the
Northern Indian ocean does not present favourable conditions for the development of
permanent ocean currents. 
Statement 2 is incorrect: Though the currents of the Northern Indian Ocean
demonstrate most convincingly the dominant effects of winds on the circulation of
ocean currents, winds alone are not a reason for the movement of ocean currents. It
is also influenced by temperature and salinity gradient in the ocean. 
Statement 3 is correct: The currents in the Northern Indian Ocean change their
flow direction twice a year due to North-East and South-West monsoon winds. 
व्याख्या:
कथन 1 सही है : उत्तरी हिं द महासागर में भूमि-जल वितरण (भूमि की प्रधानता) स्थायी धाराओं के
विकास के लिए अनु कूल परिस्थितियाँ प्रदान नहीं करता है ।
कथन 2 गलत है : हालाँ कि, उत्तरी हिं द महासागर की धाराएँ समु दर् की धाराओं के परिसं चरण पर
हवाओं के प्रमु ख प्रभाव को दर्शाती हैं , परं तु केवल हवाएँ ही महासागरीय धाराओं का कारण नहीं हैं ।
इन पर समु दर् के तापमान और लवणता का भी प्रभाव पड़ता है ।
कथन 3 सही है : उत्तर-पूर्व और दक्षिण-पश्चिम मानसूनी हवाओं के कारण उत्तरी हिं द महासागर की
धाराएँ वर्ष में दो बार अपनी प्रवाह दिशा बदलती हैं ।
 0 s

87
Q. Consider the following pairs:
 
S No Overseas territory Country
1. Gibraltar Spain
2. Falkland Islands United Kingdom
3. New Caledonia USA
4. Reunion Island France

Which of the pairs given above are correctly matched?

Q. निम्नलिखित यु ग्मों पर विचार कीजिए:


 
क् रम संख्या समुद ्री क्षे तर् दे श
1. जिब्राल्टर स्पे न
2. फ़ॉकलैं ड द्वीप यूनाइटे ड किंगडम
3. न्यू कैले डोनिया अमे रिका
4. रीयूनियन द्वीप फ् रांस

उपर्युक्त यु ग्मों में से कौन से सु मेलित हैं ?

A  1 and 3 only
केवल 1 और 3

B  2, 3 and 4 only


केवल 2, 3 और 4

C  1 and 4 only
केवल 1 और 4

D  2 and 4 only
केवल 2 और 4
Solution : D

Explanation:

Pair 1 is incorrectly matched: Gibraltar is overseas territory of the United Kingdom.

Pair 2 is correctly matched: Falkland Islands is overseas territory of the United


Kingdom.

Pair 3 is incorrectly matched: New Caledonia is overseas territory of France.

Pair 4 is correctly matched: Reunion island is overseas territory of France.


व्याख्या:

यु ग्म 1 सुमेलित नहीं है : जिब्राल्टर यूनाइटे ड किंगडम का समु दर् ी क्षे तर् है ।

यु ग्म 2 सुमेलित है : फ़ॉकलैं ड द्वीप यूनाइटे ड किंगडम का समु दर् ी क्षे तर् है ।

यु ग्म 3 सुमेलित नहीं है : न्यू कैले डोनिया फ् रांस का समु दर् ी क्षे तर् है ।

यु ग्म 4 सुमेलित है : रीयूनियन द्वीप फ् रांस का समु दर् ी क्षे तर् है ।

 0 s

88
Q. With reference to volcanoes, which of the following statements are correct?

1. The Shield volcanoes are characterized by low-explosivity. 


2. The Pacific Rim of Fire is a major chain of composite volcanoes. 
3. In Calderas, the magma chamber which supplies lava is close to the surface. 

Select the correct answer using the codes given below: 


Q. ज्वालामु खियों के सं दर्भ में निम्नलिखित में से कौन से कथन सही हैं ?

1. ढाल ज्वालामु खी की विशे षता कम विस्फोटक होना है ।


2. पै सिफिक रिम ऑफ फायर (Pacific Rim of Fire) मिश्रित ज्वालामु खी की एक प्रमु ख श्रख ृं ला है ।
3. काल्डे रा में , मै ग्मा चैं बर जो लावा की आपूर्ति करता है , सतह के करीब होता है ।

निम्नलिखित कू टों का उपयोग करके सही उत्तर चु निए:

A  1 and 2 only
केवल 1 और 2

B  2 and 3 only
केवल 2 और 3

C  1 and 3 only
केवल 1 और 3

D  1, 2 and 3
 1, 2 और 3
Solution : D

Explanation: 
Statement 1 is correct: The Shield volcanoes are the largest of all the volcanoes
on the earth, which are not steep. They become explosive if in some way water gets
into the vent; otherwise, they are characterized by low-explosivity. 
Statement 2 is correct: Composite volcanoes are characterized by outbreaks of
cooler and more viscous lavas than basalt. The Pacific Rim of Fire is a major chain
of composite volcanoes. 
Statement 3 is correct: The Calderas are known as the most explosive volcanoes
on the Earth. When they erupt, they are inclined to collapse on themselves rather
than constructing any structure. This explosiveness indicates the magma
chamber supplying lava is not only huge but also close to the surface. 
व्याख्या:
कथन 1 सही है : ढाल ज्वालामुखी पृ थ्वी पर पाए जाने वाले सभी ज्वालामु खियों में से सबसे बड़े होते
हैं , जिनकी ढलान तीव्र नहीं हैं । ये तभी विस्फोटक होते हैं यदि किसी तरह इनके मु ख में पानी प्रवे श
कर जाता है , अन्यथा, कम विस्फोटकता इनकी प्रमु ख विशे षता है ।
कथन 2 सही है : मिश्रित ज्वालामु खियों में बे साल्ट की तु लना में ठं डा और अधिक चिपचिपा लावा
होता  है । यही इनकी विशे षता है । पै सिफिक रिम ऑफ फायर मिश्रित ज्वालामु खियों की एक प्रमु ख
श्रख ृं ला है ।
कथन 3 सही है : काल्डे रा को पृ थ्वी पर सबसे विस्फोटक ज्वालामु खी के रूप में जाना जाता है । जब ये
फू टते हैं , तो किसी भी सं रचना का निर्माण करने के बजाय स्वयं पर गिरने लगते हैं । यह विस्फोटक
प्रवृ त्ति सं केत दे ती है कि लावा की आपूर्ति करने वाला इसका मै ग्मा चै म्बर न केवल विशाल होता है ,
बल्कि सतह के करीब भी होता है ।
 0 s

89
Q. It consists of the largest swamp forest in North Eastern India. The Brahmaputra
river passes through it and is home to feral horses. It is also recognised as an
Important Bird Area. The above passage describes which of the following protected
area:

Q. इसके अं तर्गत उत्तर पूर्वी भारत का सबसे बड़ा दलदली जं गल शामिल है । ब्रह्मपु तर् नदी इसके
बीच से गु जरती है और यहाँ जं गली घोड़े पाए जाते हैं । इसे एक महत्त्वपूर्ण पक्षी क्षे तर् के रूप में भी
मान्यता प्राप्त है । उपरोक्त गद्यां श निम्नलिखित में से किस सं रक्षित क्षे तर् का वर्णन करता है

A  Kaziranga National Park


काजीरं गा राष्ट् रीय उद्यान

B  Dibru-Saikhowa National Park


डिब्रू-साइखोवा राष्ट् रीय उद्यान

C  Laokhowa Wildlife Sanctuary


लोखोवा वन्यजीव अभयारण्य

D  Orang National Park


ओरं ग राष्ट् रीय उद्यान
Solution : B
Explanation:

Dibru-Saikhowa National Park lies in Dibru-Saikhowa Biosphere Reserve which


consists of the largest swamp forest in Northeastern India. The river Brahmaputra
passes through the park. It is famous for feral horses and wild buffaloes. It has been
identified as an Important Bird Area (IBA) by Birdlife international. 
व्याख्या:

डिब्रू-साइखोवा राष्ट् रीय उद्यान, डिब्रू-साइखोवा जै व मं डल रिज़र्व में स्थित है । यहाँ पूर्वोत्तर भारत
का सबसे बड़ा दलदली जं गल है । ब्रह्मपु तर् नदी इससे होकर गु जरती है । यह जं गली घोड़ों और
जं गली भैं सों के लिए प्रसिद्ध है । बर्डलाइफ इं टरने शनल (Birdlife International) द्वारा इसकी
पहचान एक 'महत्त्वपूर्ण बर्ड एरिया' (Important Bird Area-IBA) के रूप में की गई है ।
 0 s

90
Q. With reference to the shipping canals of the world, which one of the following
statements is incorrect?

Q. विश्व के नौ-परिवहन नहरों के सं दर्भ में , निम्नलिखित कथनों में से कौन सा गलत  है ?

A  The Suez canal lies between Port Said in the north and Port Suez in the south.
स्वे ज नहर उत्तर में सईद बं दरगाह और दक्षिण में स्वे ज बं दरगाह को जोड़ता है ।

B  The Kra isthmus canal is between the Andaman Sea and the Gulf of Thailand.
क् रा स्थलडमरूमध्य नहर अं डमान सागर और थाईलैं ड की खाड़ी को जोड़ता है ।

C  The Kiel canal is between the Mediterranean Sea and the Black Sea.
कील नहर भूमध्य सागर और काला सागर को जोड़ता है ।

D  The St. Lawrence Seaway connects the Atlantic Ocean with the Great Lakes.
सें ट लॉरें स समु दर् ी मार्ग अटलां टिक महासागर को ग्रेट झीलों से जोड़ता है ।
Solution : C

Explanation:

Option (a) is correct: The Suez canal is between Port Said in the north and Port Suez
in the south.

Option (b) is correct: The Kra isthmus canal is a proposed canal between the


Andaman Sea and the Gulf of Thailand.

Option (c) is incorrect: The Kiel canal is between the North Sea and the Baltic Sea.

Option (d) is correct: The St. Lawrence Seaway connects the Atlantic Ocean with the
Great Lakes.

व्याख्या:

विकल्प (a) सही है : स्वे ज नहर उत्तर में सईद बं दरगाह और दक्षिण में स्वे ज बं दरगाह को
जोड़ता है ।

विकल्प (b) सही है : क् रा स्थलडमरूमध्य नहर अं डमान सागर और थाईलैं ड की खाड़ी के बीच
प्रस्तावित नहर है ।

विकल्प (c) गलत है : कील नहर उत्तरी सागर और बाल्टिक सागर को जोड़ता है ।

विकल्प (d) सही है : सें ट लॉरें स समु दर् ी मार्ग अटलां टिक महासागर को ग्रेट झीलों से जोड़ता
है ।
 
 0 s

91
Q. Regarding the global distribution of temperature, consider the following
statements:

1. In January, the isotherms deviate from the latitudes to the north over the ocean and
to the south over the continent.
2. The thermal equator  lies generally to the north of the geographical equator.
3. The highest range of annual temperatures is found in the Siberian region.

Which of the statements given above are correct?

Q. तापमान के वै श्विक वितरण के सं दर्भ में निम्नलिखित कथनों पर विचार कीजिए:

1. जनवरी में समताप रे खा अक्षां श से उत्तर में महासागर तथा दक्षिण में महाद्वीप की ओर विचलन करती
है ।
2. तापीय भूमध्यरे खा आमतौर पर भौगोलिक भूमध्य रे खा के उत्तर में स्थित होती है ।
3. साइबे रियाई क्षे तर् में वार्षिक तापमान की उच्चतम सीमा पाई जाती है ।

उपर्युक्त कथनों में से कौन से सही हैं ?

A  1 and 2 only
केवल 1 और 2
B  2 and 3 only
केवल 2 और 3

C  1 and 3 only
केवल 1 और 3

D  1, 2 and 3
1, 2 और 3
Solution : D

Explanation:

Statement 1 is correct: In general, the isotherms are generally parallel to the


latitude. In January the isotherms deviate to the north over the ocean and to the
south over the continent.

Statement 2 is correct: Because of dominance of land over water in the Northern


hemisphere, it is warmer. So the thermal equator lies generally to the north of the
geographical equator.

Statement 3 is correct: The highest range of annual temperature is more than 60° C


over Siberia.

व्याख्या:

कथन 1 सही है : सामान्यतः समताप रे खा अक्षां श के समानांतर होती है । जनवरी में , समताप
रे खा उत्तर में महासागर की ओर तथा दक्षिण में महाद्वीप की ओर विचलन करती है ।

कथन 2 सही है : उत्तरी गोलार्ध में जल पर भूमि की प्रधानता के कारण यह गर्म है । अतः
तापीय भूमध्य रे खा आमतौर पर भौगोलिक भूमध्य रे खा के उत्तर में स्थित होती है ।

कथन 3 सही है : साइबे रिया में वार्षिक तापमान की उच्चतम सीमा 60° C से अधिक है ।
 
 0 s

92
Q. With reference to ‘Globba Andersonii’ a rare endangered plant species recently
rediscovered, consider the following statements:

1. It is endemic to the Western Ghats.


2. It is a Critically Endangered species as per IUCN Red List.
3. It is commonly known as dancing ladies.
Which of the statements given above is/are correct?

Q. ’ग्लोब्बा एं डरसोनी’ (Globba Andersonii), एक दुर्लभ लु प्तप्राय पौधे की प्रजाति, जिसे हाल
ही में फिर से खोजा गया है , के सं दर्भ में निम्नलिखित कथनों पर विचार कीजिए:

1. यह पश्चिमी घाट में पाया जाता है ।


2. IUCN की रे ड लिस्ट के अनु सार यह एक गं भीर रूप से सं कटग्रस्त प्रजाति है ।
3. इसे आमतौर पर डां सिंग ले डीज़ (Dancing Ladies) के रूप में जाना जाता है ।

उपर्युक्त कथनों में से कौन सा/से सही है /हैं ?

A  1 only
केवल 1

B  2 and 3 only
केवल 2 और 3

C  3 only
केवल 3

D  1, 2 and 3
1, 2 और 3
Solution : C

Explanation:

Botanists have rediscovered a rare and critically endangered plant species named
Globba Andersonii from the Sikkim Himalayas near the Teesta river valley region
after a gap of nearly 140 years. It is characterised by white flowers, non-appendaged
anthers and a yellowish lip.

Statement 1 is incorrect: It is restricted mainly to the Teesta River Valley


region which includes the Sikkim Himalayas and Darjeeling hill ranges.

Statement 2 is incorrect: It is listed as Endangered as per IUCN Red List of


Threatened Species.

Statement 3 is correct: It is commonly known as dancing ladies or swan flowers.

व्याख्या:

लगभग 140 वर्षों के अं तराल के बाद वनस्पति विज्ञानियों ने तीस्ता नदी घाटी क्षे तर् के पास
सिक्किम हिमालय से ग्लोब्बा एं डरसोनी (Globba Andersonii) नामक एक दुर्लभ और गं भीर
रूप से लु प्तप्राय पौधे की प्रजाति को फिर से खोजा है । इसकी विशे षता सफेद फू ल, गै र-
उपां गित (Non-Appendaged) परागकोष (पु ं केसर का भाग जिसमें पराग होता है ) और पीला
अधर है ।
कथन 1 गलत है : यह मु ख्य रूप से  तीस्ता नदी घाटी क्षे तर्  तक सीमित है जिसके अं तर्गत
सिक्किम हिमालय और दार्जिलिं ग पहाड़ी क्षे तर् शामिल हैं ।

कथन 2 गलत है : इसे  IUCN रे ड लिस्ट के अनुसार लु प्तप्राय (Endangered) के रूप में
सूचीबद्ध किया गया है ।

कथन 3 सही है : इसे आमतौर पर 'डां सिंग ले डीज़’ (Dancing Ladies)' या 'स्वान फ्लावर्स'
(Swan Flowers) के रूप में जाना जाता है ।
 
 0 s

93
Q. Consider the following statements:

1. The San Andreas fault, along the western coast of the USA, is the result of divergent
plate boundaries.
2. The Indonesian archipelago is the result of convergent plate boundaries.
3. The Alps mountains are the result of convergent plate boundaries.

Which of the statements given above is/are correct?

Q. निम्नलिखित कथनों पर विचार कीजिए:

1. सं युक्त राज्य अमे रिका के पश्चिमी तट के साथ ही सै न एं ड्रियास भ्रंश (San Andreas fault),
अपसारी प्ले ट सीमाओं का परिणाम है ।
2. इं डोने शियाई द्वीपसमूह अभिसारी प्ले ट सीमाओं का परिणाम है ।
3. आल्प्स पर्वत अभिसारी प्ले ट सीमाओं का परिणाम हैं ।

उपर्युक्त कथनों में से कौन सा/से सही है /हैं ?

A  1 only
केवल 1

B  2 only
केवल 2

C  2 and 3 only
केवल 2 और 3

D  1, 2 and 3
1, 2 और 3
Solution : C
Explanation:

Statement 1 is incorrect: San Andreas Fault along the western coast of the USA is
the best example for a transform plate boundaries. In transform plate boundaries,
the two plates slide past against each other, and there is no creation or destruction of
landform but only deformation of the existing landform.

Statement 2 is correct: Indonesian archipelago is located along the plate margins. It


is formed due to ocean-ocean convergence or convergent plate boundaries. It is
formed due to the convergence between the Sunda Oceanic plate (part of the Eurasian
plate) and Indo-Australian plate.

Statement 3 is correct: The Alps mountains are the result of convergent plate


(continent-continent) boundaries. Fold mountains are formed due to convergence
between two continental plates (Himalayas, Alps) or between an oceanic and a
continental plate (the Rockies, Andes).

व्याख्या:

कथन 1 गलत है : सं युक्त राज्य अमे रिका के पश्चिमी तट के साथ ही सै न एं ड्रियास
भ्रंश रूपांतरित प्ले ट सीमाओं का सबसे अच्छा उदाहरण है । रूपांतरित प्ले ट सीमाओं में दो
प्ले टें परस्पर रगड़ खाती हैं , और इसमें स्थलाकृति का निर्माण या विनाश नहीं होता है , केवल
मौजूदा स्थलाकृति का विरूपण होता है ।

कथन 2 सही है : इं डोने शियाई द्वीपसमूह प्ले ट मार्जिन के पास स्थित हैं । यह महासागर-
महासागर अभिसरण या अभिसारी प्ले ट सीमाओं के कारण बना है । इसका निर्माण सुं डा
महासागरीय प्ले ट (यूरेशियन प्ले ट का हिस्सा) और इं डो-ऑस्ट् रेलियाई प्ले ट के बीच
अभिसरण से हुआ है ।

कथन 3 सही है : आल्प्स पर्वत अभिसारी प्ले ट (महाद्वीप-महाद्वीप) सीमाओं का परिणाम हैं ।
वलित पर्वतों का निर्माण दो महाद्वीपीय प्ले टों (हिमालय, आल्प्स) या एक महासागरीय और
एक महाद्वीपीय प्ले ट (रॉकीज, एं डीज) के बीच अभिसरण के कारण होता है ।
 0 s

94
Q. With reference to the series of straits which connect the Black sea to the Atlantic
ocean, consider the following pairs:
 
S.no Strait Connecting
1. Gibraltar Atlatinc ocean-Mediterrarian Sea
2. Bosphorus Marmara Sea-Aegean Sea
3. Dardanelles Black Sea-Marmara Sea
Which of the pairs given above is/are correctly matched?

Q. काले सागर को अटलां टिक महासागर से जोड़ने वाली जलडमरूमध्य की श्रखृं ला के सं दर्भ में
निम्नलिखित यु ग्मों पर विचार कीजिए:
 
क् रम संख्या जलडमरूमध्य जोड़ता है
1. जिब्राल्टर अटलां टिक महासागर-भूमध्य सागर
2. बोस्पोरुस मरमरा सागर-एजियन सागर
3. डारडे नेल्स काला सागर-मरमरा सागर

उपर्युक्त यु ग्मों में से कौन सा/से सु मेलित है /हैं ?

A  1 only
केवल 1

B  2 and 3 only
केवल 2 और 3

C  1 and 3 only
केवल 1 और 3

D  1, 2 and 3 only


1, 2 और 3
Solution : A

Explanation:

Black sea is connected to the distant waters of the Atlantic Ocean by the Bosporus, the
Sea of Marmara, the Dardanelles, the Aegean Sea, and the Mediterranean Sea.

Pair 1 is correctly matched: Strait of Gibraltar connects the Atlantic ocean and


Mediterrarian Sea.

Pair 2 is incorrectly matched: Strait of Bosphorus connects Black Sea and Marmara


Sea.

Pair 3 is incorrectly matched: Strait of Dardanelles connects Marmara Sea and


Aegean Sea.
व्याख्या:

काला सागर बोस्पोरुस , मरमरा सागर, डारडने ल, एजियन सागर और भूमध्य सागर के द्वारा
अटलां टिक महासागर के सूदरू वर्ती जलीय क्षे तर् से जु ड़ा हुआ है ।

यु ग्म 1 सुमेलित है : जिब्राल्टर जलडमरूमध्य अटलां टिक महासागर और भूमध्य सागर को


जोड़ता है ।

यु ग्म 2 सुमेलित नहीं है : बोस्पोरुस जलडमरूमध्य काला सागर और मरमरा सागर को जोड़ता
है ।

यु ग्म 3 सुमेलित नहीं है : डारडे नेल्स जलडमरूमध्य मरमरा सागर और एजियन सागर को
जोड़ता है ।

 0 s

95
Q. With reference to the temperature inversion, which of the following statements
is/are correct?

1. Temperature inversion is normal throughout the year in polar regions.


2. It is a reason for the poor air quality of Delhi during winter.
3. It protects the plants from frost damage in the mountain areas.

Select the correct answer using the codes given below:

Q. तापमान व्यु त्क् रमण (Temperature Inversion) के सं दर्भ में निम्नलिखित कथनों में से कौन
सा/से सही है /हैं ?

1. ध्रुवीय क्षे तर् ों में पूरे वर्ष तापमान व्यु त्क् रमण एक सामान्य प्रक्रिया है ।
2. यह सर्दियों के दौरान दिल्ली की खराब वायु गु णवत्ता का एक कारण है ।
3. यह पर्वतीय क्षे तर् ों में पौधों को ठं ढ से होने वाले नु कसान से बचाता है ।

निम्नलिखित कू टों का उपयोग करके सही उत्तर चु निए:

A  1 and 3 only
केवल 1 और 3

B  2 only
केवल 2

C  2 and 3 only
केवल 2 और 3

D  1, 2 and 3
1, 2 और 3
Solution : D

Explanation:

Normally, temperature decreases with increase in elevation. It is called the normal


lapse rate. At times, the situation is reversed and the normal lapse rate is inverted. It
is called Inversion of temperature. A long winter night with clear skies and still air is
an ideal situation for inversion. The heat of the day is radiated off during the night,
and by early morning hours, the earth is cooler than the air above.

Statement 1 is correct: It is over the polar regions, not tropics, a temperature


inversion is normal throughout the year.

Statement 2 is correct: Temperature inversion promotes stability in the lower layers


of the atmosphere. Smoke and dust particles get collected beneath the inversion layer
and spread horizontally to fill the lower strata of the atmosphere. Dense fog in
mornings is a common occurrence, especially during the winter season. Hence,
pollutants from stubble burning, industries, vehicular exhaust etc. gets trapped in the
lower atmosphere during winter, thus reducing the air quality of Delhi.

Statement 3 is correct: The inversion takes place in hills and mountains due to air
drainage. Cold air at the hills and mountains, produced during the night. Cold air is
denser than warm air, so it flows downhill under the influence of gravity and
accumulates in low spots. It protects plants from frost damages. Frost susceptible
plants grow where there is good air drainage and experience a considerably longer
growing season. Gardens on slopes can be modified to advantage by having a low-
permeable hedge above and a very permeable one on the lower boundary.

Figure: The creation of frost


pockets: (a) natural hollows on the sides of valleys. (b) effect of solid barriers
preventing the drainage of cold air.

व्याख्या:

आम तौर पर, ऊंचाई में वृ दधि ् के साथ तापमान घटता है । इसे  सामान्य हर् ास दर (Normal
Lapse Rate) कहा जाता है । कई बार, स्थिति विपरीत हो जाती है और सामान्य ह्रास दर
उलट जाती है । इसे तापमान व्यु त्क् रमण (Inversion of Temperature) कहा जाता है । सर्दियों
की लं बी रात के साथ साफ आसमान और स्थिर हवा तापमान व्यु त्क् रमण के लिए एक आदर्श
स्थिति है ।दिन की गर्मी रात के दौरान विकरित होती है , और सु बह के समय तक पृ थ्वी ऊपर की
हवा की तु लना में ठं डी हो जाती है ।

कथन 1 सही है : यह ध्रुवीय क्षे तर् ों में होता है , उष्ण कटिबं ध में नहीं, पूरे वर्ष तापमान का
व्यु त्क् रमण एक सामान्य प्रक्रिया है ।
कथन 2 सही है : तापमान व्यु त्क् रमण से वायु मं डल की निचली परतों में स्थिरता को बढ़ावा
मिलता है । धु एं और धूल के कण व्यु त्क् रमण परत के नीचे एकत्र हो जाते हैं और वायु मं डल के
निचले हिस्से में क्षै तिज रूप से फैलते हैं । सर्दियों के मौसम में सु बह के समय घना कोहरा एक
आम घटना है । इसलिए, पराली के जलने , उद्योगों, वाहनों के धु एं आदि से निकलने वाले
ू क सर्दियों के दौरान निचले वातावरण में रह जाते हैं , जिससे दिल्ली की वायु गु णवत्ता
प्रदष
खराब हो जाती है ।

कथन 3 सही है : पहाड़ियों और पर्वतों में व्यु त्क् रमण वायु के निकास के कारण होता है ।पहाड़ों
पर ठं डी हवा रात में उत्पन्न होती है । यह गर्म हवा की तु लना में सघन होती है , इसलिए यह
गु रुत्वाकर्षण के प्रभाव में नीचे की ओर बहती है और निचले स्थानों पर एकत्रित हो जाती है ।
यह पौधों को पाले के नु कसान से बचाती है । पाले के प्रति अतिसं वेदनशील पौधे वहाँ बढ़ते हैं
जहाँ हवा की अच्छी निकासी होती है और उनकी वृ दधि ् में अधिक समय लगता है । ढलानों के
बागों को ऊपर कम पारगम्य बाड़ और निचली सीमा पर अधिक पारगम्य बाड़ लगाकर बे हतर
किया जा सकता है ।

चित्र: पाले की उत्पत्ति: (ए) घाटियों


के किनारों पर प्राकृतिक कोटर। (b) ठंडी हवा के निकास को रोकने वाले ठोस अवरोधों का
प्रभाव। 
 0 s

96
Q. Recently, which of the following state biodiversity board has declared Indigenous
Mango Heritage Area?

Q. हाल ही में , निम्नलिखित में से किस राज्य में जै व विविधता बोर्ड ने स्वदे शी आम धरोहर क्षे तर्
(Indigenous Mango Heritage Area) घोषित किया है ?
A  Kerala
केरल

B  Karnataka
कर्नाटक

C  Maharashtra
महाराष्ट् र

D  Gujarat
गु जरात
Solution : A

Explanation:

Kerala state biodiversity board has declared Kannapuram panchayat in Kannur district


as an ‘Indigenous Mango Heritage Area’. The panchayat is home to over 200 different
varieties of mangoes.

व्याख्या:

केरल राज्य जै व विविधता बोर्ड ने कन्नूर जिले की कन्नपु रम पं चायत को स्वदे शी आम धरोहर
क्षे तर् (Indigenous Mango Heritage Area)घोषित किया है । पं चायत में 200 से अधिक आम
की प्रजातियाँ हैं ।
 0 s

97
Q. With reference to the oceanic salinity, which of the following statements is/are
correct?

1. Salinity variation at the ocean bottom is maximum.


2. There is a sharp increase in the salinity in the halocline layer.
3. Salinity always decreases from lower to higher latitudes.
4. Effect of salinity is greater in determining the density of water than the temperature.

Select the correct answer using the codes given below:

Q. महासागरीय लवणता के सं दर्भ में निम्नलिखित में से कौन सा/से कथन सही है /हैं ?

1. समु दर् तल पर लवणता की विविधता अधिकतम होती है ।


2. ् होती है ।
है लोक्लाइन (Halocline) परत में लवणता में ते ज वृ दधि
3. निम्न से उच्च अक्षां शों की ओर लवणता हमे शा कम होती जाती है ।
4. जल के घनत्व को निर्धारित करने में तापमान की तु लना में लवणता का प्रभाव अधिक होता है ।
निम्नलिखित कू टों का उपयोग करके सही उत्तर चु निए:

A  1 and 4 only
केवल 1 और 4

B  2 and 4 only
केवल 2 और 4

C  2 only
केवल 2

D  1, 2, 3 and 4
1, 2, 3 और 4
Solution : C

Explanation:

Statement 1 is incorrect: Salinity at the bottom of the ocean is constant. There are
little changes in the salinity of the water at the deep waters due to less precipitation
and evaporation from the lower layers of the ocean water.

Statement 2 is correct: In the halocline layer which is just situated beneath the
surface layer, there is a sharp increase in the salinity with the depth.

Statement 3 is incorrect: Generally, due to higher evaporation in lower latitudes,


salinity is higher. In higher latitudes, due to melting of fresh snow and ice caps,
salinity is comparatively lower. However, the salinity is maximum at around 20 to 40
degree latitude and from there decreases towards equator and higher latitudes.

Statement 4 is incorrect: If there are two layers of water with the same salinity, then
the warmer water will float on top of the colder water. If the layer with higher salinity
is warmer than the layer with lower salinity, then the layer of water with higher
salinity floats on top of the water with lower salinity. Hence temperature has a greater
effect on the density of water than salinity.

व्याख्या:

कथन 1 गलत है : समु दर् के तल पर लवणता स्थिर होती है । कम वर्षा और वाष्पीकरण के कारण
समु दर् के जल की निचली परतों में लवणता में बहुत कम परिवर्तन होता है ।

कथन 2 सही है : है लोक्लाइन परत में , जो सतह की परत के ठीक नीचे स्थित होती है , बढ़ती
् होती है ।
गहराई के साथ लवणता में ते ज वृ दधि

कथन 3 गलत है : आम तौर पर, कम अक्षां शों में अधिक वाष्पीकरण के कारण लवणता अधिक
होती है । उच्च अक्षां शों में ताजे बर्फ के पिघलने के कारण लवणता तु लनात्मक रूप से कम होती
है । हालाँ कि, लवणता 20 से 40 डिग्री अक्षां श पर अधिकतम होती है और वहाँ से भूमध्य रे खा
और उच्च अक्षां शों की ओर घटती जाती है ।

कथन 4 गलत है : यदि समान लवणता की जल की दो परतें हैं , तो गर्म पानी ठं डे पानी के ऊपर
तै रने लगे गा। यदि उच्च लवणता वाली परत कम लवणता वाली परत की तु लना में गर्म होती
है , तो उच्च लवणता वाली पानी की परत कमलवणता वाले पानी के ऊपर तै रती है । इसलिए
लवणता की तु लना में पानी के घनत्व पर तापमान का अधिक प्रभाव पड़ता है ।
 0 s

98
Q. Consider the following statements regarding the Geomorphic Processes:

1. They cause physical stresses and chemical action for the formation of various
landforms on the earth.
2. All of the energy for the exogenic processes is derived from the sun.
3. Diastrophism is the process of wearing down relief.

Which of the statements given above is/are correct?

Q. भू-आकृतिक प्रक्रियाओं (Geomorphic Processes) के सं दर्भ में निम्नलिखित कथनों पर


विचार कीजिए:

1. ये पृ थ्वी पर विभिन्न भू-आकृतियों के निर्माण के लिए भौतिक तनाव और रासायनिक अभिक्रिया के


कारक होते हैं ।
2. बहिर्जनिक प्रक्रियाओं (Exogenic Processes) के लिए सभी ऊर्जा सूर्य से प्राप्त होती है ।
3. पटल विरूपण (Diastrophism) भौगोलिक सं रचनाओं के विखं डन की प्रक्रिया है ।

उपर्युक्त कथनों में से कौन सा/से सही है /हैं ?

A  1 only
केवल 1

B  2 and 3 only
केवल 2 और 3

C  1 and 3 only
केवल 1 और 3

D  1, 2 and 3
1, 2 और 3
Solution : A

Explanation:

Statement 1 is correct: The endogenic and exogenic forces causing physical stresses


and chemical actions on earth materials and bringing about changes in the
configuration of the surface of the earth are known as geomorphic processes.

Statement 2 is incorrect: The exogenic processes derive their energy from the


atmosphere determined by the ultimate energy from the sun and also the gradients
created by tectonic factors. Gravitational force acts upon all earth materials having a
sloping surface and tends to produce movement of matter in the downslope direction.

Statement 3 is incorrect: Diastrophism is the process that moves, elevates or builds


up portions of the earth’s crust. Degradation is the process where the actions of
exogenic forces result in wearing down of relief/elevations.

व्याख्या:

कथन 1 सही है : अं तर्जनित और बहिर्जनिक बल पृ थ्वी पर भौतिक तनाव और रासायनिक


क्रियाओं के कारक होते   हैं तथा पृ थ्वी की सतह के विन्यास में परिवर्तन लाने वाली भू-आकृति
प्रक्रियाओं (Geomorphic Processes) के रूप में जाने जाते हैं ।

कथन 2 गलत है : बहिर्जनिक प्रक्रियाएँ सूर्य की ऊर्जा द्वारा निर्धारित वातावरण के साथ-
साथ विवर्तिनीकी कारकों द्वारा निर्मित प्रवणता से भी अपनी ऊर्जा प्राप्त करती
हैं । गु रुत्वाकर्षण बल पृ थ्वी पर विद्यमान उन सभी वस्तु ओं पर कार्य करता है जिनकी सतह
ढलान वाली होती है और यह उन्हें नीचे की दिशा में गतिशील करता है ।

कथन 3 गलत है : पटल विरूपण (Diastrophism) वह प्रक्रिया है जो भूपर्पटी के कुछ हिस्सों


को स्थानांतरित करती है , ऊपर उठाती है या उनका निर्माण करती है । हर् ास
(Degradation) वह प्रक्रिया है जिसमें  बहिर्जनिक बलों की क्रियाओं के परिणामस्वरूप
भौगोलिक संरचनाओं का विखं डन होता है ।
 0 s

99
Q. With reference to the Cyclones, consider the following statements:

1. Tropical cyclones occur only over the seas, while extratropical cyclones occur both
over land and seas.
2. Tropical cyclones move from west to east, while extratropical cyclones move from
east to west.
3. A clear frontal system is present in the extratropical cyclones, while it is absent in
tropical cyclones.
4. Tropical cyclones affect a much larger area than the extratropical cyclones.

Select the correct answer using the codes given below:

Q. चक् रवातों के सं दर्भ में निम्नलिखित कथनों पर विचार कीजिए:


1. उष्णकटिबं धीय चक् रवात (Tropical Cyclones) केवल समु दर् के ऊपर बनते हैं , जबकि बहिरूष्ण
कटिबं धीय चक् रवात (Extra-Tropical Cyclones) भूमि और समु दर् दोनों के ऊपर बनते हैं ।
2. उष्णकटिबं धीय चक् रवात पश्चिम से पूर्व की ओर बढ़ते हैं , जबकि बहिरूष्ण कटिबं धीय चक् रवात पूर्व
से पश्चिम की ओर बढ़ते हैं ।
3. बहिरूष्ण कटिबं धीय चक् रवात में एक स्पष्ट अग्र प्रणाली विद्यमान होती है , जबकि यह
उष्णकटिबं धीय चक् रवातों में अनु पस्थित होती है ।
4. उष्णकटिबं धीय चक् रवात बहिरूष्ण कटिबं धीय चक् रवात की तु लना में बहुत बड़े क्षे तर् को प्रभावित
करते हैं ।

निम्नलिखित कू टों का उपयोग करके सही उत्तर चु निए:

A  2 and 3 only
केवल 2 और 3

B  1 and 3 only
केवल 1 और 3

C  1, 3 and 4 only


केवल 1, 3 और 4

D  1, 2, 3 and 4
1, 2, 3 और 4
Solution : B

Explanation:

Statement 1 is correct: Tropical cyclones occur only over the seas and on reaching
the land they dissipate and extratropical cyclones occur both over land and seas.

Statement 2 is incorrect: Extratropical cyclones move from west to east and tropical


cyclones move from east to west.

Statement 3 is correct: Extratropical cyclones form along the polar front. Hence, a


clear frontal system is present in the extratropical cyclones, the same is absent in
tropical cyclones.

Statement 4 is incorrect: It is the extratropical cyclones that affect larger areas than
the tropical cyclones and the tropical cyclones have greater wind speed than the
extratropical cyclones.

व्याख्या:

कथन 1 सही है : उष्णकटिबं धीय चक् रवात (Tropical Cyclones) केवल समु दर् के ऊपर बनते हैं
और भूमि तक पहुँचने पर वे बिखर जाते हैं , तथा बहिरूष्ण कटिबं धीय चक् रवात (Extra-
Tropical Cyclones) भूमि और समु दर् दोनों के ऊपर उत्पन्न होते हैं ।
कथन 2 गलत है : बहिरूष्ण कटिबं धीय चक् रवात पश्चिम से पूर्व की ओर बढ़ते हैं और
उष्णकटिबं धीय चक् रवात पूर्व से पश्चिम की ओर बढ़ते हैं ।

कथन 3 सही है : बहिरूष्ण कटिबं धीय चक् रवात ध्रुवीय क्षे तर् ों में बनते हैं । इसलिए, उनमें
अग्र प्रणाली विद्यमान होती है , जबकि यह उष्णकटिबं धीय चक् रवातों में अनु पस्थित होती
है ।

कथन 4 गलत है : बहिरूष्ण कटिबं धीय चक् रवात उष्णकटिबं धीय चक् रवातों की तु लना में बड़े
क्षे तर् ों को प्रभावित करते हैं और उष्णकटिबं धीय चक् रवातों में बहिरूष्ण कटिबं धीय चक् रवात
की तु लना में हवा की गति अधिक होती है ।
 0 s

100
Q. With reference to the Green–Ag project, consider the following statements:

1. It is funded by the Global Environment Facility.


2. It is being implemented only in the Himalayan states.
3. It is being implemented with assistance from the Food and Agriculture Organisation
(FAO).

Which of the statements given above is/are correct?

Q. 'ग्रीन एजी परियोजना' (Green-Ag Project) के सं दर्भ में निम्नलिखित कथनों पर विचार
कीजिए:

1. यह वै श्विक पर्यावरण सु विधा (Global Environment Facility– GEF) द्वारा वित्त पोषित है ।
2. इसे केवल हिमालयी राज्यों में लागू किया जा रहा है ।
3. इसे खाद्य और कृषि सं गठन (Food and Agriculture Organisation-FAO) की सहायता से लागू
किया जा रहा है ।

उपर्युक्त कथनों में से कौन सा/से सही है /हैं ?

A  1 only
केवल 1

B  2 and 3 only
केवल 2 और 3

C  1 and 3 only
केवल 1 और 3

1, 2 and 3

1, 2 और 3
 
Solution : C

Explanation:

The Green-Ag project aims to catalyze the sustainable transformation of Indian


agriculture without compromising the country’s food security and farmers’ income. It
was recently launched in Mizoram.

Statement 1 is correct: Green-Ag project is being funded by the Global Environment


Facility (GEF).

Statement 2 is incorrect: The project is being implemented in high-conservation-


value landscapes of five States namely:

1. Madhya Pradesh: Chambal Landscape,


2. Mizoram: Dampa Landscape,
3. Odisha: Similipal Landscape,
4. Rajasthan: Desert National Park Landscape and
5. Uttarakhand: Corbett-Rajaji Landscape.

Statement 3 is correct: The Ministry of Agriculture & Farmers Welfare is


implementing the project in collaboration with the Food and Agriculture Organisation
(FAO).

व्याख्या:

'ग्रीन एजी परियोजना' (Green-Ag Project) का उद्दे श्य दे श की खाद्य सु रक्षा और किसानों की
आय से समझौता किए बिना भारतीय कृषि के सतत परिवर्तन को उत्प्रेरित करना है । इसे हाल
ही में मिजोरम में प्रारं भ किया गया था।

कथन 1 सही है : 'ग्रीन एजी परियोजना' को वै श्विक पर्यावरण सु विधा (Global Environment
Facility– GEF) द्वारा वित्त पोषित किया जा रहा है ।

कथन 2 गलत है : इस परियोजना को पाँच राज्यों के निम्नलिखित उच्च सं रक्षण मूल्य वाले
क्षे तर् ों में क्रियान्वित किया जा रहा है :

1. मध्य प्रदे श: चं बल क्षे तर् ,


2. मिजोरम: डम्पा क्षे तर् ,
3. ओडिशा: सिमिलिपाल क्षे तर् ,
4. राजस्थान: डे जर्ट ने शनल पार्क क्षे तर् , और
5. उत्तराखं ड: कॉर्बे ट-राजाजी क्षे तर् ।

कथन 3 सही है : कृषि और किसान कल्याण मं तर् ालय, खाद्य और कृषि सं गठन (Food and
Agriculture Organisation-FAO) के साथ मिलकर इस परियोजना को लागू कर रहा है ।
 0 s

 Time Management Avg Time Spent Per Question 0 s


v

You might also like